Você está na página 1de 234

CNS-System

Wise 1700-by Sush and Team. 2016


Susmita, Asad, Manu, Saima, Zohaib, Savia, Shanu, Mona, Manisha, Sitara, Samreena, Sami and Komal


Dear Plabber,


• This first ever System Wise 1700 document was created thanks to 3 months of daily hard
work by the PLAB Skype group ‘Unity’ which was brought together by Dr Susmita
Chowdhury.
• Please ignore the old versions posted by my new skype member Murtaza as he did so
without permission.

The team members were:


& Susmita (Lead/most ignorant as she is working full time in public health for 13 years)
& Asad (Invaluable in IT and all types of support/the heart of the group)
& Manu (Volunteered to solve more questions/pathologist/amazing genuine person)
& Saima (Most concise clear notes/ photographic memory)
& Zohaib (Great research/a surgeon)
& Savia (Great research/multi-tasker with two little ones)
& Shanu (Very helpful after her March exam for those appearing in June)
& Mona (Great contributor in discussions)
& Manisha (Gyne/great discussion contributor)
& Sitara (Good discussion contributor)
& Samreena (Stayed a shorter time but great)
& Sami (Contributed the most early on but too brilliant for the group/still great friends)
& Komal (Knowledgeable sweet supportive girl)

• The main purpose was to break down the 1700 Q Bank System wise.

• We did our own reliable research for the options (OHCM/Patient info etc.) and concluded
these keys below on skype. This can save you 100s of hours of research. But I suggest you
also do your own.

• 90% of the document consists of Unity research. We also added information from other
circulating documents and they are referenced as Dr Khalid/Dr Rabia (and her Team).

• However, several keys may be ‘incorrect’ and so please use your own judgment as we take
no responsibility. I suggest cross checking with Dr Khalid’s latest keys (a few of which are still
debatable). Finally decide on your own key.

• Sorry if some members failed to make their answers thorough. The highlights are mostly as
per what the team members wanted to highlight. Blank tables to be ignored.

• Note that some 1700 Questions are missing from here (when members did not do their
share). Questions may not be in order due to merging of documents and there is excess
information than required. Read as much as needed.

• This has been circulated by our team as a generous contribution to the Plabbers’ success and
must not be ‘sold’.

1 Good luck and best wishes: Sush and Team
CNS-System Wise 1700-by Sush and Team. 2016
Susmita, Asad, Manu, Saima, Zohaib, Savia, Shanu, Mona, Manisha, Sitara, Samreena, Sami and Komal

Q: 6 A 67yo man after a stroke, presents with left sided ptosis and constricted
pupil. He also has loss of pain and temp on the right side of his body and left
side of his face. Which part of the brain is most likely affected?

a. Frontal cortex
b. Cerebellum
c. Pons
d. Medulla
e. Parietal cortex


Clincher(s) Stroke, left side ptosis & constricted pupil, loss of pain & temp on right side
of body
A
B
C
D Correct Answer
E
KEY D Loss of Pain & Temp on right side of the body ! Lat Medullary Syndrome
Additional Lateral Medullary Syndrome was originally believed to be the result of
Information occlusion of the posterior inferior cerebellar artery. It is now recognised that
the picture described may also result from partial occlusion of the basilar
artery or occlusion of proximal arteries such as one vertebral artery.
The terms posterior inferior cerebellar artery stroke and lateral medullary
syndrome are synonyms. It may also be referred to by the eponym
"Wallenberg's syndrome".
The syndrome is produced by a wedge-shaped infarction of the lateral medulla
and inferior surface of the cerebellum.

The clinical features of lateral medullary syndrome can be divided into those
resulting from brainstem or cerebellar dysfunction:
Cerebellar features:
• ipsilateral limb ataxia
• vertigo
• nystagmus to the side of the lesion - due to damage to the vestibulo-ocular
connections
Brain stem features:
• sudden onset of dizziness and vomiting - due to the involvement of
vestibular and vagal nuclei respectively
• dysphagia and dysarthria - due to lesion to the nucleus ambiguus and vagal
nuclei
• ipsilateral Horner's syndrome
• ipsilateral facial sensory loss - pain and temperature
• ipsilateral pharyngeal and laryngeal paralysis - cranial IX and X palsies
• contralateral sensory loss - pain and temperature of the limbs and trunk

2
CNS-System Wise 1700-by Sush and Team. 2016
Susmita, Asad, Manu, Saima, Zohaib, Savia, Shanu, Mona, Manisha, Sitara, Samreena, Sami and Komal

Horner's syndrome is a triad of features resulting from interruption of the
sympathetic pathway from the hypothalamus to the orbit, as follows:
• slight ptosis
• pupillary miosis:
due to paralysis of the sympathetically innervated Muller's muscle
which normally dilates the pupil
• anhydrosis over the forehead

3
CNS-System Wise 1700-by Sush and Team. 2016
Susmita, Asad, Manu, Saima, Zohaib, Savia, Shanu, Mona, Manisha, Sitara, Samreena, Sami and Komal




























Reference GPnotebook OHCM 726
Dr Khalid/Rabia Ans. 1. The key is D. Medulla.
Ans. 2. The name of the condition is “Lateral medullary syndrome” [ipsilateral
Horner syndrome and contralateral loss of pain and temperature sense]

Lateral medullary syndrome, also known as Wallenberg's syndrome, occurs
following occlusion of the posterior inferior cerebellar artery

Cerebellar features

4
CNS-System Wise 1700-by Sush and Team. 2016
Susmita, Asad, Manu, Saima, Zohaib, Savia, Shanu, Mona, Manisha, Sitara, Samreena, Sami and Komal

• ataxia (the loss of full control of bodily movements)
• nystagmus (rapid involuntary movements of the eyes)
Brainstem features
• ipsilateral: dysphagia, facial numbness, cranial nerve palsy e.g. Horner's
• contralateral: limb sensory loss


Q: 16 A 22yo man has a reduced conscious level and a fixed dilated pupil after being
involved in a MVC. Choose the single most appropriate option?

a. Facial nerve
b. Oculomotor nerve
c. Olfactory nerve
d. Optic nerve
e. Trigeminal nerve


Clincher(s) MVC, reduced conscious level, fixed dilated pupil
A
B Dilated pupil and absent reflexes
C
D Visual loss
E
KEY B Oculomotor Nerve Palsy incapable to move
Additional Paralysis of the third cranial nerve affects the medial, superior, and inferior
Information recti, and inferior oblique muscles.
The eye is incapable of movement upwards, downwards or inwards, and at
rest the eye looks laterally and downwards owing to the overriding influence
of the lateral rectus and superior oblique muscles respectively. The reduced
response of levator palpebrae superioris results in ptosis - a drooping of the
upper eyelid.
A third nerve palsy with pupillary sparing is often termed a medical third palsy
and often has an ischaemic or diabetic aetiology.
Full assessment of oculomotor nerve function involves testing of movement,
reaction to light, and accommodation. If all of these are normal, "PERLA" may
be written in the notes - pupils equal, reactive to light and normal
accommodation.

Signs of optic nerve dysfunction include:
• falling acuity
• impaired colour perception
• relative afferent pupil defect
• field defect - unilateral if only one eye nerve is affected, producing a central
scotoma

5
CNS-System Wise 1700-by Sush and Team. 2016
Susmita, Asad, Manu, Saima, Zohaib, Savia, Shanu, Mona, Manisha, Sitara, Samreena, Sami and Komal

• pallor of the disc head
During the neurological examination there must be a full assessment of acuity,
fields and fundi in both eyes.



Reference http://www.gpnotebook.co.uk/simplepage.cfm?ID=-
160759804&linkID=11191&cook=yes

Dr Khalid/Rabia


Q: 22 A pt was lying down on the operating table in a position with his arms hanging
down for 3 hours. Soon after he woke up, he complains of numbness and
weakness in that hand and has limited wrist movement/wrist drop and sensory
loss over dorsum of that hand, weakness of extension of the fingers and loss of
sensation at the web of the thumb. What structure is likely to be damaged?

a. Radial nerve
b. Median nerve
c. Ulnar nerve
d. Axillary nerve
e. Suprascapular nerve


Clincher(s) Wrist drop, sensory loss, weakness of extension of fingers
A
B
C
D
E
KEY A Radial Nerve
Additional The radial nerve may be damaged anywhere in its course. It is most commonly
Information affected in the upper arm where it winds round the humerus and in the
extensor muscle compartment of the forearm affecting the posterior
interosseous branch.
Finger and wrist drop are common, their severity depending upon the site of
the lesion.

! 'crutch palsy' - due to compression of the nerve above the spiral groove by
crutches as the weight is borne in the axilla
! 'Saturday night palsy' - due to compression of the nerve in the upper part of
the arm as a result of resting the medial side of the arm against a sharp edge
such as the back of a chair for a prolonged period. The person is usually
intoxicated. It may also be seen after surgery when the anaesthetized patient
is allowed to hang over the edge of the operating theatre table.

6
CNS-System Wise 1700-by Sush and Team. 2016
Susmita, Asad, Manu, Saima, Zohaib, Savia, Shanu, Mona, Manisha, Sitara, Samreena, Sami and Komal

! humeral fracture
! rarely, dislocation of the elbow

The clinical features of radial nerve palsy (weakness) depend upon the site of
the injury.
Lesions in or above the axilla result in paralysis and wasting of all the muscles
innervated. Clinically, this is manifest as:
! weakness of forearm extension and flexion - triceps and brachioradialis
! wrist drop and finger drop - paralysis of the extensors of the wrist and digits
! weakness of the long thumb abductor and extensor muscles
! sensory loss on the dorsum of hand and forearm appropriate to the
cutaneous distribution - see radial nerve anatomy


















Reference http://patient.info/doctor/radial-nerve-lesion-c5-c8

OHCM 506
Dr Khalid/Rabia


Q: 27 A young boy fell on his outstretched hand and has presented with pain around
the elbow. He has absent radial pulse on the affected hand. What is the most
likely dx?

7
CNS-System Wise 1700-by Sush and Team. 2016
Susmita, Asad, Manu, Saima, Zohaib, Savia, Shanu, Mona, Manisha, Sitara, Samreena, Sami and Komal


a. Dislocated elbow
b. Angulated supracondylar fx
c. Undisplaced fx of radial head
d. Posterior dislocation of shoulder


Clincher(s) young boy, outstretched hand, pain around elbow, absent pulse
A
B Correct answer
C
D
E
KEY B SUPRACONDYLAR fracture most common in children
Additional
Information











A supracondylar fracture most commonly occurs in children when they fall on
the hand with the elbow bent.

The distal fragment of bone is moved backwards and is tilted, although very
rarely it is displaced anteriorly.

The principal complication of supracondylar fractures is injury to the brachial
artery, which runs in close apposition to the site of this fracture. Damage to
the artery can result in oedema in the forearm with compartment syndrome
developing.
Other complications include:
! injury to the median nerve - this is rarely permanent
! tardy ulnar palsy - a late complication caused by stretching of the ulnar
nerve
! malunion, which may result in the so-called gunstock deformity

Absent radial pulse ! Damage or occlusion of the brachial artery is the cause
of absent radial pulse.

8
CNS-System Wise 1700-by Sush and Team. 2016
Susmita, Asad, Manu, Saima, Zohaib, Savia, Shanu, Mona, Manisha, Sitara, Samreena, Sami and Komal

Treatment ! open reduction to fix the occluded artery




Reference GPNotebook

http://patient.info/doctor/elbow-injuries-and-fractures


Dr Khalid/Rabia Q. 1. What is the key?
Q. 2. What is the cause of absent radial pulse?
Q. 3. What is the immediate management?
Ans. 1. b.
Ans. 2. Damage or occlusion of the brachial artery is the cause of absent radial
pulse.
Ans. 3. Open reduction to fix the occluded artery.

This scenario is classic for supracondylar fx,the distal humerus is displaced
posteriorly causes vascular and nerve injuries which if not addressed properly
leads to volkmann contracture
posterior dislocation can compromise vascular supply but falling on
outstretched hand causes it unlikely.
Other causes of pulseless radial artery except the two above ?
congenital absence,embolization, cervical rib, thoracic outlet syndrome
falling on an outstretched hands can lead to different type of fractures , like
frac of clavicle , radius , supracondylar etc. but here it says that radial pulse is

9
CNS-System Wise 1700-by Sush and Team. 2016
Susmita, Asad, Manu, Saima, Zohaib, Savia, Shanu, Mona, Manisha, Sitara, Samreena, Sami and Komal

not felt ,which is a dreaded complication of supracondylar frac




Q: 29 A man complains of loss of sensation in his little and ring finger. Which nerve is
most likely to be involved?

a. Median nerve
b. Ulnar nerve
c. Radial nerve
d. Long thoracic nerve
e. Axillary nerve


Clincher(s) Little and ring finger
A
B Correct answer
C
D
E
KEY B Supplies little finger & ring finger

10
CNS-System Wise 1700-by Sush and Team. 2016
Susmita, Asad, Manu, Saima, Zohaib, Savia, Shanu, Mona, Manisha, Sitara, Samreena, Sami and Komal

Additional
Information



The ulnar nerve may be damaged at any point in its distribution. The most
common sites are behind the elbow and in the hand; less frequent are lesions
in the cubital tunnel and the wrist. Wasting of the small muscles of the hand
and a partial clawing of the little and ring fingers are typical.

The most prominent features of ulnar nerve palsy are wasting and weakness of
the small muscles of the hand and partial clawing of the ring and little finger.
The precise extent of deformity and disability depends on the site of the ulnar
nerve lesion.

11
CNS-System Wise 1700-by Sush and Team. 2016
Susmita, Asad, Manu, Saima, Zohaib, Savia, Shanu, Mona, Manisha, Sitara, Samreena, Sami and Komal



UMRU





12
CNS-System Wise 1700-by Sush and Team. 2016
Susmita, Asad, Manu, Saima, Zohaib, Savia, Shanu, Mona, Manisha, Sitara, Samreena, Sami and Komal


Reference OH Neurology & GPNotebook
Dr Khalid/Rabia

Q: 30 A young man complains of double vision ? OTHERS? on seeing to the right.
Which nerve is most likely to be involved?

a. Left abducens
b. Right abducens
c. Left trochlear
d. Right trochlear
e. Right oculomotor

Double vision on looking down- up oblique
Clincher(s)
A
B Correct answer
C
D
E
KEY B
Additional LR6 SO4
Information

13
CNS-System Wise 1700-by Sush and Team. 2016
Susmita, Asad, Manu, Saima, Zohaib, Savia, Shanu, Mona, Manisha, Sitara, Samreena, Sami and Komal

14
CNS-System Wise 1700-by Sush and Team. 2016
Susmita, Asad, Manu, Saima, Zohaib, Savia, Shanu, Mona, Manisha, Sitara, Samreena, Sami and Komal


Reference

15
CNS-System Wise 1700-by Sush and Team. 2016
Susmita, Asad, Manu, Saima, Zohaib, Savia, Shanu, Mona, Manisha, Sitara, Samreena, Sami and Komal

Dr Khalid/Rabia

Q: 33 A pt with cerebral metastasis has polyuria and polydipsia. What part of the
brain would be affected?

a. Cerebral cortex
b. Cerebellum
c. Diencephalon
d. Pons
e. Medulla


Clincher(s)
A
B
C Correct answer
D
E
KEY C DIENCEPHALON contains hypothalamus centre for Thirst
Additional
Information























16
CNS-System Wise 1700-by Sush and Team. 2016
Susmita, Asad, Manu, Saima, Zohaib, Savia, Shanu, Mona, Manisha, Sitara, Samreena, Sami and Komal

17
CNS-System Wise 1700-by Sush and Team. 2016
Susmita, Asad, Manu, Saima, Zohaib, Savia, Shanu, Mona, Manisha, Sitara, Samreena, Sami and Komal




The thalamus is a large, dual lobed mass of grey matter cells that are located at
the top brain stem and is superior to the hypothalamus. Sometimes they are
interconnected with the massa intermedia. It sends information received from
the different regions of the brain to the cerebral cortex.


The hypothalamus is located below the thalamus and posterior to the optic
chiasmi. It links the nervous system to the endocrine system by the pituitary
glands.

The general functions of the hypothalamus are extremely important to the
body, such as:
> pituitary gland regulation

> feeding reflexes

> Hydration

> Water preservation

> testicular function

> metabolism

> Blood pressure regulation thirst

> Thirst

18
CNS-System Wise 1700-by Sush and Team. 2016
Susmita, Asad, Manu, Saima, Zohaib, Savia, Shanu, Mona, Manisha, Sitara, Samreena, Sami and Komal

> Heart rate

> Sleep cycle

> Mood and behavioral functions

The Hippocampus is part of the forebrain that is located in the medial
temporal lobe. It belongs to the limbic system. The hippocampus is mainly
involved in memory, learning, and emotion.

The Amygdala are almond-shaped groups of neurons located in the medial
temporal lobe. The Amygdala is involved in arousal,
hormonal secretion, emotional responses, and controls the autonomic
functions that are associated with fear.


Scientists have been studying the neurological mechanisms of thirst for
decades. Early on, they discovered that the body’s primary “thirst center” in
the brain is the hypothalamus, a deep structure that also regulates body
temperature, sleep, and appetite. Special sensors in the hypothalamus are
constantly monitoring the blood’s concentration of sodium and other
substances. The hypothalamus also receives inputs from sensors in the blood
vessels that monitor blood volume and pressure. When blood volume or
pressure falls too low—from bleeding, for example, or from the excessive loss
of fluid in sweat or diarrhea, or when blood sodium concentration rises too
high from eating salty snacks, or as the result of certain diseases, the
hypothalamus sends out a strong message: Drink something. Now.
For reasons that aren’t clear, age tends to dampen this message from the
brain. In rare cases, when an aneurysm or other brain injury has destroyed the
sensors in the hypothalamus that regulate blood sodium concentration, people
can lose their sense of thirst completely. They must be prescribed a fixed
amount of fluids daily to keep their body safely hydrated.
Thirst isn’t the brain’s only response to dehydration. When the body gets low
on water, the hypothalamus increases the synthesis of an antidiuretic
hormone called vasopressin, which is secreted by the pituitary gland and
travels to the kidneys. There, it causes water to be reabsorbed from the urine,
thus reducing urine flow and conserving water in the body until more fluids are
consumed.
If the pituitary gland becomes damaged, however, or if the kidneys are unable
to respond to vasopressin, the body is unable to conserve fluids. The result can
be diabetes insipidus, a condition marked by excessive urination and extreme,
uncontrollable thirst. (Diabetes insipidus should not to be confused with
diabetes mellitus, which also causes excessive thirst and urination, but which
results from an insulin deficiency or resistance that leads to high blood
glucose.)
Until scientists understood the structure of vasopressin and its role in diabetes

19
CNS-System Wise 1700-by Sush and Team. 2016
Susmita, Asad, Manu, Saima, Zohaib, Savia, Shanu, Mona, Manisha, Sitara, Samreena, Sami and Komal

insipidus, people with the condition had to drink up to 20 quarts of water daily
to stay healthy. Today, however, diabetes insipidus can be successfully treated
with the synthetic drug demopressin, which mimics the action of vasopressin.
Recently, scientists have discovered that vasopressin secretion increases (and,
thus, less body fluid is lost) during periods of physical stress. For that reason,
many medical experts are now recommending that healthy runners drink only
when thirsty during marathons to avoid retaining excess water with potentially
dire consequences.
Reference
Dr Khalid/Rabia Ans. 1. c.
Ans. 2. Cranial diabetes insipidus.

diencephalon contains hypothalamus centre for thirst.
Other causes of polyuria and thirst.

• Psychogenic or primary polydipsia (PP).


• Diabetes mellitus.
• Other osmotic diuresis - eg, hypercalcaemia.
• Diuretic abuse.

Investigation
• high plasma osmolality, low urine osmolality
• water deprivation test




Q: 35 A 45yo female complains of pain in the inner side of her right thigh. She
was dx with benign ovarian mass on the right. Which nerve is
responsible for this pain?

a. Femoral nerve
b. Obturator nerve
c. Iliohypogastric nerve
d. Ovarian branch of splanchic nerve
e. Pudendal nerve


Clincher(s) Pain inner thigh
A
B Correct answer
C
D
E
KEY B Obturator nerve responsible for sensory innervation of the skin of medial
aspect of the thigh.

20
CNS-System Wise 1700-by Sush and Team. 2016
Susmita, Asad, Manu, Saima, Zohaib, Savia, Shanu, Mona, Manisha, Sitara, Samreena, Sami and Komal

Additional
Information












Reference
Dr Khalid/Rabia

21
CNS-System Wise 1700-by Sush and Team. 2016
Susmita, Asad, Manu, Saima, Zohaib, Savia, Shanu, Mona, Manisha, Sitara, Samreena, Sami and Komal


Q: 48 A 79yo stumbled and sustained a minor head injury 2 weeks ago. He has
become increasingly confused, drowsy and unsteady. He has a GCS of 13. He
takes warfarin for Atrial fibrillation. What is the most likely dx?

a. Extradural hemorrhage
b. Cerebellar hemorrhage
c. Epidural hemorrhage
d. Subdural hemorrhage
e. Subarachnoid hemorrhage


Clincher(s) Old man, head injury 2 weeks ago, increasingly confused, drowsy and
unsteady.
A
B
C
D
E
KEY D Sub Dural Hemorrhage
Additional Any elderly patient or alcohol or epileptic –on anticoagulants- are prone to
Information subdural hematoma

Subdural haemorrhages result from rupture of cortical bridging veins. These
connect the venous system of the brain to the large intradural venous sinuses
and lie relatively unprotected in the subdural space.

Acute subdural haemorrhage is usually associated with severe brain injury
following trauma. It can occur at any age.

Chronic subdural haemorrhage may be traumatic or may arise spontaneously.
It is most common in infants and the elderly.

Blood slowly accumulates in the subdural space, usually overlying the frontal
and parietal lobes. It becomes encysted between an outer wall of highly
vascularised, granulation tissue adherent to the dura, and a thinner, inner wall
of fibrous tissue. It is frequently bilateral.

The effects develop gradually as bleeding is venous in origin and at low
pressure.
General features include:
! fluctuating conscious level:
there may be a history of gradual onset of headaches, memory loss,
personality change, dementia, confusion and drowsiness
symptoms vary from day to day with intervening lucid periods
! focal neurological signs - often, hemiparesis of the side ipsilateral to the

22
CNS-System Wise 1700-by Sush and Team. 2016
Susmita, Asad, Manu, Saima, Zohaib, Savia, Shanu, Mona, Manisha, Sitara, Samreena, Sami and Komal

lesion - a "false" localising sign
! aphasia - if the lesion is on the left side

Acute form:
! symptoms follow immediately from an identifiable injury

Chronic form:
! latent period of weeks or months before symptoms apparent
! greater extent of cerebral distortion and herniation than acute form as the
brain can initially accommodate the developing mass lesion

The investigations of choice:
! CT scan is mandatory

A skull X-ray may reveal a fracture.

The characteristic picture of a CT scan of a subdural haemorrhage is one of a
biconcave, concentric shaped, area of increased density spreading around the
surface of the cerebral hemisphere. The contralateral ventricle may dilate
owing to obstruction at the foramen of Munro.

After 10-20 days, the subdural haematoma becomes isodense with brain. Later
it becomes relatively hypo dense.

Treatment

A craniotomy is seldom necessary to treat a subdural haematoma.
Management depends upon the age of the patient.
In infants:
! evacuate haematoma through the anterior fontanelle by repeated needle
aspiration
! subdural peritoneal shunt for persistent subdural collections

In adults:
! patients in whom conscious level is depressed:
evacuate haematoma through 2-3 burr holes, and irrigate cavity with saline
! nursing in the head down position is recommended to prevent recollection
! patients in whom conscious level is not depressed:
! consider conservative measures - steroid treatment over several weeks

23
CNS-System Wise 1700-by Sush and Team. 2016
Susmita, Asad, Manu, Saima, Zohaib, Savia, Shanu, Mona, Manisha, Sitara, Samreena, Sami and Komal

24
CNS-System Wise 1700-by Sush and Team. 2016
Susmita, Asad, Manu, Saima, Zohaib, Savia, Shanu, Mona, Manisha, Sitara, Samreena, Sami and Komal


Reference OHCM 486 , GPNotebook, Passmedicine
Dr Khalid/Rabia The gradual onset of symptoms supports the dx
Subdural hematoma as old shrunken /alcoholic brains are prone to develop
tear in the veins which bleed slowly and eventually the hematoma gets big
enough to show the symptoms
Also in this case the patient is on warfarin


Q: 53 A 23yo woman has been having pain at the base of her thumb, the pain is
reproduced when lifting her 3-month old baby or changing diapers and
also with forceful abduction of the thumb against resistance. What is the
likely cause?

a. Avascular necrosis of scaphoid
b. Trigger finger
c. De Quervain’s tenosynovitis


Clincher(s)
A
B A trigger finger (also known as stenosing tenosynovitis) is a finger that
becomes 'locked' after it has been bent (flexed). It is difficult to
straighten out without pulling on it by the other hand.
C De Quervain’s tenosynovitis
D
E
KEY C De Quervain’s tenosynovitis (inflammation of the sheath that surrounds
a tendon)

25
CNS-System Wise 1700-by Sush and Team. 2016
Susmita, Asad, Manu, Saima, Zohaib, Savia, Shanu, Mona, Manisha, Sitara, Samreena, Sami and Komal

Additional In de Quervain's disease, the sheath containing extensor pollicis brevis
Information and abductor pollicis longus tendons becomes thickened and inflamed.

! De Quervain's tenosynovitis is a stenosing tenosynovitis of the first
extensor compartment of the wrist and leads to pain on the radial
(thumb) side of the wrist and impaired function of the wrist and hand

1) condition is colloquially termed 'washerwoman's thumb' due to
the high incidence seen in women who wash clothes by the
traditional hand method
2) occurs commonly in women and usually affects 30-50-year-olds
3) occurs as the tendon sheath containing the extensor pollicis
brevis and abductor pollicis longus becomes thickened and
inflamed
4) patient presents with pain, tenderness and swelling over the
radial side of the wrist and difficulty gripping
Features
• pain on the radial side of the wrist
• tenderness over the radial styloid process
• abduction of the thumb against resistance is painful
• Finkelstein's test: with the thumb is flexed across the palm of the
hand, pain is reproduced by movement of the wrist into flexion
and ulnar deviation
Management
• analgesia
• steroid injection
• immobilisation with a thumb splint (spica) may be effective
• surgical treatment is sometimes required

Trigger Finger:




Reference GPNotebook , http://patient.info/health/tendonitis-and-tenosynovitis
Dr Khalid/Rabia Can be diagnosed by Finkelstein’s test:
The physician grasps the thumb and the hand is ulnar deviated sharply. If
sharp pain occurs along the distal radius (top of forearm, about an inch

26
CNS-System Wise 1700-by Sush and Team. 2016
Susmita, Asad, Manu, Saima, Zohaib, Savia, Shanu, Mona, Manisha, Sitara, Samreena, Sami and Komal

below the wrist), de Quervain's syndrome is likely.

Q:1060 1060. A man with hx of fall had confusion and laceration mark on the head.
Which is the most appropriate vessel affected?
a. Basilar artery
b. Middle meningeal artery
c. Vertebral artery
d. Diploic vein

Clincher(s) The history of fall and laceration and confusion
Remember extra dural haemorrhage always has history of head injury
A Berry aneurysm involved in subarachnoid haemorrhage. Gives brain stem and
cerebellar problems.
B history of fall and laceration mark suggests an extradural hemorrhage.
Therefore, artery affected would be middle meningeal.
Test : CT scan to confirm hemorrhage.
Stabilize and transfer urgently (with skilled medical and nursing escorts) to a
neurosurgical unit for clot evacuation ± ligation of the bleeding vessel. Care of
the airway in an unconscious patient and measures to ICP often require
intubation and ventilation (+ mannitol IVI
Prognosis Excellent if diagnosis and operation early. Poor if coma, pupil
abnormalities, or decerebrate rigidity are present pre-o
C Most common disease affecting vertebral artery is atherosclerosis
Mostly causes transient ischaemic attack if involved
D Spread of infections occurs via diploid veins
The drainage of emissary veins and diploid veins causes intracranial Focal
infections and septic thrombosis can occur
E
KEY B (epidural )
Additional Sub arachnoid haemorrhage is bleeding from berry aneurysm from circle Willis
Information Sub dural haemorrhage if temporoparietal locus involved which is most likely
then it is middle meningeal artery
Frontal locus – anterior ethmoidal artery
Occipital locus-transverse or sigmoid sinuses
Vertex locus- superior Sagittal sinus

27
CNS-System Wise 1700-by Sush and Team. 2016
Susmita, Asad, Manu, Saima, Zohaib, Savia, Shanu, Mona, Manisha, Sitara, Samreena, Sami and Komal



Reference Sampsons notes :
INTRACRANIAL BLEED:
1. Subarachnoid Haemorrhage:
• Common in young patients 30-50 years
• Cause is Berry aneurysm - located at posterior or anterior communication
artery.
• May present with collapse while exercising
• It is associated with Polycystic Kidney Disease, coarctation of aorta, Ehler-
Danlos syndrome.
Signs and symptoms
1. Sudden onset headache at the back of the head (occipital)
2. Projectile vomiting
3. Neck stiffness
4. Photophobia
5. There is usually family history
N.B: Sudden onset of headache is always subarachnoid haemorrhage until
proven otherwise. Severe
headache but no history of head injury.
Investigation:
1. CT scan head
2. If CT scan head does not show bleeding, do lumbar puncture at least 12 hours

28
CNS-System Wise 1700-by Sush and Team. 2016
Susmita, Asad, Manu, Saima, Zohaib, Savia, Shanu, Mona, Manisha, Sitara, Samreena, Sami and Komal

after onset of
headache. Usually you look for bilirubin in the CSF (xanthochromia).
Treatment:
1. Nimodipine for the pain
2. Refer to neurosurgeon
3. Treatment is usually surgery (clipping of the aneurysm)
2. Subdural Haematoma:
• Common in elderly with recurrent falls
• Also in alcoholics and boxers due to recurrent head injury
• changing level of consciousness (progressive drowsiness)
• Trauma may happen long time and patient may forget about it e.g. 2 weeks
ago
• Signs of raised ICP which are headache, papilloedema and vomiting
• Focal neurological signs i.e. weakness of the legs or sensory loss
• Can present with cognitive impairment
Treatment:
1. Refer to neurosurgeon for evacuation of haematoma
2. Surgery - Burr hole (done as an emergency to relieve increased intracranial
pressure)
3. Extradural Haematoma:
• Rapid deterioration of consciousness
• There is lucid interval i.e. in minutes to hours
• Almost/always there is history of head injury
• Signs of raised ICP (headache, vomiting and papillaoedema)
• Fits
• Focal neurological signs
• Up going plantar reflexes (Babinski sign)
• Monoparesis
Investigation: CT scan
Treatment: 1. Surgery for evacuation of haematoma
2.Burr hole may be used in emergency to relieve the pressure
4. Intracerebral bleeding (haemorrhagic stroke)
• Elderly patient with history of hypertension, usually uncontrolled or untreated
hypertension.
• Sudden loss of consciousness, preceeded by headache
• History of ischaemic heart disease or diabetes mellitus
• Normally focal neurological signs
• Up going plantar reflex
• Dilated pupils
• UMN signs in limbs
• Intracerebral bleed is same as haemorrhagic stroke
Investigation: CT scan head.
Treatment: Refer to neurosurgeon to evacuate haematoma
Dr Khalid/Rabia

29
CNS-System Wise 1700-by Sush and Team. 2016
Susmita, Asad, Manu, Saima, Zohaib, Savia, Shanu, Mona, Manisha, Sitara, Samreena, Sami and Komal

Q: 1197. A 65yo lady who is on thiazide suffers from falls in the morning. What is
the cause for her
symptoms?
a. Orthostatic hypotension
b. TIA
c. Epilepsy

Clincher(s) It's a side effect of thiazides diuretic
A a. Orthostatic hypotension
Common adverse effects
• dehydration
• postural hypotension
• hyponatraemia, hypokalaemia
• gout
• impaired glucose tolerance

B Duration will be mentioned is it less than 24 hours or more than 24 hours
C Gives history of seizures
Like drooling of saliva
Face turning to one side
Involuntary micturition
Up rolling of eyes
Clenching of fist
And features depends on type of epilepsy
Like
From sampsons notes:
EPILEPSY (Abnormal electrical discharge in the brain)
CLASSIFICATION:
1. Partial
2. Primary Generalised
PARTIAL:
this is when electrical discharge from 1 hemisphere
Focal (remains in 1 hemisphere)
b. Secondary Generalised (spreads to both hemispheres) e.g. Jacksonian attack
(jerking starts from the
thumb -> hands -> body.
PRIMARY GENERALISED:
Electrical discharge from both hemispheres
a. Infantile spasms (salaam attack) common in infants. This child jerks back and
forth like the way
muslims pray. It can also cause severe developmental delay
b. Absent seizures -> in children. A period the child stops for about 10 seconds
and then resumes what
he is doing
c. Tonic clonic
d. Atonic (floppy)

30
CNS-System Wise 1700-by Sush and Team. 2016
Susmita, Asad, Manu, Saima, Zohaib, Savia, Shanu, Mona, Manisha, Sitara, Samreena, Sami and Komal

e. Myoclonic (twitching of the muscles of the face and the whole body)
Epilepsy:
1. Simple (no loss of consciousness)
2. Complex (loss of consciousness)
Symptoms:
1. Jerking (fit)
2. Urinary or faecal incontinence
3. Tongue biting
4. Loss of consciousness
5. Post-ictal status
Investigations:
1. EEG: Investigation of choice is EEG
2. CT Scan (to rule out brain tumour)
a. If seizure of new onset at night (during sleep)
b. If seizure is associated with prolong headaches
c. New onset focal seizures in adults (epilepsy is usually develops in a child)
Treatment:
1. Primary Generalised epilepsy - Sodium valproate
2. Absence seizure - Ethosuximide
3. Partial seizure - Carbamazapine
4. Infantile Spasm - Vigabatrin
If seizures without any cause -> send to 1
General advice to give patient:
1. Do not drive and advice patient to inform DVLA
2. Avoid unsafe activities e.g. swimming alone, mountain climbing, riding a
bicycle
3. Take showers rather than bath
4. Avoid precipitants like night club, watching TV with strobe lights on
Status Epilepticus:
This is seizure lasting more than 30 minutes or repeated attacks of seizures
without gaining
consciousness in between.



Treatment of Status Epilepticus:
IV lorazepam

IV lorazepam
(repeat)

Phenytoin IV


Phenobarbiturate IV

31
CNS-System Wise 1700-by Sush and Team. 2016
Susmita, Asad, Manu, Saima, Zohaib, Savia, Shanu, Mona, Manisha, Sitara, Samreena, Sami and Komal

Anaesthetise and Intubate
If after PR Diazepam if patient is still fitting and you have gained an
intravenous line give IV Lorazepam
Drug Side Effects
1. Carbamezapine: Rash (in short term), Renal failure, diplopia, hyponatraemia,
SIADH, neutropenia
1. Sodium Valproate: weight gain, liver failure, tremor, sedation, rash, low
platelets, hair loss
1. Phenytoin: gum hypertrophy, cerebellar signs and depression.

i. Nystagmus
ii. Diplopia
iii. Ataxia
iv. Dysarthria
v. Dizziness
1. Ethosuximide: bone marrow suppression, headache, lethargy, ataxia,
agranulocytosis, GIT irritation.
1. Benzodiazepines: IM injection may cause cold abscess

D
E
KEY A
Additional BNF:
Information

Side-effects

Side-effects of thiazides and related diuretics include mild gastro-


intestinal disturbances, postural hypotension, altered plasma-lipid
concentrations, metabolic and electrolyte disturbances including
hypokalaemia (see also Potsassium loss (thiazide and loop diuretics)),
hyponatraemia, hypomagnesaemia, hypercalcaemia, hyperglycaemia,
hypochloraemic alkalosis, hyperuricaemia, and gout. Less common side-
effects include blood disorders such as agranulocytosis, leucopenia, and
thrombocytopenia, and impotence. Pancreatitis, intrahepatic cholestasis,
cardiac arrhythmias, headache, dizziness, paraesthesia, visual
disturbances, and hypersensitivity reactions (including pneumonitis,
pulmonary oedema, photosensitivity, and severe skin reactions) have
also been reported.

32
CNS-System Wise 1700-by Sush and Team. 2016
Susmita, Asad, Manu, Saima, Zohaib, Savia, Shanu, Mona, Manisha, Sitara, Samreena, Sami and Komal

Common or serious possible side-effects include:of thiazides diuretics

A possible increase in your blood sugar level. Some people with diabetes may
need more treatment to keep the blood sugar level normal.

A possible increase in the level of uric acid. So, if you have gout, you may
have more gout attacks. A first attack of gout is sometimes triggered by
taking a diuretic.

The salt balance in the bloodstream is sometimes upset which can cause a
low blood level of potassium, sodium, and magnesium, and a high level of
calcium. These effects may cause weakness, confusion, and rarely, abnormal
heart rhythms to develop. You may be advised to have a blood test to check
for these problems.

Other problems, such as:

Upset stomach.

Dizziness on standing (due to too low blood pressure).

Erection problems (impotence) - often reversible on stopping treatment.

Skin sensitivity to sunlight.


Reference
Dr Khalid/Rabia


Q:1223 1223. A 56yo pt has been dx with MS. She presents with a positive Romberg’s
test. She also has weakness and loss of sensations in all her 4 limbs. Which site
is most likely to be affected?
a. Cerebral cortex
b. Cerebellum
c. Cervical spinal cord
d. Thoracic spinal cord
e. Brain stem


Clincher(s) In positive Romberg sign
Proprioception and vestibular apparatus are involved ,
A
B From GP note book
Romberg's sign is positive if the patient requires vision to stand steadily.

33
CNS-System Wise 1700-by Sush and Team. 2016
Susmita, Asad, Manu, Saima, Zohaib, Savia, Shanu, Mona, Manisha, Sitara, Samreena, Sami and Komal

The patient is asked to stand with the feet together. If the patient is steady
with eyes open but unsteady with eyes closed then there are signs of
Rombergism.

Romberg's sign is said to be positive in patients with sensory ataxia and
negative in cerebellar ataxia. Then it is sensory ataxia. Ask to close his eyes,
loose his balance: then it’s positive and this excludes cerebellar ataxia.

In practise Romberg's sign has a low specificity.

In cerebellar disease, the patient is often unsteady with the eyes open or
closed.
C
D Only loss of sensation in thoracic
E
KEY c. Cervical spinal cord
Additional Taken from USMLE FORUM
Information •Romberg test is only positive if the patient has intact balance with eyes OPEN
but impaired balance with eyes CLOSED.
If the patient has trouble with eyes OPEN (and, by extension, he will surely
have impaired balance with eyes closed also), it is not a positive romberg test
and that is probably more indicative of cerebellar etiology. The reason
Romberg is a proprioceptive test is because it only applies when the patient
has a discrepancy regarding balance with eyes open and with eyes closed. An
easy way to justify this is you need 2 out of the 3 systems working at any given
time to maintain balance.

If your eyes are closed, those are out, and if you have impaired proprioception,
that's out, thus you are only relying on cerebellum and will have impaired
balance (positive romberg).

If your eyes are open, and you still have trouble with balance, what does that
mean? (If you can answer this, then I think you understand my horrible
explanation) so
To maintain gait, you need:
(a) positional sense (comes from visual, vestibular, and proprioceptive input)
(b) motor output (both corticospinal tract and basal ganglia)
(c) integration of senses (a) with motor output (b) (cerebellum)

RABIA notes :
Romberg’s Test is used to identify instability of either peripheral or central
cause -
The patient stands up straight with feet together (or at a distance for them to
be steady) with arms outstretched. Then ask them to shut their eyes.
• If they are unable to maintain their balance with their eyes closed, the test is
positive (usually fall to the side of the lesion so stay close by to prevent them

34
CNS-System Wise 1700-by Sush and Team. 2016
Susmita, Asad, Manu, Saima, Zohaib, Savia, Shanu, Mona, Manisha, Sitara, Samreena, Sami and Komal

falling).
• A positive test suggests a problem with proprioception or vestibular function.
Romberg's test can also be positive in neuromuscular disorders and may not
be reliable in very elderly people.
Multiple sclerosis causes demyelination of neurons in brain and spinal cord
leading to neurological symptoms and loss of sensation, difficulty in movement,
coordination and balance. In this case, all limbs are affected, so the level of
lesion will be higher, ie. cervical spine.
Positive Romberg sign is 90% sensitive for lumbar spinal stenosis
Wikipedia:
“Proprioception ( ), from Latin , meaning "one's own", "individual" and
perception, is the sense of the relative position of neighbouring parts of
the body and strength of effort being employed in movement. It is
provided by proprioceptors in skeletal striated muscles and in joints. It
is distinguished from exteroception, by which one perceives the outside
world, and interoception, by which one perceives pain, hunger, etc., and
the movement of internal organs. The brain integrates information from
proprioception and from the vestibular system (vestibule of the ear) into
its overall sense of body position, movement, and acceleration..“
Reference GP note book
Dr Khalid/Rabia


Q:1242
1242. A 32yo woman suffers an episode of severe occipital headache with
vomiting and LOC. She is brought to the hosp where she is found to be
conscious and completely alert. Exam: normal pulse and BP with no abnormal
neurological sign. What is the next step in her management?
a. Admission for observation
b. CT brain
c. MRI head
d. Reassurance and discharge home
e. XR skull



Clincher(s) Episode of severe occipital head ache and vomitting and loc
A Admission depends on CT scan findings done in A and E department
B Should be first choice to rule out any organic cause for vomitting
C MRI to be done if CT scan reports are doubtful
D Can't discharge a patient with such clinical features one should rule out the
cause
E There is no history of trauma
For the presenting complaints it has no role
KEY Ct scan (to exclude SAH as occipital headache)
Additional 1. Subarachnoid Haemorrhage:
Information • Common in young patients 30-50 years

35
CNS-System Wise 1700-by Sush and Team. 2016
Susmita, Asad, Manu, Saima, Zohaib, Savia, Shanu, Mona, Manisha, Sitara, Samreena, Sami and Komal

• Cause is Berry aneurysm - located at posterior or anterior
communication artery.
• May present with collapse while exercising
• It is associated with Polycystic Kidney Disease, coarctation of aorta,
Ehler-Danlos syndrome.
Signs and symptoms
1. Sudden onset headache at the back of the head (occipital)
2. Projectile vomiting
3. Neck stiffness
4. Photophobia
5. There is usually family history
N.B: Sudden onset of headache is always subarachnoid haemorrhage
until proven otherwise. Severe
headache but no history of head injury.
Investigation:
1. CT scan head
2. If CT scan head does not show bleeding, do lumbar puncture at least
12 hours after onset of
headache. Usually you look for bilirubin in the CSF (xanthochromia).
Treatment:
1. Nimodipine for the pain
2. Refer to neurosurgeon
3. Treatment is usually surgery (clipping of the aneurysm)
Chronic headaches in the back of the head--also known as occipital headaches-
-are often variants of tension headaches or migraines. New-onset occipital
headaches are a more urgent concern. Possible causes include blood vessel
abnormalities, nerve disorders and serious traumatic injuries.
It can be either of the below two conditions
Subarachnoid Hemorrhage:
Three protective tissue layers called the meninges protect the brain. The pia
mater covers the brain surface. The arachnoid mater is the middle layer of the
meninges. The dura mater is the outermost meningeal layer. A subarachnoid
hemorrhage is bleeding that occurs between the pia mater and the arachnoid
mater. A severe headache in the back of the head is the hallmark symptom of
a subarachnoid hemorrhage. Other symptoms associated with subarachnoid
hemorrhage include nausea, vomiting, confusion, visual disturbances and
decreased level of consciousness. A subarachnoid hemorrhage is a medical
emergency.

Occipital Neuralgia
Occipital neuralgia is an irritation of the occipital nerves, which causes severe
pain in the back of the head. The occipital nerves emerge from the spinal cord
at the top of the back of the neck, and fan out over the scalp. The headache of
occipital neuralgia follows the path of the nerves, and is usually limited to one
side of the head. Possible causes of occipital neuralgia include tumors in the
neck, vasculitis, arthritis, injury and having the head tilted forward for lengthy
periods. Notably, there are no brain-related symptoms associated with

36
CNS-System Wise 1700-by Sush and Team. 2016
Susmita, Asad, Manu, Saima, Zohaib, Savia, Shanu, Mona, Manisha, Sitara, Samreena, Sami and Komal

occipital neuralgia. Anti-inflammatory medications, resting the neck and
muscle relaxant medications are the primary treatments for occipital
neuralgia.
Severe headache with LOC (B)
- The next step in this case should be CT-Brain to look for or rule out any
organic cause.
- Reassurance and discharging the patient is a risky option.
- X-ray Skull does not have any utility in this condition.
- MRI and Admission should follow CT-Brain if required
Reference NHS
Dr Khalid/Rabia


Q: 1304. A 67yo man presents with a hx of increasing confusion and drowsiness.
He had a fall 2wk ago. CT head reveals a chronic subdural hematoma. What is
the best management for this pt?
a. Craniotomy
b. Burr hole drainage
c. Conservative management
d. Excision and biopsy
E. Burr hole craniotomy.

Clincher(s) I feel it's incomplete question
There should be GCS scale or comment on pupils
Ct report saying any shift midline shift of haematoma or size
A SurgEry is done if it meets the criteria of acute sub dural haematoma if there is
significant cerebral swelling or contusion ass Surgery is typically indicated for:
Subdural haematoma of >10 mm or a midline shift >5 mm with any GCS
GCS <9 that has dropped ≥2 points between injury and emergency
department, with a subdural haematoma of <10 mm and midline shift <5 mm

GCS <9, with a subdural haematoma of <10 mm and midline shift <5 mm, and
fixed or asymmetrical pupils

GCS <9, with a subdural haematoma of <10 mm and midline shift <5 mm, and
ICP >20 mmHg.
B Burr hole drainage shouldn't be done
If drain is put chances of recurrence will be more
C Is done if it meets the criteria of acute sub dural haematoma
Conservative management is generally deemed appropriate for acute subdural
haematomas that are small, do not cause significant midline shift or cisternal
encroachment, and do not cause significant neurological impairment (beyond
headaches).
Typically, this includes patients with:
GCS 9 to 15 with a subdural haematoma <10 mm and midline shift <5 mm
GCS <9 with a haematoma <10 mm and midline shift of <5 mm who have been

37
CNS-System Wise 1700-by Sush and Team. 2016
Susmita, Asad, Manu, Saima, Zohaib, Savia, Shanu, Mona, Manisha, Sitara, Samreena, Sami and Komal

stable between injury and emergency department and with reactive,
symmetrical pupils and ICP <20 mmHg.


With subdural; first craniostomy and then craniotomy

D Not advice at all
E This is the recommended treatment for chronic
KEY E
Additional 2. Subdural Haematoma:
Information • Common in elderly with recurrent falls
• Also in alcoholics and boxers due to recurrent head injury
• changing level of consciousness (progressive drowsiness)
• Trauma may happen long time and patient may forget about it e.g. 2
weeks ago
• Signs of raised ICP which are headache, papilloedema and vomiting
• Focal neurological signs i.e. weakness of the legs or sensory loss
• Can present with cognitive impairment
Investigation: CT scan

Treatment:
1. Refer to neurosurgeon for evacuation of haematoma
2. Surgery - Burr hole (done as an emergency to relieve increased
intracranial pressure)
Surgical management may be frontotemporoparietal craniotomy, burr hole
craniotomy with irrigation, or twist-drill craniotomy with drain placement.
[53] [54] Recurrent subdural haematomas that have a fluid consistency may
be treated with a subdural-peritoneal shunt
Reference Nhs and nice n bmj
Dr Khalid/Rabia

Q:1309 1309. A 49yo man first presented with increasing difficulty in swallowing.
Several months later he developed weakness in his right foot. Now he can no
longer feed himself, he chokes on food and has become confined to a
wheelchair. What is the most likely dx?
a. Cerebral tumor
b. Myasthenia gravis (eye involvement)
c. Lambert-Eaton syndrome
d. Motor neuron disease
e. Cerebro-vascular disease


Clincher(s) Involvement of bulbar muscles then muscles of hands and feet
A Symptoms of a brain tumour
The symptoms of a brain tumour vary depending on the exact part of the brain
that's affected. Common symptoms include:

38
CNS-System Wise 1700-by Sush and Team. 2016
Susmita, Asad, Manu, Saima, Zohaib, Savia, Shanu, Mona, Manisha, Sitara, Samreena, Sami and Komal

severe, persistent headaches
seizures (fits)
persistent nausea, vomiting and drowsiness
mental or behavioural changes, such as memory problems or changes in
personality
progressive weakness or paralysis on one side of the body, vision problems, or
speech problems
B MYASTHENIA GRAVIS
It is due to reduction in the number of nicotine AChR at neuromuscular
junction.
This is due to acetylcholine receptor antibodies formed against the
acetylcholine receptor.
There is abnormality of the thymus gland in 75% of the cases either in a form
of hyperplasia or thymoma.
It is an autoimmune condition.
It is associated with other autoimmune conditions like SLE, Pernicious
Anaemia, Grave’s disease,
Rheumatoid Arthritis
Symptoms:
1. Painless muscle weakness which increases with exercise
2. Generalised weakness
3. Dysphagia, dysphonia, dysarthria, limb weakness
4. There is fatiguability and weakness which worsens by the end of the day or
whenever patient works
hard.
5. Eye muscle weakness may present with diplopia
Investigation:
1. Serum AChR antibodies are diagnostic of myasthenia gravis
2. Tensilon (edrophonium) test diagnostic
3. CT mediastinum to look for thymus gland
Treatment:
1. Cholinesterase inhibitor e.g. Pyridostigmine
2. Steroid can be used if symptoms not adequately controlled by
cholinesterase inhibitors
3. Azathioprine if steroid is contraindicated
4. Other immune suppressant e.g. ciclosporin, methotrexate may also be used
5. Plasma exchange and immunoglobulin are used in patients with myasthenia
crisis.
6. Thymomectomy may be used in patients with AChR antibodies and under 45
years of age.

C Lambert-Eaton myasthenic syndrome (LEMS) is a rare disorder caused by a
problem with the transmission of nerve signals to the muscles.
results in muscle weakness and sometimes dryness of the mouth, constipation
and impotence.due to release of antibodies at the nerve endings
The immune system mistakenly attacks the nerve endings, which weakens the

39
CNS-System Wise 1700-by Sush and Team. 2016
Susmita, Asad, Manu, Saima, Zohaib, Savia, Shanu, Mona, Manisha, Sitara, Samreena, Sami and Komal

nerve impulses from the nerves to the muscles and prevents the muscles
contracting (tightening) properly.
In about 50% of people with LEMS, the disease is triggered by an underlying
lung cancer (the type associated with smoking). These people usually develop
the disease in middle age or later.
The autonomic nervous system is the nervous system in charge of automatic
body functions we don't need to think about, such as breathing, sexual arousal
and swallowing.
LEMS cannot be cured, but the symptoms can be managed through a variety of
approaches:
by stimulating greater release of acetylcholine – through the use of 3,4-
diaminopyridine
by prolonging the activity of the acetylcholine that is released – with a drug
called pyridostigmine
by reducing the levels of the antibodies causing the condition – with steroids
or immunosuppressant drugs : OHCM- 516

key: autonomic involvement, impoves after exercise. here

D In MND, motor nerves become damaged and eventually stop working.
Therefore, the muscles that the damaged nerves supply gradually lose their
strength. There are various subtypes of MND. In each type, symptoms tend to
start in different ways. However, as the disease progresses, the symptoms of
each type of MND tend to overlap. This means that symptoms in the later
stages of each type of MND become similar. The main types of MND are:
Amyotrophic lateral sclerosis (ALS). This is the classical MND and the most
common type. About 8 in 10 people with MND have this type. Symptoms tend
to start in the hands and feet. The muscles tend to become stiff as well as
weak at first.
Progressive bulbar palsy (PBP). About 2 in 10 people with MND have this type.
The muscles first affected are those used for talking, chewing and swallowing
(the bulbar muscles).
Progressive muscular atrophy (PMA). This is an uncommon form of MND. The
small muscles of the hands and feet are usually first affected but the muscles
are not stiff.
Primary lateral sclerosis (PLS). This is a rare type of MND. It mainly causes
weakness in the leg muscles. Some people with this type may also develop
clumsiness in the hands or develop speech problems.
E In CVA symptoms of stroke >24hours
Symptoms of TIA <24hrs will be there
Symptoms:
1. Carotid atherosclerosis or carotid artery disease:
a. Frontal lobe - hemiparesis, monoparesis, dysphasia if dominant
b. Temporal lobe - memory loss
c. Parietal lobe - sensory loss
d. Eye - amarousis fugax

40
CNS-System Wise 1700-by Sush and Team. 2016
Susmita, Asad, Manu, Saima, Zohaib, Savia, Shanu, Mona, Manisha, Sitara, Samreena, Sami and Komal

e. If dominant hemisphere aphasia/dysphasia
2. Vertebrobasilar artherosclerosis or vertebrobasilar insufficiency:

a. Dysarthria
b. Vertigo
c. Nystagmus
d. Cerebellar ataxia
e. Hypotonia
3. Brainstem: Cranial nerves are involved and causes difficulty swallowing
( UMN sign lesions in limbs )
KEY D
Additional
Information
Reference Sampsons and nhs.co.uk
Dr Khalid/Rabia

Q:1312 1312. A pt with vesicles in the maxillary division of trigeminal nerve. Which
area of mucus membrane will be involved?
a. Palate
b. Cheek
c. Cornea
d.conjunctiva

Clincher(s) Area of mucous membrane
A Maxillary with mucous membrane
B Maxillary nerve only externally
C Ophthalmic nerve
D Ophthalmic nerve
E
KEY A
Additional
Information

41
CNS-System Wise 1700-by Sush and Team. 2016
Susmita, Asad, Manu, Saima, Zohaib, Savia, Shanu, Mona, Manisha, Sitara, Samreena, Sami and Komal


The ophthalmic nerve (V1) carries sensory information from the scalp and
forehead, the upper eyelid, the conjunctiva and cornea of the eye, the nose
(including the tip of the nose, except alae nasi), the nasal mucosa, the frontal
sinuses and parts of the meninges (the dura and blood vessels). The maxillary
nerve (V2) carries sensory information from the lower eyelid and cheek, the
nares and upper lip, the upper teeth and gums, the nasal mucosa, the palate
and roof of the pharynx, the maxillary, ethmoid and sphenoid sinuses and
parts of the meninges. The mandibular nerve (V3) carries sensory information
from the lower lip, the lower teeth and gums, the chin and jaw (except the
angle of the jaw, which is supplied by C2-C3), parts of the external ear and
parts of the meninges. The mandibular nerve carries touch-position and pain-
temperature sensations from the mouth. Although it does not carry taste
sensation (the chorda tympani is responsible for taste), one of its branches—
the lingual nerve—carries sensation from the tongue.
for mucous membrane its palate for Maxillary... if they had not mentioned
mucous then mid face externally is supplied by Maxillary. Mandibular is
division that gives buccal branch and supplies the buccal mucosa with sensory
fibers...
Ans should be palate as question is being asked about the mucous membrane
Reference
Dr Khalid/Rabia Rabia


Q:1319 1319. A 55yo woman with breast ca which has spread to lung, liver and bone
now presents with increasing constipation, weakness, thirst and anorexia for
the past 3d. Her only medication is haloperidol for hiccoughs. Today she is
disorientated and has left sided weakness. What is the most likely dx?
a. Brain mets
b. Hypercalcemia
c. Liver failure


Clincher(s) Primary ca in breast causing secondaries mets
And having features of hypercalcaemia

42
CNS-System Wise 1700-by Sush and Team. 2016
Susmita, Asad, Manu, Saima, Zohaib, Savia, Shanu, Mona, Manisha, Sitara, Samreena, Sami and Komal

A It will have focal motor weakness
Other features are as follows :
A persistent headache with or without vomiting.
Seizures (Change in type or frequency )
Double vision, mental changes, speech difficulty.
Stroke (brain hemorrhage). Certain brain metastases, such as melanoma, renal
carcinoma, and choriocarcinoma bleed easily.
Confusion
Any numbness, arm or leg weakness, back pain, difficulty walking.
Loss of control of bowel or bladder.
B Pt has all the features of Hyepercalcemia except the left sided weakness. But,
Brain mets can only cause localized weakness .
C Signs of jaundice will be there in presentation
Symptoms of cirrhosis can include:
tiredness and weakness
loss of appetite
weight loss and muscle wasting
feeling sick (nausea) and vomiting
tenderness or pain around the liver area
tiny red lines (blood capillaries) on the skin above waist level
very itchy skin
yellowing of the skin and the whites of the eyes (jaundice)
a tendency to bleed and bruise more easily, such as frequent nosebleeds or
bleeding gums
hair loss
fever and shivering attacks
swelling in the legs, ankles and feet due to a build-up of fluid (oedema)
swelling in your abdomen (tummy), due to a build-up of fluid known as ascites
(severe cases can make you look heavily pregnant)
D
E
KEY A
Additional Hypercalcaemia can be defined as a serum calcium concentration of 2.65
Information mmol/L or higher, on two occasions, following adjustment (correction) for
the serum albumin concentration [Davies et al, 2002; Smellie et al, 2008].
Reference ranges may vary according to local laboratories.

Hypercalcaemia can be classified as mild, moderate, or severe based on the


adjusted serum calcium concentration. However, the severity of symptoms
also depends on the rate of onset of hypercalcaemia [Smellie et al, 2008].

Mild hypercalcaemia is an adjusted serum calcium concentration of 2.65–


3.00 mmol/L.

Moderate hypercalcaemia is an adjusted serum calcium concentration of

43
CNS-System Wise 1700-by Sush and Team. 2016
Susmita, Asad, Manu, Saima, Zohaib, Savia, Shanu, Mona, Manisha, Sitara, Samreena, Sami and Komal

3.01–3.40 mmol/L.

Severe hypercalcaemia is an adjusted serum calcium concentration of greater


than 3.40 mmol/L.

Common causes (accounting for 90% of people with hypercalcaemia):

Primary hyperparathyroidism .

Malignancy such as breast or lung cancer or multiple myeloma.

Less common causes:

Drugs including:

Thiazide diuretics.

Lithium.

Vitamin D.

Vitamin A.

Calcium co-prescribed with antacids or calcium and vitamin D preparations


('calcium-alkali syndrome').

Granulomatous diseases : such as sarcoidosis or tuberculosis.

Chronic kidney disease stage 4 or 5/transplantation .

Familial hypocalciuric hypercalcaemia .

Non-parathyroid endocrine diseases including:

Thyrotoxicosis.

Addison's disease.

Phaeochromocytoma.

Vasoactive intestinal polypeptide hormone-producing tumour (VIPoma; a


type of islet cell tumour).

Immobility for example in people with Paget's disease.

Clinical features of hypercalcaemia include:

44
CNS-System Wise 1700-by Sush and Team. 2016
Susmita, Asad, Manu, Saima, Zohaib, Savia, Shanu, Mona, Manisha, Sitara, Samreena, Sami and Komal

Skeletal

Bone pain.

Fractures associated with underlying bone disorders (fragility fractures in


hyperparathyroidism or pathological fractures in malignancy).

Neuromuscular and neuropsychiatric

Drowsiness, delirium, coma.

Fatigue, muscle weakness.

Impaired concentration and memory.

Depression.

Neurological signs (for example upper motor neurone deficits and ataxia).

Gastrointestinal

Nausea, vomiting, anorexia, weight loss.

Constipation, abdominal pain.

Peptic ulcer, pancreatitis (both rare).

Renal

Renal colic due to renal stones.

Polyuria, polydipsia, and dehydration (due to nephrogenic diabetes


insipidus).

Renal impairment (due to nephrocalcinosis).

Cardiovascular

Hypertension.

Shortened QT interval on electrocardiogram (ECG).

Cardiac arrhythmias (rare).

Other

Itching, keratitis, conjunctivitis, and corneal calcification

45
CNS-System Wise 1700-by Sush and Team. 2016
Susmita, Asad, Manu, Saima, Zohaib, Savia, Shanu, Mona, Manisha, Sitara, Samreena, Sami and Komal

Management depends on the presentation of the scenario
Reference Nice guidelines
Dr Khalid/Rabia


Q:1329 1329. A 24yo woman presents with tingling and twitching of her fingers
followed by throbbing
unilateral headache. What is the most likely dx?
a. Tension headache
b. Migraine
c. Cluster headache
d. TIA
e. SAH

Clincher(s) Young female
Sensory and visual aura
Unilateral head ache
A TENSION HEADACHE:
• Bilateral band-like headache, usually comes when patient is stressed
Treatment: Paracetamol/ aspirin/ ibuprofen or stronger painkillers

B Migraine is a common type of primary headache. It is characterised typically
by:

• a severe, unilateral, throbbing headache
• associated with nausea, photophobia and phonophobia
• attacks may last up to 72 hours
• patients characteristically go to a darkened, quiet room during an attack
• 'classic' migraine attacks are precipitated by an aura. These occur in around
one-third of migraine patients
• typical aura are visual, progressive, last 5-60 minutes and are characterised
by transient hemianopic disturbance or a spreading scintillating scotoma
• formal diagnostic criteria are produced by the International Headache
Society (see below)

Epidemiology

• 3 times more common in women
• prevalence in men is around 6%, in women 18%

Common triggers for a migraine attack

• tiredness, stress
• alcohol
• combined oral contraceptive pill
• lack of food or dehydration

46
CNS-System Wise 1700-by Sush and Team. 2016
Susmita, Asad, Manu, Saima, Zohaib, Savia, Shanu, Mona, Manisha, Sitara, Samreena, Sami and Komal

cheese, chocolate, red wines, citrus fruits
• menstruation
• bright lights
Sampsons notes: MIGRAINE
• Young, female patient typically
• Recurrent headaches: throbbing, pulsatile, unilateral
• Associated with nausea and vomiting
• Aura – can be sensory or visual
a. Sensory aura usually tingling or numbness in upper limb
b. Visual aura usually patient sees zig-zag lines but it can be any other form of
visual loss)
• Aura usually lasts seconds to minutes and within 1 hour of aura headache
follows
• there is usually family history
Investigations: routine investigations, if diagnosis is not clear, to rule out other
causes
Treatment: ACUTE = high dose aspirin 900mg or ketoprofin
Prophylaxis:
1. Beta-Blocker (Propranolol)
2. Topiramate
3. Amitryptalline

C CLUSTER HEADACHE:
• Common in young middle aged men
• Usually unilateral severe headache, which radiates to the forehead
• Associated with redness of the eye and lacrimation.
• Severe headache which makes patient cry
• Headache occurs in clusters (comes and goes in periods e.g. 2 months of
headaches at the same
time and then 8 months of free headache and followed up by another period
of headaches) this is
why it is called cluster headache
• Treatment is with high 100% oxygen and sumatriptan.

D Cardiovascular Accident
1. Stroke: symptoms >24 hours
2. TIA: symptoms <24 hours
STROKE:
1. Ischaemic
2. Haemorrhagic -> intracerebral haemorrhage (associated with increased
blood pressure, uncontrolled
or untreated hypertension)
Symptoms:
1. Carotid atherosclerosis or carotid artery disease:
a. Frontal lobe - hemiparesis, monoparesis, dysphasia if dominant
b. Temporal lobe - memory loss

47
CNS-System Wise 1700-by Sush and Team. 2016
Susmita, Asad, Manu, Saima, Zohaib, Savia, Shanu, Mona, Manisha, Sitara, Samreena, Sami and Komal

c. Parietal lobe - sensory loss
d. Eye - amarousis fugax
e. If dominant hemisphere aphasia/dysphasia
2. Vertebrobasilar artherosclerosis or vertebrobasilar insufficiency:
a. Dysarthria
b. Vertigo
c. Nystagmus
d. Cerebellar ataxia
e. Hypotonia
3. Brainstem: Cranial nerves are involved and causes difficulty swallowing
( UMN sign lesions in limbs )
A. Investigations to find the cause:
• If Atrial Fibrillation then Echocardiogram to see intra-cardiac emboli
• If recurrent Myocardial Infarction then Echocardiogram to see mural
thrombi
• If Carotid bruit then Doppler ultrasound of the neck
• If history of ischaemic heart disease, it means atherosclerosis is present
then Doppler ultrasound
(carotid artery and vertebrobasilar artery) needs to be done
• If diabetic patient then Blood glucose
• Blood pressure measurement to rule out Hypertension
• Heart murmur it means there is vulvular heart disease which can cause
emboli Echocardiogram to
see an emboli
• Thoracic Aortic Aneurysm then CT scan chest
B. Investigation to make diagnosis:
CT Scan of head
1. Ischaemic
2. Haemorrhagic
General Management of cerebral vascular accident
• If patient has symptoms of stroke do CT scan head to exclude haemorrhagic
stroke.
• If no haemorrhagic stroke then give Aspirin 300mg stat
• If patient presents within 3 hours and no haemorrhagic stroke, arrange
thrombolysis.
• If patient had symptoms of stroke, which have now resolved it means he
had TIA - give aspirin
300mg orally, even before doing a CT scan because symptoms have resolved
which suggests it is
TIA.
Prophylaxis: clopidogrel
If clopidogrel allergy substitute with Aspirin + dipyridamole


E SPACE OCCUPYING LESION:
Causes:

48
CNS-System Wise 1700-by Sush and Team. 2016
Susmita, Asad, Manu, Saima, Zohaib, Savia, Shanu, Mona, Manisha, Sitara, Samreena, Sami and Komal

1. Tumour
2. Abscess
3. Haematoma
4. Brain metastasis
5. Aneurysm
6. Granuloma
Symptoms:
• Usually in elderly patients
• Weight loss, anorexia
• Anaemia, tiredness
• Increased ICP: headache, vomiting and papilloedema
• Focal neurological signs (weakness in the limbs)
• Seizures
Investigations: CT scan head
Brain tumour:
Usually in elderly patients
• Symptoms are usually gradual onset
• Progressive worsening headache usually bilateral
• Adult onset seizure is always brain tumour until proven otherwise
• Headaches worse in the morning and on bending forward due to raised
intracranial pressure
Treatment: 1.Dexamethasone if headaches or raised ICP to reduce oedema
around tumour
2. surgery if localised tumour
3. Radiotherapy if metastasis but appropriate in that patient


KEY B
Additional Migraine diagnostic criteria
Information
A.At least 5 attacks fulfilling criteria B-D

B.Headache attacks lasting 4-72 hours* (untreated or unsuccessfully treated)

C .Headache has at least two of the following characteristics:

• 1. unilateral location*

• 2. pulsating quality (i.e., varying with the heartbeat)

• 3. moderate or severe pain intensity

• 4. aggravation by or causing avoidance of routine physical activity (e.g.,


walking or climbing stairs)

D.During headache at least one of the following:

49
CNS-System Wise 1700-by Sush and Team. 2016
Susmita, Asad, Manu, Saima, Zohaib, Savia, Shanu, Mona, Manisha, Sitara, Samreena, Sami and Komal

• 1. nausea and/or vomiting*

• 2. photophobia and phonophobia

E.Not attributed to another disorder (history and examination do not suggest


a secondary headache disorder or, if they do, it is ruled out by appropriate
investigations or headache attacks do not occur for the first time in close
temporal relation to the other disorder)

OTHER HEADACHES :

GIANT CELL ARTERITIS:

• Typical age above 50 or elderly patients

• Common in women

• Unilateral headache on the temple areas

• Headache worse with combing hair, chewing (jaw claudication)

• Usually associated with weight loss, anorexia and weakness of the upper
limbs

• In 25% it is associated with polymyalgia rheumatic which is an autoimmune


condition mainly

affecting the muscles especially those of the upper limb which make patient
difficulty to stand up

from the chair.

Investigation:

1. Initial is ESR

2. Definitive is temporal artery biopsy

Management:

If symptoms of GCA and ESR is raised then next step is treatment with IV
methylprednisolone for 3

days followed by high dose oral prednisolone for 2 - 3 years i.e. long term.

Definitive diagnosis of GCA is made by temporal artery biopsy within 3 days

50
CNS-System Wise 1700-by Sush and Team. 2016
Susmita, Asad, Manu, Saima, Zohaib, Savia, Shanu, Mona, Manisha, Sitara, Samreena, Sami and Komal

of provisional diagnosis.

1. TRIGEMINAL NEURALGIA:

• Usually electric shock like or knife like or stabbing pain in the face

• Usually in the distribution of the trigeminal nerve branch e.g. mandibular,


maxillary or ophthalmic

• Pain can be triggered by shaving or chewing

• Facial pain runs up and down the face

Treatment: Anti-epileptic medication e.g. Carbamazepine, gabapentin

ACUTE CLOSED ANGLE GLAUCOMA:

• Typically pain is in the eye with redness and lacrimation

• Usually associated with nausea, vomiting and loss of vision

• Unilateral headache

• Severe ocular pain

• Previous intermittent headache

• Usually there is family history

• Common in females

• Haloes around the light

• On examination there is corneal oedema (fine ground glass) and fixed


dilated pupil oval shaped

• Raised intraocular pressure usually more than 40mmHg (normal IOP


<22mmHg)

1. SINUSITIS:

• Usually pain between the eyes or forehead on the sinuses

• Usually there is coryza symptoms: sneezing, running nose and cough

• Facial pain which is worse on bending forward

51
CNS-System Wise 1700-by Sush and Team. 2016
Susmita, Asad, Manu, Saima, Zohaib, Savia, Shanu, Mona, Manisha, Sitara, Samreena, Sami and Komal

Investigation: CT Scan

Treatment: Antibiotics +/- washout of the sinuses

1. BENIGN RAISED INTRACRANIAL PRESSURE (pseudotumour cerebri)

• This is common in young obese women

• Morning headaches

• Papilloedema

• 6

th

nerve palsy

• Plus or minus diplopia

• Signs of raised intracranial pressure i.e. headache, vomiting, papilloedema

Investigation: CT head to rule out brain tumour

Treatment: weight loss, acetazolamide

1. BRAIN TUMOUR:

• Usually progressive headache

• Usually elderly patient

• Signs of raised intracranial pressure (papilloedema, vomiting, headache)

• Plus or minus focal neurological symptoms (e.g. weakness or sensory loss in


the limbs)

Investigation: CT brain

Treatment:

1. Dexamethasone to reduce raised ICP or as treatment of headaches

2. Surgery if no metastasis

3. Radiotherapy if metastasis

52
CNS-System Wise 1700-by Sush and Team. 2016
Susmita, Asad, Manu, Saima, Zohaib, Savia, Shanu, Mona, Manisha, Sitara, Samreena, Sami and Komal


Reference Sampsons
Dr Khalid/Rabia


Q:1341 1341. A 24yo man after a head injury presents with difficulty dressing himself,
difficulty in writing and inability to differentiate the fingers of his hand. Which
part of the brain is most likely to be affected?
a. Frontal lobe
b. Parietal lobe
c. Temporal lobe
d. Occipital lobe
e. Brainstem


Clincher(s)
Personality changes as mentioned in question

A . Frontal lobe:
• Personality change
• Intellectual impairment
• Broca’s aphasia (if dominant hemisphere)
• Mono or hemiparesis
• Urinary Incontinence

B Parietal lobe:
• Sensory loss
• Astereognosis (failure to recognise objects by touch)
• Loss of 2 point discrimination
• Wernicke’s aphasia (if dominant hemisphere)
• Lower quadrant anopia

C Temporal lobe:
• Memory loss (amnesia)
• Déjà vu (a feeling everything is familiar)
• Je’mais vu (failure to recognise situations which have been encountered
before)
• Wernicke’s aphasia if dominant hemisphere
• Upper quadrant anopia
• Agnosia (loss of perception
D Occipital lobe:
• Homonymous hemianopia
• Cortical blindness due to bilateral occipital lobe infarction (patient is blind
but does not understand

53
CNS-System Wise 1700-by Sush and Team. 2016
Susmita, Asad, Manu, Saima, Zohaib, Savia, Shanu, Mona, Manisha, Sitara, Samreena, Sami and Komal

that he is blind. He has no insight to his blindness)

E
KEY B (Gerstman man syndrome OHCM- 714)
Additional Cerebellum (is mainly responsible for coordination):
Information ◦ Diplopia
◦ Dizziness
◦ Nystagmus
◦ Dysarthria (slurred speech)
◦ Hypotonia
◦ Past pointing
◦ Dysdiadokinesis
◦ Heel-shin test positive
◦ Ataxia (cerebellar ataxia)
6. Cerebellopontine angle
• There will be cerebellar signs
• Cranial nerves V, VII and VIII are affected together, because they pass
through the cerebellopontine
angle e.g. acoustic neuroma (schwanoma) tumour of the VIII nerve)

The parietal lobe is one of the four major divisions of the cerebral cortex. This
lobe receives and processes sensory information from the body.
Functions of major lobes are :
1. Frontal lobe—conscious thought; damage can result in mood changes,
social differences, etc. The frontal lobes are the most uniquely human of all
the brain structures.
2. Parietal lobe—plays important roles in integrating sensory information
from various senses, and in the manipulation of objects; portions of the
parietal lobe are involved with visuospatial processing
3. Occipital lobe—sense of sight; lesions can produce hallucinations
4. Temporal lobe—senses of smell and sound, as well as processing of
complex stimuli like faces and scenes.
5. Limbic lobe—emotion, memory
6. Insular cortex—pain, some other senses
Reference Samsons , Page: 503 OHCM
Dr Khalid/Rabia



Q:438 438. A 44yo man comes with hx of early morning headaches and vomiting. CT
brain shows ring enhancing lesions. What is the single most appropriate
option?
a. CMV
b. Streptococcus
c. Toxoplasmosis
d. NHL Non Hodgkins lymphoma

54
CNS-System Wise 1700-by Sush and Team. 2016
Susmita, Asad, Manu, Saima, Zohaib, Savia, Shanu, Mona, Manisha, Sitara, Samreena, Sami and Komal

e. Pneumocystis jerovii


Clincher(s) RING ENHANCING LESION (any focal lesion will give a deposit/ring enhancing
lesion)
A CMV (or others CDE options as not causing encephalistis in immune
compromised patients)
B STREPTOCOCCUS (can affect immunocompromised patients)- causing
encephalitis /cerebral abscess
C TOXOPLASMOSIS
D NHL
E PNEUMOCYSTIS JEROVII
KEY C
Additional DR MAGIC L- RING ENHANCING CEREBRAL LESION
Information
In some institutions, "L" is added to the mnemonic for lymphoma.

• D: demyelinating disease
• R: radiation necrosis or resolving haematoma

• M: metastasis
• A: abscess
• G: glioblastoma
• I: infarct (subacute phase)
• C: contusion

• L: lymphoma


Reference
Dr Khalid/Rabia Causes of ring enhancing lesions on CT brain:
-Brain abscess
-Primary or secondary tumour
-CNS lymphoma
-CNS toxoplasmosis
-Nocardia infection.
>Out of the options, toxoplasmosis is the right answer, and it is commonly
found in HIV patients. TREATMENT with pyrimethamine/sulfadiazine and
folinic acid. OR clindamycin if intolerant FOR 4-6 WEEKS.
If immunocompromised, PROPHYLAXIS with Trimethoprim+sulfamethoxazole.



Q:449 A young pt is complaining of vertigo whenever she moves sideways on the bed
while lying supine. What would be the most appropriate next step?
a. Head roll test

55
CNS-System Wise 1700-by Sush and Team. 2016
Susmita, Asad, Manu, Saima, Zohaib, Savia, Shanu, Mona, Manisha, Sitara, Samreena, Sami and Komal

b. Reassure
c. Advice on posture
d. Carotid Doppler
e. CT


Clincher(s)
A Inv: Head roll test is only test for clincher
B Reassure
C Advice on Posture
D Carotid Doppler
E CT
KEY A
Additional Benign paroxysmal positional vertigo (BPPV)
Information
a) This is a clinical presentation where patients experience frequent but brief
episodes of vertigo, usually of less than three minutes duration. Each episode
is triggered by a specific movement.

b) BPPV is the most common cause of vertigo on neck extension, such as


looking upwards, and has often been misdiagnosed in the past as
vertebrobasilar insufficiency. However, in BPPV there are no other symptoms
of interruption to the posterior circulation blood supply.

c) Vertigo also occurs on first lying down in bed at night, first getting out of
bed in the morning, bending forward and turning in bed, usually to one side
only.

d) BPPV is caused by an accumulation of otoliths within a semicircular canal,


and the pathophysiological mechanism of generation of symptoms is
considered to be canalolithiasis.15,16 This otoconial debris is free to move
within the lumen of the canal, an effect that occurs during the postural
movements described above. The movement of otoliths causes fluid
movement and is perceived as vertigo.

e) Fifty percent of cases are of unknown aetiology but known causes include
infection, vascular problems, and trauma.
Reference
Dr Khalid/Rabia Dx? Benign Paroxysmal Positional Vertigo (BPPV)- Most common cause of
vertigo. Vertigo triggered by change in head position. Might be accompanied
by nausea and nystagmus. Less commonly, vomiting and syncope.
Diagnosis: Dix-Hallpike and Head roll test.
Management: Epley and Semont Maneuver


Q: 452 A 50yo woman presents following a fall. She reports pain and weakness in her

56
CNS-System Wise 1700-by Sush and Team. 2016
Susmita, Asad, Manu, Saima, Zohaib, Savia, Shanu, Mona, Manisha, Sitara, Samreena, Sami and Komal

hands for several months, stiff legs, swallowing difficulties, and has bilateral
wasting of the small muscles of her hands. Reflexes in the upper limbs are
absent. Tongue fasciculations are present and both legs show increased tone,
pyramidal weakness and hyper-reflexia with extensor plantars. Pain and
temp sensation are impaired in the upper limbs. What is the most likely dx?
a. MS
b. MND
c. Syringobulbia
d. Syringomyelia
e. Myasthenia gravis

Clincher(s)
A MS- Usually monosymptomatic: unilateral optic neuritis (pain on eye
movement and rapid central vision); numbness or tingling in the limbs; leg
weakness;brainstem or cerebellar symptoms (eg diplopia, ataxia). Symptoms
may worsen with heat (eg hot bath) or exercise. Rarely polysymptomatic.

Presents with one thing at a time: remission and replase

B MND- Think of MND in those >40yrs with stumbling spastic gait, foot-drop }
proximal myopathy, weak grip (door-handles don’t turn) and shoulder
abduction (hair-washing is hard), or aspiration pneumonia. Look for UMN
signs: spasticity, brisk refl exes, plantars; and LMN signs: wasting, fascic
ulation of tongue, abdomen, back, thigh. Is speech or swallowing aff ected
(bulbar signs)? Fasciculation is not enough to diagnose an LMN lesion: look for
weakness too. Frontotemporal dementia occurs in ~25%.

Only about motor (no sensory)

C SYRINGOBULBIA- ; Syringo bulbia (brain stem involvement):nyst agmus, tongue
atrophy,

syringe/ empty spaces in spinal tube

Will have dysphagia, due to vagus nerve/ brain stems

D SYRINGOMYELIA- Cardinal signs: Dissociated sensory loss (absent pain and T°
sensation, with preserved
light touch, vibration, and joint-position sense) due to pressure from the syrinx
on the decussating antero lateral pathway (fi g 1) in a root dis tribution
reflecting the location of the syrinx (eg for typical cervical syrinx then sensory
loss is over trunk and arms); wasting/weakness of hands } claw-hand (then
arms sho ulders respirat ory muscles). Anterior horn cells are also
vulnerable.Other signs: Horner’s syndrome; UMN leg signs; body asymm etry,
limb hemi-hypertrophy,or uni lat eral odo- or chiromegaly (enlarged hand or
foot), perhaps from release of trophic factors via anterior horn cells; Syringo

57
CNS-System Wise 1700-by Sush and Team. 2016
Susmita, Asad, Manu, Saima, Zohaib, Savia, Shanu, Mona, Manisha, Sitara, Samreena, Sami and Komal

bulbia (brain stem involvement):nyst agmus, tongue atrophy,dysphagia,
pharyngeal/palatal weak ness,Vth nerve sensory loss. Charcot’s
(neuropathic)joints: Increased range of movement (from lost joint pro prio
ception), destroys the joint, which becomes swollen
and mobile (see fi g 2, p205).380 Causes: tabes dorsalis (eg knee), diabetic
neuropathy, paraplegia (eg hips),381 syringomyelia (shoulder,wrist),382
leprosy, spinal osteolysis/ cord atrophy (systemic sclerosis).382

Dissociated sensory loss- pain, and temp sensation lost, so will not feel . So
burn marks

Ascending progressive disorder- from tongue to hands to lower limbs, muscle
weakness

So tongue fasciculation is clincher

E MYASTHENIA GRAVIS-Increasing muscular fatigue. Muscle groups aff ected, in
order: extraocular;
bulbar (swallowing, chew ing); face; neck; limb girdle; trunk. Look for: ptosis,
diplopia, myasthenic snarl on smiling, ‘peek sign’.1 On counting to 50, the
voice fades (dysphonia is a rare presentation). Tendon refl exes are normal.
Weakness is
worsened by: pregnancy, K+DECREASES , infection, over-treatment, change of
climate, emotion, exercise, gentamicin, opiates, tetracycline, quinine,
procainamide, BETA blockers.

In ladies, problem with acetylecholine, affect muscle groups, gets worse at
night.

KEY C
Additional
Information
Reference
Dr Khalid/Rabia In MS, there are characteristic relapse and remission which is absent here.
MND is purely motor, there is no sensory deficit;

In myasthenia gravis there is muscular weakness without atrophy.

Syringomyelia is a condition in which there is fluid-filled tubular cyst (syrinx)
within the central, usually cervical, spinal cord. The syrinx can elongate,
enlarge and expand into the grey and white matter and, as it does so, it
compresses the nervous tissue of the corticospinal and spinothalamic tracts
and the anterior horn cells. This leads to various neurological symptoms and
signs, including pain, paralysis, stiffness and weakness in the back, shoulders
and extremities. It may also cause loss of extreme temperature sensation,
particularly in the hands, and a cape-like loss of pain and temperature

58
CNS-System Wise 1700-by Sush and Team. 2016
Susmita, Asad, Manu, Saima, Zohaib, Savia, Shanu, Mona, Manisha, Sitara, Samreena, Sami and Komal

sensation along the back and arms.
** If the syrinx extends into the brainstem, syringobulbia results. This may
affect one or more cranial nerves, resulting in facial palsies. Sensory and motor
nerve pathways may be affected by interruption and/or compression of
nerves.


Q:463 A young adult presents to the ED after a motorcycle crash. The pt has bruises
around the left orbital area. GCS=13, examination notes alcoholic breath.
Shortly afterwards, his GCS drops to 7.
What is the single most important initial assessment test?
a. MRI brain
b. CT brain
c. CXR
d. CT angio brain
e. Head XR

Clincher(s) GCS DROP OF 13 TO 7
A MRI BRAIN
B CT BRAIN
C CXR- NOT INDICATED FOR HEAD INJURY
D CT ANGIO BRAIN
E HEAD XR
KEY B
Additional GCS dropped to 7 so CT
Information
INDICATION OF CT

GCS <13 at initial assessment
• GCS <15 at 2h post injury
• Open or depressed skull or basal
skull fracture
• Post-traumatic fi t
• Focal CNS defi cit
• >1 episode of vomiting
• Retrograde amnesia >.h
Reference OHCS 729
Dr Khalid/Rabia


Q:475 A 32yo woman presents to the ED with headache and vomiting. She was
decorating her ceiling that morning when the headache began, felt mainly
occipital with neck pain. Some 2hs later she felt nauseated, vomited and was
unable to walk. She also noticed that her voice had altered. She
takes no reg meds and has no significant PMH. Exam: acuity, field and fundi
are normal. She has upbeat nystagmus in all directions of gaze with normal

59
CNS-System Wise 1700-by Sush and Team. 2016
Susmita, Asad, Manu, Saima, Zohaib, Savia, Shanu, Mona, Manisha, Sitara, Samreena, Sami and Komal

facial muscles and tongue movements.
Her uvulas deviated to the right and her speech is slurred. Limb exam: left arm
past-pointing and dysdiadochokinesis with reduced pin prick sensation in her
right arm and leg. Although power is Normal, she can’t talk as she feels too
unsteady. Where is the most likely site of lesion?
a. Right medial medulla
b. Left medial pons
c. Left cerebellar hemisphere
d. Right lateral medulla
e. Left lateral medulla
E
Clincher(s)
A Right medial medulla
B Left medial pons
C Left cerebellar hemisphere
D Right lateral medulla
E Left lateral medulla
KEY E
Additional Medial medullary syndrome
Information
Reference
Dr Khalid/Rabia Lateral medullary syndrome affects: (also involves cerebellar symptoms)
-Contralateral spinothalamic tract (loss of pain and temperature on the
opposite side of the body)
-Ipsilateral Sympathetic tract- Horner’s syndrome .
-Ipsilateral Spinal trigeminal nucleus (loss of pain,temperature and corneal
reflex on same side of the face)
-Nucleus ambigous- Dysphagia and Dysarthria
-Inferior cerebellar peduncle- Ataxia
Ipsilateral Cranial nerves- IX, X and XI (dysphagia, loss of gag reflex, palate
paralysis)
Cause- Occlusion of PICA (posterior inferior cerebellar artery)

Medial medullary syndrome affects: (
Contralateral corticospinal tract/pyramids- weakness of arms and legs
opposite side.
Contralateral Medial lemniscus/dorsal column- loss of proprioception and
vibration.
Ipsilateral hypoglossal nerve- weakness of tongue on the same side.
[Hypoglossal nerve affection manifests as protrusion of the tongue to the side
of the weakness while at rest, it deviates to the contralateral side)
Cause- Occlusion of Anterior spinal artery

60
CNS-System Wise 1700-by Sush and Team. 2016
Susmita, Asad, Manu, Saima, Zohaib, Savia, Shanu, Mona, Manisha, Sitara, Samreena, Sami and Komal


Q:502 For a pt presenting with Parkinson’s disease which of the following drugs is
most useful in the management of the tremor?
a. Apomorphine
b. Cabergoline
c. Selegiline
d. Amantadine
e. Benzhexol

Clincher(s)
A Apomorphine
B Cabergoline
C Selegiline
D Amantadine
E Benzhexol
KEY E
Additional
Information
Reference
Dr Khalid/Rabia Antimuscarinics
• block cholinergic receptors
• now used more to treat drug-induced parkinsonism rather than
idiopathic Parkinson's disease
• help tremor and rigidity
• e.g. procyclidine, benzotropine, trihexyphenidyl (benzhexol)

61
CNS-System Wise 1700-by Sush and Team. 2016
Susmita, Asad, Manu, Saima, Zohaib, Savia, Shanu, Mona, Manisha, Sitara, Samreena, Sami and Komal



Q:503 A 26yo woman has become aware of increasing right sided hearing deficiency
since her recent pregnancy. Her eardrums are normal. Her hearing tests show:
BC-normal. Weber test lateralizes to the right ear. What is the single most
likely dx?
• a. Encephalopathy
• b. Functional hearing loss
• c. Tympano-sclerosis
• d. Otosclerosis
• e. Sensorineural deafness



Clincher(s)
A Encephalopathy
B Functional hearing loss
C Tympano-sclerosis
D Otosclerosis
E Sensorineural deafness
KEY D
Additional
Information
Reference
Dr Khalid/Rabia key is D. (conductive deafness) Otosclerosis. [There are no features of
encephalopathy. As Weber test is lateralized it is unlikely to be functional
hearing loss. In tympanosclerosis ear drum becomes chalky white. So as the
ear drum is normal it is not tympanosclerosis. Weber test is lateralized to right
and deafness is also on the right. So it not sensorineural deafness but
conductive deafness which makes otosclerosis as the most likely diagnosis.
Rinne's test
· air conduction (AC) is normally better than bone conduction (BC)
· if BC > AC then conductive deafness

Weber's test
· in unilateral sensorineural deafness, sound is localised to the unaffected side
· in unilateral conductive deafness, sound is localised to the affected side



Q:537 537. A woman presented with blurred vision and intermittent clumsiness for
3m. Reflexes are brisk in her arm and optic disc is pale. What is the single most
appropriate test to confirm dx?
a. CSF analysis
b. CT
c. MRI

62
CNS-System Wise 1700-by Sush and Team. 2016
Susmita, Asad, Manu, Saima, Zohaib, Savia, Shanu, Mona, Manisha, Sitara, Samreena, Sami and Komal

d. EEG
e. EMG

Clincher(s)
A CSF analysis
B CT
C MRI
D EEG
E EMG
KEY C
Additional MS: Blurred vision and clumsiness (UMN lesion), optic disc pale: if given
Information replasing-

Triad for MS: SIN: Scanning speech, Intension tremor, Nystagmus
Young, white woman,

Reference
Dr Khalid/Rabia multiple sclerosis, investigation of choice is gadolinium enhanced mri.

Multiple sclerosis: features


• 3 times more common in women
• most commonly diagnosed in people aged 20-40 years

Patient's with multiple sclerosis (MS) may present with non-specific features,
for example around 75% of patients have significant lethargy.

Visual
• optic neuritis: common presenting feature
• optic atrophy
• Uhthoff's phenomenon: worsening of vision following rise in body
temperature
• internuclear ophthalmoplegia

Sensory
• pins/needles
• numbness
• trigeminal neuralgia
• Lhermitte's syndrome: paraesthesiae in limbs on neck flexion

Motor
• spastic weakness: most commonly seen in the legs

Cerebellar
• ataxia: more often seen during an acute relapse than as a presenting
symptom
• tremor

63
CNS-System Wise 1700-by Sush and Team. 2016
Susmita, Asad, Manu, Saima, Zohaib, Savia, Shanu, Mona, Manisha, Sitara, Samreena, Sami and Komal


Others
• urinary incontinence
• sexual dysfunction
• intellectual deterioration



Q:538 A 63yo man presents after having a seizure. Exam: alert, orientated,
inattention on the left side and hyperreflexia of the arm. What is the most
probable dx?
a. Cerebral tumor
b. Pituitary adenoma
c. Cerebellar abscess
d. Huntingtons chorea
e. Parkinsonism

Clincher(s)
A CEREBRAL TUMOR
B PITUITARY ADENOMA
C CEREBELLAR ABSCESS
D HUNTINGTON’S CHOREA
E PARKINSONISM
KEY A
Additional Cerebral tumor as it’s none of the others.
Information
Reference
Dr Khalid/Rabia Inattention or neglect is a feature of parietal lobe lesion. If lesion is on right
side there will be left sided inattention. That is patient is unaware of his left
side and he when shaves do it only to right half of face, during eating eats only
from the right half of plate and can not drive as he only aware of his right side
and totally unaware of left side of the road
nothing else fit thats the best expl.. pituitary will give bitemporal vision loss
cerebellar signs are nystagmus ataxia etc. Chorea is repeated movements.. its
focal ant lobe lesion.
Seizure n hyperflexia (exaggerated reflexes) are indicators of space occupied
lesion (SOL)
Because the pt has got upper motor neuron signs.
Its not pituitary because no signs of optic n compression and its not bilateral.
Its not cerebellar dis other wise he should have ataxia, pass pointing
rombergism.
Its not chorea cuz no symptoms of chorea.
Not parkinson because no hypokinesia, tremor
seizure causing neuro deficit in elderly... first D/D should b cerebral tumour
unless specified otherwise

64
CNS-System Wise 1700-by Sush and Team. 2016
Susmita, Asad, Manu, Saima, Zohaib, Savia, Shanu, Mona, Manisha, Sitara, Samreena, Sami and Komal


Q:539 A 40yo man with a 25y hx of smoking presents with progressive hoarseness of
voice, difficulty swallowing and episodes of hemoptysis. He mentioned that he
used to be a regular cannabis user. What is the single most likely dx?
a. Nasopharyngeal cancer
b. Pharyngeal carcinoma
c. Sinus squamous cell carcinoma
d. Squamous cell laryngeal cancer
e. Hypopharyngeal tumor


Clincher(s)
A Nasopharyngeal cancer
B Pharyngeal carcinoma
C Sinus squamous cell carcinoma
D Squamous cell laryngeal cancer
E Hypopharyngeal tumor
KEY D
Additional Larynx- involvement
Information
Reference
Dr Khalid/Rabia Hoarseness of voice is localizing the problem to the larynx
The symptoms of cancer of the pharynx differ according to the type:
· Oropharynx: common symptoms are a persistent sore throat, a lump in the
mouth or throat, pain in the ear.
· Hypopharynx: problems with swallowing and ear pain are common
symptoms and hoarseness is not uncommon.
· Nasopharynx: most likely to cause a lump in the neck but may also cause
nasal obstruction, deafness and postnasal discharge.

Laryngeal Cancer
· Smoking is the main avoidable risk factor for laryngeal cancer
· Chronic hoarseness is the most common early symptom.
· urgent CXR to decide where to refer
· Flexible laryngoscopy is the best way to inspect the larynx
· staging include CT and/or MRI scans.




Q:1001 An 82yo man has woken up with incoherent speech and difficulty in finding the
right words. Exam: otherwise normal, good comprehension. Which anatomical
site is most likely to be affected?
a. Broca’s area
b. Wernicke’s area
c. Midbrain

65
CNS-System Wise 1700-by Sush and Team. 2016
Susmita, Asad, Manu, Saima, Zohaib, Savia, Shanu, Mona, Manisha, Sitara, Samreena, Sami and Komal

d. Parietal cortex
e. Pons

Clincher(s) Incoherent speech, difficulty in finding right words
A Broca’s (expressive) anterior dysphasia: Non-fl uent speech produced with
effort and frustration with malformed words, eg ‘spoot’ for ‘spoon’ (or ‘that
thing’).Reading and writing are impaired but comprehension is relatively
intact. Patients understand questions and attempt to convey meaningful
answers. Site of lesion:infero-lateral dominant frontal lobe

He knows and understands but cannot tell you

B Wernicke’s (receptive) posterior dysphasia: Empty, fluent speech, like talking
ragtime with phonemic (‘fl ush’ for ‘brush’) and semantic (‘comb’ for ‘brush’)
paraphasias/neologisms (may be mistaken for psychotic speech). The patient is
oblivious of errors. Reading, writing, and comprehension are impaired (replies
are
inappropriate). Site of lesion: posterior superior dominant temporal lobe.

Comprehensions are always affected: words without meaning WC- Wernick
comprehension

C The midbrain or mesencephalon is a portion of the central nervous system
associated with vision, hearing, motor control, sleep/wake, arousal (alertness),
and temperature regulation.
D
E
KEY A
Additional
Information


Reference
Dr Khalid/Rabia Reason: Broca’s area is associated with motor part of speech, incoherent
speech, though the patient is able to understand speech.
Wernicke area is associated with sensory part of speech ,it affects
understanding and as well as speech production


Q:1004 A 35yo man presents with a headache that worsens on bending his head forward.
What is the most likely dx?
a. Chronic sinusitis

66
CNS-System Wise 1700-by Sush and Team. 2016
Susmita, Asad, Manu, Saima, Zohaib, Savia, Shanu, Mona, Manisha, Sitara, Samreena, Sami and Komal

b. SAH
c. Migraine
d. Cluster headache
e. Tension headache

Clincher(s) Headache that worsens on bending head
A Pain:maxillary(cheek,teeth),ethmoidal(between eyes)worst on bending
head.Associated with nasal congestion,post nasal dripping,anosmia and systemic
symptom e.g.fever.
B The most characteristic feature of sub arachnoid hemorhageis a sudden explosive
headache at the back of the head(occipital headache). Sudden (usually, but not
always, within seconds) devastating typically occipital headache—“I tho ught I’d
been kicked I n the head”. Vomiting, collapse seizures and coma often follow.
Coma/drowsiness may last for days.Sudden onset headache is always subarachnoid
hemorahage until proven otherwise.
C Migraine:Classically: •Visual or other aura (see below) lasting 15–30min followed
within 1h by unilateral, throbbing headache. Or: •Isolated aura with no headache;
•Epi sodic severe headaches without aura, often premenstrual, usually unilateral,
with
nausea, vomiting ± photophobia/phono phobia (‘common migraine’). There may be
allodynia—all stimuli produce pain: “I can’t brush my hair, wear earrings or glasses,
or
shave, it’s so pain ful”. Si
D Cluster headache: Rapid-onset of excruciating pain around one eye that may
become watery and bloodshot with lid swelling, lacrimation, noctornul facial fl
ushing, rhinorrhoea,miosis ± ptosis (20% of attacks). Pain is strictly unilateral and
almost always affectsthe same side. It lasts 15–160min, ccurs once or twice a day,
and is often nocturnal. Clusters last 4–12wks and are followed by pain-free periods
of months or even 1–2yrs before the next cluster. Sometimes it is chronic, not
episodic similar to migrane but no aura) cluster is associated with male and
somokers. Migraine more in F

E Tension: The usual cause of bilateral, non-pulsatile headache } scalp
muscle tenderness, but without vomiting or sensitivity to head movement.
Stress relief, eg massage or antidepressants, may be helpful
KEY A



67
CNS-System Wise 1700-by Sush and Team. 2016
Susmita, Asad, Manu, Saima, Zohaib, Savia, Shanu, Mona, Manisha, Sitara, Samreena, Sami and Komal

68
CNS-System Wise 1700-by Sush and Team. 2016
Susmita, Asad, Manu, Saima, Zohaib, Savia, Shanu, Mona, Manisha, Sitara, Samreena, Sami and Komal

Additional
Information

69
CNS-System Wise 1700-by Sush and Team. 2016
Susmita, Asad, Manu, Saima, Zohaib, Savia, Shanu, Mona, Manisha, Sitara, Samreena, Sami and Komal

70
CNS-System Wise 1700-by Sush and Team. 2016
Susmita, Asad, Manu, Saima, Zohaib, Savia, Shanu, Mona, Manisha, Sitara, Samreena, Sami and Komal

Reference OHCM

Dr Khalid/Rabia



Q:1007 A 34yo male presents with headache and vomiting. Exam: temp=38.5C, neck
stiffness, discharge from left ear and right sided hyper-reflexia with an
extensor plantar response. What is the most likely dx?
a. Cerebral tumor
b. Meningitis
c. Cerebellar tumor
d. Cerebral abscess
e. Normal pressure hydrocephalus

Clincher(s) Headache and vomiting, temp.38.5c, right sided hyper reflexia
A Fever excludes tumor
B Focal neurological signs are absent in meningitis
C
D Onset may be sudden or subacute over several weeks.
• Common presenting symptoms include fever, headache, changes in
mental state (drowsiness, confusion), focal neurological deficits,
grand mal seizures, nausea and vomiting, neck stiffness.
• A suddenly worsening headache, followed by emerging signs of
meningism, are often associated with rupture of the abscess.
Signs
• Fever.
• Focal motor or sensory deficits.
• Raised blood pressure and bradycardia associated with raised
intracranial pressure.
• Papilloedema.
• Ataxia.
• Confusion, drowsiness.
• Bulging fontanelle in infants.
The symptoms of brain abscess are caused by a combination of increased
intracranial pressure due to a space-occupying lesion (headache, vomiting,
confusion, coma), infection (fever, fatigue etc.) and focal neurologic brain
tissue damage (hemiparesis, aphasia etc.). The most frequent presenting
symptoms are headache, drowsiness, confusion,seizures, hemiparesis or
speech difficulties together with fever with a rapidly progressive course. The
symptoms and findings depend largely on the specific location of the abscess
in the brain
Diagnosis: Ct brain , shows space occupying lesion after a few days there is ring
enhancement .
Management: IV antibiotics , sensitivity should be done . hyperbaric oxygen
therapy .
1008.

71
CNS-System Wise 1700-by Sush and Team. 2016
Susmita, Asad, Manu, Saima, Zohaib, Savia, Shanu, Mona, Manisha, Sitara, Samreena, Sami and Komal


E Normal pressure hydrocephalus (NPH) describes the condition of ventricular
dilatation in the absence of raised CSF pressure on lumbar puncture,
characterised by a triad of gait abnormality, urinary (usually) incontinence
and dementia.
KEY D (discharge – due to complication of ) hyper-reflexia s focal neurological sign
for this, in contrast to meningitis.
Additional
Information
Reference Patient.info
Dr Khalid/Rabia


Q:1008 A 26yo male presents with speech difficulties. Exam: nystagmus. Which anatomical site is most
likely to be affected?
a. Midbrain
b. Pons
c. Cerebellum
d. Cerebrum
e. Vestibule cochlear nerve

Clincher(s)
A


B There are many important nerves that originate in the pons. The trigeminal nerve is responsible for
feeling in the face. I also controls the muscles that are responsible for biting, chewing, and
swallowing. The abducens nerve allows the eyes to look from side to side. The facial nerve controls
facial expressions, and the vestibulocochlear nerve allows sound to move from the ear to the brain.
All of these nerves start within the pons.
C

D
E VESTIBULOCHOLEAR nerve injury results in dizziness,vertigo and hearing loss
KEY C
Additional
Information
Reference ohcm
Dr
Khalid/Rabia

72
CNS-System Wise 1700-by Sush and Team. 2016
Susmita, Asad, Manu, Saima, Zohaib, Savia, Shanu, Mona, Manisha, Sitara, Samreena, Sami and Komal


Q:1011 A 54yo pt wakes up with right sided weakness. His current medication is
bendroflumethiazide for HTN. Pulse=92bpm, BP=160/90mmHg. CT shows left
cerebral infarct. What is the most appropriate tx?
a. Alteplase
b. Aspirin
c. Clopidogrel
d. Dipyridamole
e. Simvastatin

Clincher(s)
A If a patient with ischemic stroke presents within 4.5 hours of ischemic episode
thrombolysis can be tried.
B
C
D
E
KEY B (<4.5 – alteplase)
Additional
Information
Reference
Dr Khalid/Rabia

Q:1012 A 33yo man presented to the GP with hx of headaches and photophobia. The
GP examines him and finds a rash and is now ringing you at the hospital for
advice. What would you advice the GP?
a. Send pt home
b. Start IV benzylpenicillin (prevents spreading)
c. Conduct LP
d. Start IV ceftriaxone

Clincher(s) Headache,photophobia,skin rash (meningococcal meningitis)
A
B • Intramuscular or intravenous benzylpenicillin should be given before
urgent transfer to hospital only if there is suspected meningococcal
septicaemia with a non-blanching rash.

C LP should be done at the hospital only
D Can be started in hospital
E
KEY B (bp only when there is rash only)
Additional Meningitis is an inflammation of the leptomeninges and underlying
Information subarachnoid cerebrospinal fluid (CSF).[1] The inflammation may be caused by
infection with viruses, bacteria, other micro-organisms, or non-infective
causes.

73
CNS-System Wise 1700-by Sush and Team. 2016
Susmita, Asad, Manu, Saima, Zohaib, Savia, Shanu, Mona, Manisha, Sitara, Samreena, Sami and Komal

Viral meningitis is more common and usually more benign than bacterial
meningitis but all cases of suspected meningitis should be managed as though
having bacterial meningitis, until proven otherwise. Meningococcal disease is
the leading infectious cause of death in early childhood

Clinical presentation of meningitis may include:[2]

• Fever, headache.
• Stiff neck (generally not present in children under the age of one year
or in patients with altered mental state), back rigidity, bulging
fontanelle (in infants), photophobia, opisthotonus (if severe).
• Altered mental state, unconsciousness, toxic/moribund state.
• Shock: signs of shock include tachycardia and/or hypotension,
respiratory distress, altered mental state and poor urine output.
• Kernig's sign (pain and resistance on passive knee extension with hips
fully flexed).
• Brudzinski's sign (hips flex on bending the head forward).
• Paresis, focal neurological deficits (including cranial nerve involvement
and abnormal pupils).
• Seizures.

Perform lumbar puncture (unless contraindicated)(Nice)

Perform a lumbar puncture as a primary investigation unless this is


contraindicated.(contraindication is raised ICP)
If there are contraindications, consider delaying lumbar puncture until there
are no longer contraindications.
Do not allow lumbar puncture to delay the administration of parenteral
antibiotics.
Submit CSF to the laboratory to hold for PCR testing for N meningitidis and S
pneumoniae, but only perform the PCR testing if the CSF culture is negative.
Be aware that CSF samples taken up to 96 hours after admission to hospital
may give useful PCR results.
If raised ICP then do CT.
Management:
• Transfer any patient with suspected bacterial meningitis or suspected
meningococcal septicaemia to secondary care as an emergency by
telephoning 999112/911.
• Intramuscular or intravenous benzylpenicillin should be given before
urgent transfer to hospital only if there is suspected meningococcal
septicaemia with a non-blanching rash.
• Benzylpenicillin should not be given if there is a history of anaphylaxis
associated with penicillins or if giving antibiotics will delay urgent
transfer to hospital.
• If urgent transfer to hospital is not possible (eg, remote locations or
adverse weather conditions), antibiotics should be given to any person
with suspected bacterial meningitis.

74
CNS-System Wise 1700-by Sush and Team. 2016
Susmita, Asad, Manu, Saima, Zohaib, Savia, Shanu, Mona, Manisha, Sitara, Samreena, Sami and Komal


• Management includes supportive treatment with analgesia,


antipyretics, nutritional support and hydration.
• Do not restrict fluids unless there is evidence of raised intracranial
pressure or increased antidiuretic hormone (ADH) secretion.[2]
• The choice of antibiotics and the duration of therapy should be guided
by the microbiological diagnosis but initial 'blind' antibiotic therapy
must be started immediately
• The National Institute for Health and Care Excellence (NICE)
recommendation to children (over 3 months old) is for dexamethasone
to be given for suspected or confirmed bacterial meningitis as soon as
possible.[2]
• Corticosteroids given to patients of all ages with bacterial meningitis
have been shown to reduce hearing loss and neurological sequelae
significantly but there is no evidence that they reduce overall
mortality.[16]
• Choice of antibiotic is usually determined by local guidelines and close
liaison with a microbiologist.
• Initial blind therapy: · Children 3 months and older and young people
should be given intravenous ceftriaxone as empirical treatment before
identification of the causative organism. If calcium-containing infusions
are required at the same time, cefotaxime is preferable.
• · Children younger than 3 months should be given intravenous
cefotaxime plus either amoxicillin or ampicillin. NB: ceftriaxone should
not be used in premature babies or in babies with jaundice,
hypoalbuminaemia or acidosis,


Reference NICE,Patient.info
Dr Khalid/Rabia

Q:1015 1015. A 42yo man presents with stroke. He is not able to walk straight and his
speech is slurred. What is the initial appropriate inv?
a. CT brain
b. PET brain
c. MRI brain
d. Carotid angiography
e. Monitor for 24h

Clincher(s) Not able to walk straight(ataxia), slurred speech(dysarthia)

75
CNS-System Wise 1700-by Sush and Team. 2016
Susmita, Asad, Manu, Saima, Zohaib, Savia, Shanu, Mona, Manisha, Sitara, Samreena, Sami and Komal

A
B
C MRI is the modality of choice for cerebellar stroke
D
E
KEY C
Additional For every cerebellar lesion> MRI;
Information
for ischaemic stroke: MRI (if there are symptoms of carotic bruit/heart
involvement),

For hemorrhagic: CT (symp of internal bleeding/ raised inter-cranial pressure)

Reference
Dr Khalid/Rabia

Q:1029 A 35yo woman presents with visual problems. CT brain reveals pituitary tumor.
What is the single most likely defect?
a. Homonymous hemianopia
b. Homonymous upper quadrantopia
c. Bitemporal hemianopia
d. Cortical blindness
e. Homonymous lower quadrantopia

Clincher(s) Visual problem,Pituitary tumor
A In homonymous hemianopia - literally, a loss of vision on the same side in both
eyes - the field loss is usually the result of a lesion in the optic radiation or tract
on one side. The visual defect is dependent on the site of the lesion:
• right-hand sided lesions lead to left temporal and right nasal field loss
• left-hand sided lesions lead to right temporal and left nasal field loss

B upper homonymous quadrantanopia describes the loss of the same upper
quadrant from each visual field.
Upper homonymous quadrantanopias are usually caused by damage to the
optic radiation as it passes throught the temporal lobes.
For example a lesion in the left temporal lobe will cause a right homonymous
upper quadrantanopia, that is the loss of the upper nasal quadrant from the
left eye and the upper temporal quadrant from the right eye.

C Bilateral hemianopia is a loss of temporal field vision in both eyes. This may be
caused by a lesion that affects the centre of the optic chiasm and damages
fibres from the nasal halves of the retina as they decussate.

Possible causes of this condition include:

• pituitary tumour

76
CNS-System Wise 1700-by Sush and Team. 2016
Susmita, Asad, Manu, Saima, Zohaib, Savia, Shanu, Mona, Manisha, Sitara, Samreena, Sami and Komal

• craniopharyngioma
• suprasellar meningioma


D Bilateral lesions of the primary visual cortex may cause blindness.
The cortically blind patient may have no vision but:
• pupillary responses are intact:
o fibres terminate in the pretectal region of the midbrain and do
not involve the cortex
• normal fundoscopy
• normal visual imagination and dreamin

E Homonymous inferior quadrantanopia is when the lower quadrant of the
visual field ceases to function. Homonymy refers to the fact that the same
quadrant (meaning both left or both right) is nonfunctional in each eye. For
right homonymous inferior quadrantanopia, both lower right quadrants of the
visual field experience loss of visual function; the cause of this visual defect is
commonly located in the left hemisphere. Left homonymous inferior
quadrantanopia involves visual defects in the lower left quadrant of each eye,
with the cause usually being located in the right hemisphere of the brain.
KEY C
Additional
Information


Reference GP notebook
Dr Khalid/Rabia SEE DIAGRAM to understand (important and to read from Samson)

Q:1035 A 40yo woman presents with dysphagia. Exam: febrile with erythema and
middle neck swelling. What is the best management strategy?
a. IV antibiotics and drainage
b. Antipyretics
c. XR neck

77
CNS-System Wise 1700-by Sush and Team. 2016
Susmita, Asad, Manu, Saima, Zohaib, Savia, Shanu, Mona, Manisha, Sitara, Samreena, Sami and Komal

d. Endoscopic diverticulectomy
e. I&D


Clincher(s) Febrile, middle neck swelling
A Neck abcess, so treatment is IV antibiotics and drainage
B
C
D
E
KEY A
Additional
Information
Reference
Dr Khalid/Rabia


Q:1043 A 50yo man with a known hx of stroke. He can’t remember anything about his
life. What is the single most likely defect?
a. Homonymous hemianopia
b. Homonymous upper quadrantanopia
c. Bitemporal hemianopia
d. Binasal hemianopia
e. Homonymous lower quadrantanopia

Clincher(s)
A
Memory storing site is in the temporal lobe. Lesion of the temporal lobe leads
to homonymous upper quadrantanopia.


B
C
D
E
KEY B (to posted/discussed)
Additional Already explained in q 1029
Information
Reference
Dr Khalid/Rabia


Q: 177 A 4yo baby has generalized tonic-clonic seizure and fever of 39C. his mother
informs you that this has happened 3-4x before. What is the most likely dx?
a. Febrile convulsion

78
CNS-System Wise 1700-by Sush and Team. 2016
Susmita, Asad, Manu, Saima, Zohaib, Savia, Shanu, Mona, Manisha, Sitara, Samreena, Sami and Komal

b. Absence seizures
c. Epilepsy
d. Partial complex seizure

Clincher(s)
A
B Brief..less than 10second pauses, eg suddenly stops talking in miod sentence,
then carries on where left off.
C
D Awareness is impaired, may have aura or impaired awareness at onset.
E
KEY The key is C. Epilepsy! Probably wrong key! Epilepsy doesn’t occur with fever!
Likely correct key is A. Febrile convulsion.

Additional FEBRILE CONVULSION is a single tonic–clonic, symmetrical generalized seizure
Information lasting <20min, occurring as T° rises rapidly in a febrile illness—typically in a
normally developing child (½–5yrs old).
Think of meningo-encephalitis, CNS lesion, epilepsy, trauma, hypoglycemia,
dec Ca2+, or dec Mg2+ if: • Focal CNS signs or CNS abnormality • Previous
history of epilepsy • The seizure lasts >15min • There is >1 attack in 24h.
Examination: Find any infection; if any neck stiffness consider meningitis. :
Management: Put in recovery position; if fit is lasting >5min: lorazepam IV,
buccal midazolam or diazepam PR. Tepid sponging if hot; paracetamol syrup
Labs: Consider FBC, U&E, Ca2+, glucose, MSU, CXR, ENT swabs.
Avoid LP in the postictal period. If you suspect meningitis, then treat now.
Parental education: Allay fear (a child is not dying during a fit). For the 30%
having
recurrences, teach carers to use buccal midazolam or rectal diazepam 0.5mg/
kg
Further prevention: Diazepam PR during fevers has a role; other
anticonvulsants are ‘never’ needed. Explain that all fevers (eg vaccination-
associated) should prompt oral antipyretics.
Prognosis: In typical febrile convulsions there is no progress to epilepsy in 97%.

Reference
Dr Khalid/Rabia


Q: 197 . A young girl complains of episodic headaches preceded by fortification
spectra. Each episode
last for 2-3 days. During headache pt prefers quiet, dark room. What is the tx
of choice for acute
stage?
a. Paracetamol
b. Aspirin
c. Sumatriptan

79
CNS-System Wise 1700-by Sush and Team. 2016
Susmita, Asad, Manu, Saima, Zohaib, Savia, Shanu, Mona, Manisha, Sitara, Samreena, Sami and Komal

d. Gabapentin
e. Cafergot


Clincher(s)
A
B
C
D
E
KEY The key is B. Aspirin. [OHCM, 9th Eition, page-462 where NSAIDS like
ketoprophen or dispersible aspirin 900 mg/6 hr are recommended as
treatment in acute stage].


Additional Migraine:
Information
Symptoms Classically: •Visual or other aura lasting 15–30min followed within
1h by unilateral, throbbing headache. Or: •Isolated aura without headache;
•Episodic severe headaches without aura, often premenstrual, usually
unilateral, with nausea, vomiting ± photophobia/phonophobia (‘common
migraine’). There may be allodynia—all stimuli produce pain: “I can’t brush my
hair, wear earrings or glasses, or shave, it’s so painful”.
Signs: None.
Aura: • Visual: chaotic cascading, distorting, ‘melting’ and jumbling of lines,
dots, or zigzags, scotomata or hemianopia; • Somatosensory: paresthesia
spreading from fingers to face; • Motor: dysarthria and ataxia (basilar
migraine), ophthalmoplegia, or hemiparesis; • Speech: (8% of auras) dysphasia
or paraphasia, eg phoneme substitution.
Criteria for diagnosis if no aura ≥5 headaches lasting 4–72h + nausea/vomiting
(or photo/phonophobia) + any 2 of: • Unilateral • Pulsating • Impairs (or
worsened by) routine activity.
Partial triggers Seen in 50%: CHOCOLATE or: Chocolate, Hangovers,
Cheese, Oral contraceptives, Lie-ins, Alcohol, Tumult, or Exercise.

Treatment:
Acute:
Step one: simple analgesic with or without anti-emetic. In patients who have
tried step 1 and didnt respond and in patients with moderate-to-severe
migraine, move to step three.

• Use early in the attack to avoid gastric stasis.
• Use soluble aspirin 600-900 mg (not in children) or ibuprofen 400-
600 mg. Use prochlorperazine 3 mg buccal tablet if there is nausea
and vomiting.

80
CNS-System Wise 1700-by Sush and Team. 2016
Susmita, Asad, Manu, Saima, Zohaib, Savia, Shanu, Mona, Manisha, Sitara, Samreena, Sami and Komal

Step two: rectal analgesia and rectal anti-emetic.

Step three: specific anti-migraine drugs Triptans (5HT1-receptor agonists) or


ergotamine (the use of ergotamine is limited by absorption problems and side-
effects such as nausea, vomiting and abdominal pain

Triptans are Contra indicated if IHD, coronary spasm, uncontrolled BP, recent
lithium, SSRIS,
or ergot use.

Prevention Remove triggers; ensure analgesic rebound headache is not
complicating
matters. Drugs eg if frequency equal or >2 a month or not responding to
drugs—
1st-line: Propranolol, amitriptyline (SE: drowsiness, dry mouth, vision),
topiramate (SE: memory) or Ca2+ channel blockers.
2nd-line: Valproate, pizotifen (effective, but unacceptable weight gain in
some), gabapentin, pregabalin, ACE-i, NSAIDS



Reference Ohcm 462
Dr Khalid/Rabia


Q: 200 A 77yo male presents with hx of enuresis and change in behavior. Exam:
waddling gait. What is the most likely dx?
a. Subdural hemorrhage
b. Brain tumor
c. Normal pressure hydrocephalus
d. Psychotic depression


Clincher(s)
A
B
C
D
E
KEY The key is C. Normal pressure hydrocephalus. [age (usually occurs in 60s or
70s), loss of bladder control (enuresis), waddling gait and behavior change are
all features of normal pressure hydrocephalus].

Additional Normal Pressure Hydrocephalus:

81
CNS-System Wise 1700-by Sush and Team. 2016
Susmita, Asad, Manu, Saima, Zohaib, Savia, Shanu, Mona, Manisha, Sitara, Samreena, Sami and Komal

Information describes the condition of ventricular dilatation in the absence of raised CSF
pressure on lumbar puncture, characterised by a triad of gait abnormality,
urinary (usually) incontinence and dementia.

Cause: Idiopathic or it may be secondary to:

Subarachnoid haemorrhage. Meningitis. Head injury. Central nervous system


(CNS) tumour.

Symptoms: The (gradually progressive) classic triad of symptoms is:

• Gait disturbance - this is due to distortion of the corona radiata by


the dilated ventricles. Movements are slow, broad-based and
shuffling. The clinical impression is thus one of Parkinson's disease,
except that rigidity and tremor are less marked and there is no
response to carbidopa/levodopa. Gait disturbance is referred to as
gait apraxia.
• Sphincter disturbance - this is also due to involvement of the sacral
nerve supply. Urinary incontinence is predominant although bowel
incontinence can also occur.
• Dementia - this is due to distortion of the periventricular limbic
system. The prominent features are memory loss, inattention,
inertia and bradyphrenia (slowness of thought). The dementia
progresses less rapidly than that seen with Alzheimer's disease.

Signs
• Pyramidal tract signs may be present.
• Reflexes may be brisk.
• Papilloedema is absent (but there has been found to be an
association with glaucoma, so glaucomatous optic disc changes
may be noticed).

Investigations

• Neuroimaging - MRI or CT scanning may show ventricular


enlargement out of proportion to sulcal atrophy and
periventricular lucency.
• CSF: Large-volume lumbar puncture (spinal or CSF tap test) - CSF
pressure will be normal, or intermittently raised.
• Intraventricular monitoring

Management:
Medical treatment of NPH includes acetazolamide and repeated lumbar
puncture.

82
CNS-System Wise 1700-by Sush and Team. 2016
Susmita, Asad, Manu, Saima, Zohaib, Savia, Shanu, Mona, Manisha, Sitara, Samreena, Sami and Komal

Surgical:

The mainstay of treatment is surgical insertion of a CSF shunt. This could be to


the peritoneum, the right atrium or, more recently, via external lumbar
drainage.



Reference
Dr Khalid/Rabia


Q: 227 A 32 yo woman suffers an episode of severe occipital headache with vomiting
and loss of
consciousness. She is brought to the hospital where she is found to be
conscious and completely alert. Exam: normal pulse & BP. No abnormal
neurological signs. What is the next step in management?
a. Admission for observation
b. CT brain
c. MRI head
d. Reassurance and discharge
e. XR skull


Clincher(s)
A
B
C
D
E
KEY The key is B. CT brain.
Additional basilar migraine can cause severe headache and LOC. But there occurs no
Information neurological deficit and recovering from unconsciousness becomes completely
alert. But to diagnose basilar migraine there should at least history of two
migraine attacks with aura. As here diagnostic criteria of basilar migraine is not
fulfilled we can not discharge the patient without neuroimaging like CT or MRI.

Reference
Dr Khalid/Rabia


Q: 232 A 63 yo female is noted to have left pupil unresponsive to light and is dilated.
What is the most probably dx?
a. Pontine hemorrhage

83
CNS-System Wise 1700-by Sush and Team. 2016
Susmita, Asad, Manu, Saima, Zohaib, Savia, Shanu, Mona, Manisha, Sitara, Samreena, Sami and Komal

b. Subdural hemorrhage
c. Cerebellar hemorrhage
d. Extradural hemorrhage
e. Subarachnoid hemorrhage


Clincher(s)
A Here there will be pin point pupil
B Local neurological signs like unequal pupils occurs late.
C Will present with other symptoms
D If bleeding continues or is prolonged then ipsilateral pupil dilates
E (will post and change)
KEY Incomplete scenario.
Original key was D. Extradural hemorrhage. It is a wrong key. Correct key is E
subarachanoiod hemorrhage. Third nerve palsy due to a posterior
communicating artery aneurysm.


In a 63 year old extradural hemorrhage is extremely unlikely. As no clinical
picture is described in question except 3rd nerve palsy E. SAH is more logical
answer!

Additional Sub Arachnoid Haemorrhage
Information Spontaneous bleeding into the subarachnoid space is often catastrophic
Causes: Rupture of saccular aneurysms (80%); arteriovenous malform ations
(AVM; 15%). No cause
is found in <15%.
Risk factors: Smoking, alcohol misuse, inc BP, bleeding disorders, mycotic
aneurysm (SBE), perhaps post-menopausal oestrogen >45yrs old).

Berry aneurysms Common sites: junction of posterior communicating with the
internal carotid or of the anterior communicating with the anterior cerebral
artery or bifurcation of the middle cerebral artery. Some are hereditary.
Associations: Polycystic kidneys, coarctation of the aorta, Ehlers–Danlos
syndrome

Symptoms Sudden (usually, but not always, within seconds) devastating
typically occipital headache—“I thought I’d been kicked in the head”.
Vomiting, collapse, seizures and coma often follow. Coma/drowsiness may last
for days.
Signs Neck stiffness, Kernig’s sign (takes 6h to develop), retinal, subhyaloid and
vitreous bleeds (=Terson’s syndrome; it carries a worse prognosis) Focal
neurology at presentation may suggest site of aneurysm (eg pupil changes
indicating a IIIrd nerve palsy with a posterior communicating artery aneurysm)
or intracerebral haematoma. Later deficits suggest complications

84
CNS-System Wise 1700-by Sush and Team. 2016
Susmita, Asad, Manu, Saima, Zohaib, Savia, Shanu, Mona, Manisha, Sitara, Samreena, Sami and Komal

Tests CT detects >90% of SAH within the 1st 48h. LP if CT –ve and no
contraindication >12h after headache onset. CSF in SAH is uniformly bloody
early on, and becomes xanthochromic (yellow) after several hours due to
breakdown products of Hb (bilirubin). Finding xanthochromia confirms SAH

Management Refer all proven SAH to neurosurgery immediately.
• Re-examine CNS often; chart BP, pupils and GCS. Repeat CT if deteriorating.
• Maintain cerebral perfusion by keeping well hydrated, and aim for SBP
>160mmHg. Treat BP only if very severe.
• Nimodipine is a Ca2+ antagonist that reduces vasospasm and consequent
morbidity from cerebral ischaemia.
• Endovascular coiling is preferred to surgical clipping where possible. Do
catheter
or CT angiography to identify single vs multiple aneurysms before intervening.
Intracranial stents and balloon remodelling enable treating wide-necked
aneurysms.

Complications Rebleeding is the commonest cause of death, and occurs in
20%,
often in the 1st few days. Cerebral ischaemia due to vasospasm may cause a
permanent
CNS deficit, and is the commonest cause of morbidity. Hydrocephalus, due to
blockage
of arachnoid granulations, requires a ventricular or lumbar drain.
Hyponatraemia
is common but should not be managed with fluid restriction. Seek expert help


Reference Ohcm 482
Dr Khalid/Rabia

Q: 239 A 27 yo woman has hit her neck in an RTA without complains of tingling or
motor loss. What is the next most appropriate inv?
a. MRI
b. XR
c. CT cervical
d. Diagonal XR


Clincher(s) Rta. No complains of sensory or motor loss.
A
B
C
D
E

85
CNS-System Wise 1700-by Sush and Team. 2016
Susmita, Asad, Manu, Saima, Zohaib, Savia, Shanu, Mona, Manisha, Sitara, Samreena, Sami and Komal

KEY The key is B. X-ray. Next appropriate investigation (C1- T7) level
Additional As there is no neurological deficit we can exclude any fracture by x-ray
Information first].[Diagonal x ray means ,oblique view of cervical spine. By this view we can
assess facet joint arthopathy. This doesn't related to RTA.

Reference
Dr Khalid/Rabia

Q: 248 A 74yo man presents with weakness in his arm and leg from which he
recovered within a few days and short term memory loss. He has an extensor
plantar response. He has similar episodes 2 years ago and became unable to
identify objects and to make proper judgment. What is the most appropriate
dx?
a. Alcoholic dementia
b. Pick’s dementia
c. Huntington’s disease
d. Alzheimer’s disease
e. Vascular dementia


Clincher(s)
A
B Dementia, aphasia, parkinsonian aspect, associated with picks bodies.
C Autosomal dominant. Chorea and dementia. Early onset
D
E
KEY The key is E. Vascular dementia (age, hx of weakness, memory loss) It
represents cumulative effect of many small strokes
Additional Vascular dementia. [hemiparesis, memory impairment, extensor plantar reflex,
Information inability to identify objects, poor judgment are features of vascular dementia].

Vascular dementia: ~25% of all dementias. It represents the cumulative effects
of many small strokes, thus sudden onset and stepwise deterioration is
characteristic
Look for evidence of vascular pathology (BP, past strokes, focal CNS signs).

Diagnosis:

• Presence of dementia - cognitive decline from higher level of
functioning. This can be demonstrated as memory loss plus
impairment in two or more different cognitive domains . This
should be established by clinical examination and
neuropsychological testing. Deficits should be severe enough to
interfere with activities of daily living - not secondary effects of the
cerebrovascular event alone.
• Cerebrovascular disease, defined by the presence of signs on

86
CNS-System Wise 1700-by Sush and Team. 2016
Susmita, Asad, Manu, Saima, Zohaib, Savia, Shanu, Mona, Manisha, Sitara, Samreena, Sami and Komal

neurological examination and/or by brain imaging.
• A relationship between the above two disorders inferred by:
o Onset of dementia within three months following a
recognised stroke.
o An abrupt deterioration in cognitive functions.
o Fluctuating, stepwise progression of cognitive deficits.

Management: Like other dementias the treatment is symptomatic, addressing


the individual's main problems and supporting the carers. Detecting and
addressing cardiovascular risk factors is also very important to try to slow
progression




Reference Ohcm 490…dr rabia and dr khalid
Dr Khalid/Rabia


Q: 257 257. A 29yo woman who was dx to have migraine presents with severe onset
of occipital headache.
She lost her consciousness. CT=normal. Neurological exam=normal. What is
the most
appropriate management?
a. Repeat CT
b. MRI
c. LP
d. XR
e. No inv required


Clincher(s)
A
B
C
D
E
KEY Key = E (no investigation required)
Or C?? rule out SAH? Absence of neurological s/s does not rule out SAH. In
SAH more than 90 percent cases, CT will detect . LP will conclude SAH but
time period not given as we need to wait for 12 hours for LP. LP is
contraindicated in raised intracranial pressure.

Additional Points in favour = occipital headache is common in migraine. Normal CT and
Information neuro exam means there is no need for any further investigations.

87
CNS-System Wise 1700-by Sush and Team. 2016
Susmita, Asad, Manu, Saima, Zohaib, Savia, Shanu, Mona, Manisha, Sitara, Samreena, Sami and Komal

Loss of consciousness in patients with migraine is an alarming sign and should
prompt for imaging straight away. A normal CT and no focal neurological
deficit on examination reassure that there is no new pathology. The young age
of the patient gives us a clue as well. So there is no need of any intervention in
this scenario.

Dx = Migraine
Classification = Migraine with aura, Migraine without aura and Chronic
migraine
EPI = Common in women than men (vice versa in children)
First attack often in childhood with over 80% having their first attack
before the age of 30. If onset is at age over 50 other pathology should be
sought.
Family history is usually positive.
Severity decreases with advancing age.

Presentation : Typically the headaches last between 4 and 72 hours and have
at least two of the following features:
• Unilateral.
• Pulsating.
• Moderate or severe intensity of pain.
• Aggravated by, or resulting in the avoidance of, routine physical
activity.
In addition, there is at least one of:
• Nausea and vomiting during migraine attacks. These are common
symptoms that affect at least 60% of patients suffering from migraines.
• Photophobia and phonophobia, which are also very common.

Aura is highly variable in nature (visual,sensory etc) but tends to be consistent
for an individual. Headache starts maximum within one hour of an aura.

Investigations : Diagnosis is purely clinical. Investigations are only done to rule
out secondary causes of headache or when alarming symptoms like loss of
consciousness, seizure. memory loss etc present.

Treatment : Acute attack = Step one = Simple analgesics with or without anti
emetics
Step two = Anal analgesics and anal anti emetics
Step three = Triptans or ergotamine ( Patients
having moderate to severe migraine should be moved directly to step 3 ).
Prophylaxis = The NICE guidelines and CKS suggests the following indications
for prophylaxis :
• Frequent attacks are two or more attacks per month that produce
disability lasting for three days or more.
• Medication overuse is a risk when medication is used on more than two
days per week on a regular basis. Overuse needs to be addressed

88
CNS-System Wise 1700-by Sush and Team. 2016
Susmita, Asad, Manu, Saima, Zohaib, Savia, Shanu, Mona, Manisha, Sitara, Samreena, Sami and Komal

before further treatment can begin.
• Prophylaxis should be used when standard analgesia and triptans are
either contra-indicated or ineffective.
Beta blockers, amitriptyline, sodium valproate and Botulinum toxin A all can
be used as drugs for prophylaxis. Identifying and refraining from triggering
factor (if any) can also be used to avoid attacks such as stress, dietary factors
like cheese,chocolate,alcohol etc.


Reference
Dr Khalid/Rabia


Q: 263 A 60yo woman was found by her son. She was confused and had urinary
incontinence. She has recovered fully after 6h with no neurological complaints.
What is the most likely dx?
a. Stroke
b. Vestibular insufficiency (Vestibular A suppies cerebellum)
c. TIA
d. Intracranial hemorrhage


Clincher(s)
A
B
C
D
E
KEY Key = C (TIA)
Any patient coming with TIA will be treated like stroke.
Aspirin 300mg for 2 weeks and then 75mg + diperdomol
If patient is aspirin allergic.

Prophylaxis will be clopidogrel (See NICE or OCHM- 480)

For heart and stroke related: warfarin started as per scoring guidelines. Chad
score (OHCM- 477)

Warfarin takes a while to work as anticoagulant.

Additional Points in favour = Complete recovery in less than 24 hours
Information A transient ischaemic attack (TIA) is a temporary inadequacy of the circulation
in part of the brain (a cerebral or retinal deficit) that gives a clinical picture
similar to a stroke except that it is transient and reversible. Hence, TIA is a

89
CNS-System Wise 1700-by Sush and Team. 2016
Susmita, Asad, Manu, Saima, Zohaib, Savia, Shanu, Mona, Manisha, Sitara, Samreena, Sami and Komal

retrospective diagnosis. The duration is no more than 24 hours and a deficit
that lasts longer than 24 hours is defined as a stroke. The majority of TIAs last
for less than 30 minutes.

Management
Secondary prevention (see below) includes the use of antiplatelet therapy,
antihypertensive, and lipid-modifying treatments, the management of atrial
fibrillation if present and the management of any other underlying or risk
factors, including diabetes.

Assessment of the risk of stroke


An ABCD2 score of more than 4 suggests high risk of an early stroke.[5][6]

Scoring System for Risk of Stroke after TIA (ABCD2 Score)

Age Age >60 1

Blood pressure BP>140 systolic and/or >90 1


diastolic

Clinical features Unilateral weakness 2


Speech disturbance without 1
weakness


Other 0

Duration of >60 minutes 2


symptoms

90
CNS-System Wise 1700-by Sush and Team. 2016
Susmita, Asad, Manu, Saima, Zohaib, Savia, Shanu, Mona, Manisha, Sitara, Samreena, Sami and Komal


10-59 minutes 1


<10 minutes 0

Diabetes Presence of diabetes 1



Reference OHCM-
Dr Khalid/Rabia


Q: 299 A young girl presenting with fever, headache, vomiting, neck stiffness and
photophobia. She has no rashes. What is the most appropriate test to confirm
dx?
a. Blood culture
b. Blood glucose
c. LP
d. CXR
e. CT


Clincher(s) Fever with neck stiffness, young girl
A
B
C
D
E
KEY Key = C
Points in favour = This is suspected meningitis. Do Lp to confirm.

Additional
Information
Reference
Dr Khalid/Rabia



Q:348 A 56yo woman with MS presents with drooping of the left side of her lips. She
also has loss of sensation over her face, hearing impairment and some in-

91
CNS-System Wise 1700-by Sush and Team. 2016
Susmita, Asad, Manu, Saima, Zohaib, Savia, Shanu, Mona, Manisha, Sitara, Samreena, Sami and Komal

coordination of her movements. What is the most likely anatomical site
affected?
a. Cerebellum
b. Cerebrum
c. Spinal cord
d. Brain stem
e. Optic nerve

Clincher(s) Dropping of left side of lips,loss of sensation of face, hearing impairment,
incoordination of movements.
A Signs of nystagmus, ataxia or vertigo will be there. No typical DASHING
B
C Spinal cord lesion mainly presents as paraplegia, tetraplegia, UMN or LMN
lesion.
D Features of various cranial nerves together.
E Eye features not present
KEY D
Additional The cranial nerves III-XII emerge from the brainstem.[4] These cranial nerves
Information supply the face, head, and viscera. (The first two pairs of cranial nerves arise
from the cerebrum).

Diseases of the brainstem can result in abnormalities in the function of
cranial nerves that may lead to visual disturbances, pupil abnormalities,
changes in sensation, muscle weakness, hearing problems, vertigo,
swallowing and speech difficulty, voice change, and co-ordination problems.
Reference Brain stem. Features of 5, 7, 8
of brainstem lesion.


Dr Khalid/Rabia Dr khalid / Wikipedia


Q: 360 A 72 yo man has been on warfarin for 2yrs because of past TIA and stroke.
What is the most important complication that we should be careful with?
a. Headache
b. Osteoporosis
c. Ear infection
d. Limb ischemia (rare severe S/E)
e. Diarrhoea


Clincher(s) SE of warfarin in stroke patient.
A Headaches especially unusual ones, pt should seek immediate medical
attention.
Nhs.uk
B Not the SE

92
CNS-System Wise 1700-by Sush and Team. 2016
Susmita, Asad, Manu, Saima, Zohaib, Savia, Shanu, Mona, Manisha, Sitara, Samreena, Sami and Komal

C Not the SE
D Warfarin severe but rare side effect is skin necrosis/ tissue death. This
condition is caused by blood clots blocking the blood vessels, leading to
damaged areas of skin.
E common side effect
KEY A
Additional It is important to check for bleeding (haemorrhages) when taking warfarin as
Information anticoagulant medication can cause excessive bleeding.You must seek
medical attention and have an urgent blood test if you experience any of the
following:

passing blood in your urine or faeces, passing black faeces, severe bruising,
long nosebleeds (lasting longer than 10 minutes), bleeding gums, blood in
your vomit or coughing up blood, unusual headache, (in women) heavy or
increased bleeding during your period, or any other bleeding from the vagina

Zohaib: B: osteoporosis is the SE of warfarin (will be posted)



Reference Nhs.uk
Dr Khalid/Rabia The given key is E. Diarrhoea which is considered as a wrong key and A.
Headache is chosen as the correct key by previous plabbers.


Q:390 A 64yo man has presented to the ED with a stroke. CT shows no hemorrhage.
ECG shows atrial fib. He has been thrombolyzed and he is awaiting discharge.
What prophylactic regimen is best for him?
a. Warfarin
b. Heparin
c. Aspirin
d. Statins
e. Beta blockers

Clincher(s) Atrial fib in an elderly stroke pt.
A If Risk of emboli is high such as Past ischemic stroke, TIA or emboli then
warfarin is preferable. Here the pt has atrial fibrillation hence he's on high risk.
OHCM pg 125
B We give heparin in acute AF pts who are waiting for elective cardio version for
> 48 hrs. Or AF started <48 hrs but no high risk.
SourceOHCM pg 125
C Aspirin can be given if pt waiting for anticoagulant therapy
OHCM pg 476
D Statins are given for pt to for primary prevention of stroke.
E No use in this clinical picture
KEY A
Additional key is A. Warfarine. [Atrial fibrillation: post stroke- following a stroke or TIA
Information warfarin should be given as the anticoagulant of choice. NICE guideline].

93
CNS-System Wise 1700-by Sush and Team. 2016
Susmita, Asad, Manu, Saima, Zohaib, Savia, Shanu, Mona, Manisha, Sitara, Samreena, Sami and Komal

(heparin given in beginning in AF).
Reference
Dr Khalid/Rabia


Q:391 A 54yo man after a CVA presents with ataxia, intention tremors and slurred
speech. Which part of the brain has been affected by the stroke?
a. Inner ear
b. Brain stem
c. Diencephalon
d. Cerebrum
e. Cerebellum


Clincher(s) ataxia, intention tremors and slurred speech
A No connection with the qs
B Brain stem manifest normally with cranial nerves problems III - XII
C Diencephalon consists of structure such as thalamus and hypothalamus.
D The above features are not related to cerebrum
E Typical features described above
KEY E
Additional Features of cerebellum insult: DASHING
Information Ataxia ii) slurred speech or dysarthria iii) dysdiodokokinesis iv) intention
tremor v) nystagmus vi) hypotonia vii) gait
Reference
Dr Khalid/Rabia


Q:426 A 64yo man presents with a hx of left sided hemiparesis and slurred speech.
He was absolutely fine 6h after the episode. What is the most appropriate
prophylactic regimen?
a. Aspirin 300mg for 2 weeks followed by aspirin 75mg
b. Aspirin 300mg for 2 weeks followed by aspirin 75mg and dipyridamole
200mg
c. Clopidogrel 75mg
d. Dipyridamole 200mg
e. Aspirin 300mg for 2 weeks

Clincher(s) Symptoms resolved after 6 hrs ( TIA if resolved within 24 hrs)
A
B
C
D
E
KEY C

94
CNS-System Wise 1700-by Sush and Team. 2016
Susmita, Asad, Manu, Saima, Zohaib, Savia, Shanu, Mona, Manisha, Sitara, Samreena, Sami and Komal

Additional TIA :
Information
Inadequate circulation in part of the brain, gives a picture similar to stroke
but duration < 24 hours.

Common in old age. Men > women. ^ in black race.

Important risk factors- HTN, smoking, DM, Hyperlipidemia, Heart

disease. Management is by: Antiplatelets, anti HTN, lipid modifying agent,


AF

tx and any risk factors like DM.

Treatment: Aspirin + dypiridamole (dyp-if there is history of recurrent


strokes/TIA) (each as 300mg loading then 75mg

daily) + statin. [NICE guidelines]

TIA and Stroke: exclude haemorrhage by CT and then intially Aspirin 300mg
for 2 weeks.

For both stroke and TIA maintenance is clopigegral, if they can’t tolerate
clopi then Aspirin + dypiridamole

Antithrombotic therapy

clopidogrel is recommended first-line (as for patients who've had a stroke)

aspirin + dipyridamole should be given to patients who cannot tolerate


clopidogrel

these recommendations follow the 2012 Royal College of Physicians National


clinical guideline for stroke. Please see the link for more details (section 5.5)

these guidelines may change following the CHANCE study (NEJM


2013;369:11). This study looked at giving high-risk TIA patients aspirin +
clopidogrel for the first 90 days compared to aspirin alone. 11.7% of aspirin
only patients had a stroke over 90 days compared to 8.2% of dual
antiplatelet patients

95
CNS-System Wise 1700-by Sush and Team. 2016
Susmita, Asad, Manu, Saima, Zohaib, Savia, Shanu, Mona, Manisha, Sitara, Samreena, Sami and Komal

Patients with TIAs not being anticoagulated should be on modified-release
dipyridamole in combination with aspirin (modified-release dipyridamole
alone if aspirin is not tolerated).

Patients with ischaemic stroke (not due to AF) should be on clopidogrel (only
use modified-release dipyridamole in combination with aspirin if clopidogrel
is not tolerated). [14] Clopidogrel is also the preferred treatment option in
patients with peripheral arterial disease or multivascular disease.
Reference Patient info/ dr Rabia
Dr Khalid/Rabia

Q:537 A woman presented with blurred vision and intermittent clumsiness for 3m.
Reflexes are brisk in her arm and optic disc is pale. What is the single most
appropriate test to confirm dx?
a. CSF analysis
b. CT
c. MRI
d. EEG
e. EMG

Clincher(s) Blurred vision, intermittent clumsiness, brisk reflexes, pale optic disc (typical
of MS)
A It's for infection presence mainly
B To diagnose bone diseases, stages of tumor, injuries to internal organ, strokes.
C MRI is the choice.
D Mainly to investigate epilepsy.
E Electromyography (EMG) is a technique for evaluating and recording the
electrical activity produced by skeletal muscles
KEY C (multiple sclerosis, investigation of choice is gadolinium enhanced mri.)
Additional Multiple sclerosis: 3 times more common in women
Information
• most commonly diagnosed in people aged 20-40 years

Patient's with multiple sclerosis (MS) may present with non-specific


features, for example around 75% of patients have significant lethargy.

Visual :

• optic neuritis: common presenting feature


• optic atrophy
• Uhthoff’s phenomenon: worsening of vision following rise in body
temperature.
• internuclear ophthalmoplegia

Sensory:

96
CNS-System Wise 1700-by Sush and Team. 2016
Susmita, Asad, Manu, Saima, Zohaib, Savia, Shanu, Mona, Manisha, Sitara, Samreena, Sami and Komal

• pins/needles
• numbness
• trigeminal neuralgia
• Lhermitte's syndrome: paraesthesiae in limbs on neck flexion

Motor:

• spastic weakness: most commonly seen in the legs

Cerebellar:

• ataxia: more often seen during an acute relapse than as a presenting


symptom
• tremor

Others :

• urinary incontinence
• sexual dysfunction
• intellectual deterioration


Reference Dr Rabia
Dr Khalid/Rabia

Q:538 A 63yo man presents after having a seizure. Exam: alert, orientated,
inattention on the left side and hyperreflexia of the arm. What is the most
probable dx?
a. Cerebral tumor
b. Pituitary adenoma
c. Cerebellar abscess
d. Huntingtons chorea
e. Parkinsonism

Clincher(s) Hyper reflexia, Inattention on the left side, seizure episode
A Inattention is a feature of parietal lobe
B Will give bitemporal loss
C Cerebellum signs are absent
D Chorea is repeated movements
E No hypokinesia,tremor
KEY A
Additional • Inattention or neglect is a feature of parietal lobe lesion. If lesion is on

97
CNS-System Wise 1700-by Sush and Team. 2016
Susmita, Asad, Manu, Saima, Zohaib, Savia, Shanu, Mona, Manisha, Sitara, Samreena, Sami and Komal

Information right side there will be left sided inattention. That is patient is
unaware of his left side and he when shaves do it only to right half of
face, during eating eats only from the right half of plate and can not
drive as he only aware of his right side and totally unaware of left side
of the road
• pituitary will give bitemporal vision loss
• cerebellar signs are nystagmus ataxia etc.
• Chorea is repeated movements.. its focal ant lobe lesion.

Seizure n hyperflexia (exaggerated reflexes) are indicators of space


occupied lesion (SOL) Because the pt has got upper motor neuron signs.

• Its not pituitary because no signs of optic n compression and its not
bilateral.

Its not cerebellar dis other wise he should have ataxia, pass pointing
rombergism.

Its not chorea cuz no symptoms of chorea.

Not parkinson because no hypokinesia, tremor

seizure causing neuro deficit in elderly... first D/D should b cerebral tumour
unless specified otherwise..


Reference Dr Rabia
Dr Khalid/Rabia


Q:541 A 50yo lady with weak limbs when examined was found to have burn marks on
finger tips, wasted and weak hands with diminished reflexes. She also has
weak spastic legs and dissociated sensory loss. What is the dx? (Upper motor
neuron and lower motor signs)
a. MS
b. Syringomyelia
c. MND
d. Guillian-barre
e. Freidriech’s ataxia


Clincher(s) Burn marks on finger tips, wasted hands and weak hands,diminished
reflexes, spastic legs, dissociated sensory loss
A MS cause UMN with brisk reflexes not cause dissociated sensory loss
B Causes of dissociate sensory loss causes
• Syringomyelia

98
CNS-System Wise 1700-by Sush and Team. 2016
Susmita, Asad, Manu, Saima, Zohaib, Savia, Shanu, Mona, Manisha, Sitara, Samreena, Sami and Komal

• Brown sequerd syndrome
C Motor only
D Guillain-Barre syndrome (polyneuropathy) is an acute, inflammatory,
postinfectious polyneuropathy. (there will be always a history)
prodromal malaise with vomiting, headache, fever and limb pains is rapidly
surmounted by a progressive and ascending paralysis. This can lead to
respiratory dysfunction, and as such, the acute presentation can be a
neurological emergency.
Source: gpnotebook

E Friedreich's ataxia is a progressive neurodegenerative disorder, typically with
onset before 20 years of age.
Signs and symptoms include progressive ataxia, ascending weakness and
ascending loss of vibration and joint position senses, pes cavus, scoliosis,
cardiomyopathy, and cardiac arrhythmia
Source patient info
KEY B
Additional Motor + sensory loss = syringomyelia
Information
Motor + tongue= syringobulbia

Motor only= MND




Syringomyelia: Most present in the 20s and 30s but can present in childhood
or later life.
Damage to the spinal cord often leads to progressive weakness in the arms
and legs, stiffness in the back, shoulders, arms, or legs, and chronic, severe
pain. These can be present at present at presentation
Other symptoms may include headaches, a loss of the ability to feel extremes
of hot or cold (especially in the hands), and loss of bladder and other
functions.
Reference
Dr Khalid/Rabia Dr Rabia


Q:545 A 73yo male presents with a 12m hx of falls. His relatives have also noticed
rather strange behavior of late and more recently he has had episodes of
enuresis. Exam: disorientation to time and place, broad-based, clumsy gait.
What is the most probable dx?
a. Dementia
b. Pituitary adenoma
c. CVD
d. Syringomyelia

99
CNS-System Wise 1700-by Sush and Team. 2016
Susmita, Asad, Manu, Saima, Zohaib, Savia, Shanu, Mona, Manisha, Sitara, Samreena, Sami and Komal

e. Normal pressure hydrocephalu

Clincher(s) Gait, dementia and enuresis
A It's a more broader term here which doesn't fit with scenario.
B Normally present with visual field defects particularly bitemporal hemianopsoa
which is a type of partial blindness where vision is missing in both the outer
half of left and right usual field. This occur due to tumor compression on optic
nerve.
These tumors compress the specific visual pathway at optic chiasma
C Not the case here
D Motor plus sensory is syringomyelia.
E Wet, wacky and wobbly grandpa ( typical presentation) ***
KEY E
Additional Normal pressure hydrocephalus : is a reversible cause of dementia. Seen in
Information elderly patients. Secondary to reduced CSF absorption at the arachnoid villi.

Classical triad of features:-

* Urinary incontinence

* Dementia

* bradyphenia is a neurological term referring to the slowness of thought


common to many disorders of the brain

* Gait abnormality

Imaging: Hydrocephalus with an enlarged 4th ventricle

Management: ventriculao peritoneal shunting

Different types of dementia differentiation:

• one in which person talks vulgar things---> frontotemporal dementia


• One in which dementia fluctuates---> lewy body dementia
• One asso with parslysis---> vascular dementia
• One with stepwise deterioration---> Alzheimer's disease


Reference
Dr Khalid/Rabia Dr Rabia / Wikipedia


Q:546 A 75yo nursing home resident complains of headache, confusion and impaired
vision for 4days. She has multiple bruises on her head. What is the most likely
cause of confusion in this pt/

100
CNS-System Wise 1700-by Sush and Team. 2016
Susmita, Asad, Manu, Saima, Zohaib, Savia, Shanu, Mona, Manisha, Sitara, Samreena, Sami and Komal

a. Alcohol intoxication
b. Infection
c. Subdural hematoma
d. Hypoglycemia
e. Hyponatremia


Clincher(s) Headache, confusion, impaired vision,multiple bruises on her head
A It's a risk factor
B No hx or symptoms of infection
C Fits perfectly with this scenario
D Hypoglycaemia again causes confusion and vision trouble and can be a risk
factor here but not the cause
Ohcm 206
E Hyponatremia increases the risks of falls in elderly so again can be a risk
factor.
OHCM page 686
KEY C
Additional Subdural Hepatoma:
Information
Bleeding into the outermost meningeal layer. Most commonly occur around
the frontal and parietal lobes.

Risk factors include old age, alcoholism and anticoagulation.

Slower onset of symptoms than a epidural haematoma.


Reference
Dr Khalid/Rabia Dr Rabia/ OHCM



Q: 553 A 69yo woman has had a stroke. Her left upper and lower limbs are paralyzed
and she is having difficulty in speaking. Which anatomical site is most likely
affected?
a. Hippocampus
b. Cerebellum
c. Internal capsule (purely motor stroke presentations)
d. Thalamus
e. Brain stem




Clincher(s)

101
CNS-System Wise 1700-by Sush and Team. 2016
Susmita, Asad, Manu, Saima, Zohaib, Savia, Shanu, Mona, Manisha, Sitara, Samreena, Sami and Komal

A Hippocampus (sea-hoarse in Greek) belongs to the limbic system and plays
important roles in the consolidation of information from short-term
memory to long-term memory and spatial navigation. The hippocampus is
located under the cerebral cortex
B Cerebellum responsible for coordination: DASHING
C (typical picture – all motor loss)
D Thalamus situated between the cerebral cortex and the midbrain. Some of its
functions are the relaying of sensory and motor signals to the cerebral cortex,
and the regulation of consciousness, sleep, and alertness, emotion. The two
parts of the thalamus surround the third ventricle. It is the main product of
the embryonic diencephalon.

E Brain stem: (midbrain, pons, medulla) connect the body to the brain, it keeps
you alive
KEY
Additional c. Internal capsule
Information Internal capsule. Its lacunar infarct. Internal capsule has both corticospinal and
corticobulbar (bulbur means-brain stem) fibers. Infarct results in hemiparesis/
hemiplegia with dysphagia/ dysarthria by exclusion method for me:
A would've caused memory deficit.
B some coordination signs.
D sensory loss ipsilat.
E at least 2 cranial nerves involved
Wiki Internal capsule is supplied by branches from the MCA, ACA and the ant.
Choroidal art (the latter is a branch from the internal carotid art.)


corticobulbar (=motor pathway connecting the cerebral cortex to
the brainstem primarily involved in carrying the motor function of the non-
oculomotor cranial nerves.)

102
CNS-System Wise 1700-by Sush and Team. 2016
Susmita, Asad, Manu, Saima, Zohaib, Savia, Shanu, Mona, Manisha, Sitara, Samreena, Sami and Komal


Reference Wikipedia, Rabia
Dr Khalid/Rabia


Q: 554 554. A 72yo man brought to the ED with onset of paraplegia following a trivial
fall. He was treated for prostatic malignancy in the past. What is the single
most probable dx?
a. Paget’s disease
b. Osteoporotic fx of vertebre
c. Secondary
d. Multiple myeloma
e. Spondylosis

72 y, prostate Ca hx, trivial fall
Clincher(s)
A Paget’s disease most common complaints are bone pain and/or deformity.
pathological fractures elevated serum alkaline phosphatase or characteristic
abnormality on X-ray. Secondary metastatis of bone
B Osteoporosis is more common in old ladies
C Secondary most probable here
D Multiple myeloma can present with a wide variety of symptoms including
hypercalcaemia, anaemia, renal impairment and bone pain.

E Spondylosis inflammatory back pain and enthesitis (inflammation at the site of
bone insertion of ligaments and tendons) or arthritis with radiological findings
KEY c. Secondary

Additional
Information c. Secondary
Bones weakened from metastatic cancer may break (fracture). The fracture
can happen with a fall or injury, but a weak bone can also break during
everyday activities. These fractures often cause sudden, severe pain. The pain
may keep you from moving much at all. In some cases, a fracture is the first
sign of bone metastasis.
The most common sites of fractures are the long bones of the arms and legs

103
CNS-System Wise 1700-by Sush and Team. 2016
Susmita, Asad, Manu, Saima, Zohaib, Savia, Shanu, Mona, Manisha, Sitara, Samreena, Sami and Komal

and the bones of the spine. Sudden pain in the middle of the back, for
example, is a common symptom of a bone in the spine breaking and collapsing
from cancer.
c.a prostate led to vertebral mets(most common bone secondary in c.a
prostate),that lead to fracture on trivial injury and paraplegia due to spinal
cord compression


.

.

Reference
Dr Khalid/Rabia Bone secondary

• Brostate cancer
• Breast cancer
• Bronchia cancer
• Bidney cancer
• Bhyroid cancer



Q: 571. A 74yo female presents with headache and neck stiffness to the ED.
Following a LP the pt was started on IV ceftriaxone. CSF culture = listeria
monocytogenes. What is the appropriate tx?
a. Add IV amoxicillin
b. Change to IV amoxicillin + gentamicin
c. Add IV ciprofloxacin
d. Add IV co-amoxiclav
e. Continue IV ceftriaxone as mono-therapy


Clincher(s) Listeria
A
B
C
D
E
KEY B (memorise below)

104
CNS-System Wise 1700-by Sush and Team. 2016
Susmita, Asad, Manu, Saima, Zohaib, Savia, Shanu, Mona, Manisha, Sitara, Samreena, Sami and Komal

Additional
Information


Reference passmedicine
Dr Khalid/Rabia


Q: 573 A 14yo boy fell and hit his head in the playground school. He didn’t lose
consciousness. He has swelling and tenderness of the right cheek with a
subconjuctival hemorrhage on his right eye.
What is the most appropriate initial inv?
a. CT brain
b. EEG
c. MRI
d. Skull XR
e. Facial XR



Clincher(s)
A
B
C
D
E
KEY e. Facial XR

Additional there's no indication of CT scan here- he is conscious and has not vomited
Information
The difference between Skull and facial x ray is view. Skull PA view is done in
prone position for seeing Skull bones. Facial is simply reverse i.e AP view done
in supine position and gives more clear view of facial bones. Suspected injury
to facial bones is a CONTRAINDICATION for PA view as patient cannot be asked
to lie down in prone position/ or to lean forward with face down.

105
CNS-System Wise 1700-by Sush and Team. 2016
Susmita, Asad, Manu, Saima, Zohaib, Savia, Shanu, Mona, Manisha, Sitara, Samreena, Sami and Komal

A facial or sinus X-ray may be done to:
• Find problems of the sinuses of the face and nose, such as sinusitisor
abnormal growths (polyps or tumors).
• Find fractures of the facial bones and nose.
• Check the bones around the eye (orbital cavity).
• Check the sinuses before surgery.
• Check for metal objects around the eyes before a magnetic resonance
imaging (MRI) test.
• Look for the cause of pain in the face.

Reference
Dr Khalid/Rabia


Q:583 583. A 30yo man is suffering from fever, rash and photophobia. Doctors are
suspecting he is suffering from meningitis. Which is the best medication for
this condition?
a. Ampicilling
b. Cefotaxime
c. Tetracycline
d. Acyclovir
e. Dexamethasone

Clincher(s)
A
B
C
D
E
KEY B initial empirical therapy (from 3m to 50 years- only cefotaxime; younger
and older: add amoxicillin)
Additional Q 571 above
Information
Reference
Dr Khalid/Rabia

Q: 585 585. A pt was admitted to the ED after a head injury. When examined on
arrival his GCS (glaxo comma scale)=15 and then at night his GCS deteriorated
to 12. What investigation should be done?
a. CT head
b. XR skull
c. IV mannitol
d. Drill a burr hole
e. Shift to OR

106
CNS-System Wise 1700-by Sush and Team. 2016
Susmita, Asad, Manu, Saima, Zohaib, Savia, Shanu, Mona, Manisha, Sitara, Samreena, Sami and Komal

Clincher(s)
A
B
C
D
E
KEY A
Additional Head injury: NICE guidance
Information

NICE has strict and clear guidance regarding which adult patients are safe to
discharge and which need further CT head imaging. The former group are also
divided into two further cohorts, those who require an immediate CT head and
those requiring CT head within 8 hours of injury:

CT head immediately

• GCS < 13 on initial assessment


• GCS < 15 at 2 hours post-injury
• suspected open or depressed skull fracture.
• any sign of basal skull fracture (haemotympanum, 'panda' eyes,
cerebrospinal fluid leakage from the ear or nose, Battle's sign- buise
on mastoid bone).
• post-traumatic seizure.
• focal neurological deficit.
• more than 1 episode of vomiting


CT head scan within 8 hours of the head injury - for adults with any of the
following risk factors who have experienced some loss of consciousness or
amnesia since the injury:

• age 65 years or older


• any history of bleeding or clotting disorders
• dangerous mechanism of injury (a pedestrian or cyclist struck by a
motor vehicle, an occupant ejected from a motor vehicle or a fall from
a height of greater than 1 metre or 5 stairs)
• more than 30 minutes' retrograde amnesia of events immediately
before the head injury


If a patient is on warfarin who have sustained a head injury with no other
indications for a CT head scan, perform a CT head scan within 8 hours of the
injury.

107
CNS-System Wise 1700-by Sush and Team. 2016
Susmita, Asad, Manu, Saima, Zohaib, Savia, Shanu, Mona, Manisha, Sitara, Samreena, Sami and Komal

Reference NICE
Dr Khalid/Rabia GCS: oxford (sami will put in drop box)

Q: 589. A 42yo man has been tired and sleepy for the last few weeks in the
morning. His work has started getting affected as he feels sleepy in the
meetings. His BMI=36. What is the single most likely dx?
a. Idiopathic hypersomnia
b. Narcolepsy
c. Chest hyperventilation syndrome
d. OSAS Obstructive sleep apnea syndrome
e. REM-related sleep disorder


BMI 36, male, 42 yo
Clincher(s)
A Idiopathic hypersomnia is a disease, thought to be a neurological disorder,
which is characterized primarily by severe excessive daytime sleepiness (EDS).
It has historically been rarely diagnosed and is often very difficult to diagnose
at an early stage; it is usually a lifelong chronic disease, which is often
debilitating. There is a very low level of public awareness of idiopathic
hypersomnia, which often leads to stigma for those who suffer from it.
Currently, there is no cure, there are no FDA-approved treatments, and
research funding for its study is scarce.

B Narcolepsy tetrad of classic symptoms: excessive daytime sleepiness (EDS),
cataplexy, hypnagogic hallucinations and sleep paralysis.

C is a condition in which minute ventilation exceeds metabolic demands,
resulting in hemodynamic and chemical changes that produce characteristic
dysphoric symptoms. Inducing a decrease in arterial partial pressure of carbon
dioxide (PaCO2) through voluntary hyperventilation reproduces these
symptoms.
D
E In a person with REM sleep behavior disorder (RBD), the paralysis that
normally occurs during REM sleep is incomplete or absent, allowing the person
to "act out" his or her dreams. RBD is characterized by the acting out of
dreams that are vivid, intense, and violent. Dream-enacting behaviors include
talking, yelling, punching, kicking, sitting, jumping from bed, arm flailing, and
grabbing. An acute form may occur during withdrawal from alcohol or
sedative-hypnotic drugs.
KEY d. OSAS
Additional
Information Risk factors include:
• Obesity (strongest risk factor).
• Male gender.
• Middle age (55-59 in men, 60-64 in women).

108
CNS-System Wise 1700-by Sush and Team. 2016
Susmita, Asad, Manu, Saima, Zohaib, Savia, Shanu, Mona, Manisha, Sitara, Samreena, Sami and Komal

• Smoking.
• Sedative drugs.
• Excess alcohol consumption.

All patients with OSA causing excessive daytime or awake time sleepiness need
to cease driving until satisfactory control of symptoms has been attained.


Reference
Dr Khalid/Rabia


Q: 600 600. Which artery runs in the anterior inter-ventricular groove?
a. Acute marginal branch
b. Left ant descending artery
c. Septal branches
d. Circumflex artery
e. Right coronary artery



Clincher(s)
A
B

109
CNS-System Wise 1700-by Sush and Team. 2016
Susmita, Asad, Manu, Saima, Zohaib, Savia, Shanu, Mona, Manisha, Sitara, Samreena, Sami and Komal

C
D
E
KEY b. Left ant descending artery

Additional
Information


Reference
Dr Khalid/Rabia


Q: 609 Which of the following reflexes and innervating spinal nerves are correctly
paired?
a. Anal reflex – S1
b. Ankle jerk – L5
c. Biceps jerk – C7 & C8
d. Knee jerk – L3 & L4
e. Triceps jerk – T1



Clincher(s)
A
B
C
D
E
KEY d. Knee jerk – L3 & L4

Additional bicep: C5-C6

110
CNS-System Wise 1700-by Sush and Team. 2016
Susmita, Asad, Manu, Saima, Zohaib, Savia, Shanu, Mona, Manisha, Sitara, Samreena, Sami and Komal

Information Knee jerk – L3 & L4
Tricep: C7-C8
Ankle: S1, S2

Reference Passmedicine
Dr Khalid/Rabia


Q: 611 611. A 68yo lady complains of falls to the ground without any warning,
maintains consciousness and no confusion. She says this has occurred at
number of times. What is the dx?
a. Stokes Adams attack
b. Hypoglycemia
c. Vasovagal syncope
d. Drop attacks
e. Epilepsy


Clincher(s)
A There is transient arrhythmia. It is transient complete heart block which leads
to hypoperfusion of
the brain cause confusion or TIA.
• Any elderly patient with repeated collapse or syncope without warning is
always Stoke Adams until
proven otherwise
• Patient usually go pale before falling
Investigation: 24 hour ECG (ambulatory ECG)
Treatment: may need permanent pace maker
B Sweating, hx of DM
C • vasovagal: triggered by emotion, pain or stress. Often referred to as
'fainting', young girls (situational stress)
• Rx reassure

D Drop attacks Sudden weakness of the legs causes the patient, usually an older
woman, to fall to the ground. There is no warning, no LOC and no confusion
afterwards. The condition is benign, resolving spontaneously after a number of
attacks.
E
KEY d. Drop attacks

Additional
Information d. Drop attacks
Sudden falling to ones knees without LOC and without warning. Recovery is
immediate
brief LOC in SAA without warning,while vv syncope- triggers with loc
because she maintains consciousness. Stokes Adams and Vaso-vagal--> brief

111
CNS-System Wise 1700-by Sush and Team. 2016
Susmita, Asad, Manu, Saima, Zohaib, Savia, Shanu, Mona, Manisha, Sitara, Samreena, Sami and Komal

loss of consciousness. Stokes Adams patient gets pale before attack. Vaso-
vagal mostly in young girls
age is not for epilepsy , other than a,b,c preceding particular s/s with brief loc
in a&b ,
Adams is associated Av heart block bradyarrhythmia,hypoglycemia has hx of
dm or sweating tachycardia,vasovagal is suddenly change in position and
epilepsy is hx and initial convulsive and postictal phase


Reference Passmed, OHCM p. 464
Dr Khalid/Rabia (Saima)

Stokes: arythmia/palpitation (can happen in any position), hx of loss of consc

Vaso vagal: when standing up, always a prodromal symptom (e.g.
nauseous/sweaty). NO loss of consciousness




Q: 612 A 50yo man complains of being pursued by the police for a crime he denies
committing. He has poor concentration and impaired short-term memory. He
admits to drinking large amounts of alcohol for the last 20yrs. What is the most
probable dx?
a. Dementia
b. Hallucination
c. Wernicke’s encephalopathy
d. Schizophrenia
e. Korsakoff psychosis

Clincher(s) Story, poor concentration, impaired short-term memory, alcohol hx
A Dementia - progressive and largely irreversible clinical syndrome that is
characterized by global deterioration in intellectual function, behaviour and
personality in the presence of normal consciousness and perception
B Hallucinations are auditory, visual gustatory, or tactile sensations occurring
without any stimulus
C Wernicke's encephalopathy: a classic triad of symptoms (mental confusion,
ataxia and ophthalmoplegia). Pt. will also have flapping tremor and
headache.
D Schizophrenia is a long-term mental health condition that causes a range of
different psychological symptoms, including:
• hallucinations – hearing or seeing things that do not exist
• delusions – unusual beliefs not based on reality that often contradict
the evidence
• muddled thoughts based on hallucinations or delusions
• changes in behaviour

112
CNS-System Wise 1700-by Sush and Team. 2016
Susmita, Asad, Manu, Saima, Zohaib, Savia, Shanu, Mona, Manisha, Sitara, Samreena, Sami and Komal

E
KEY: E Korsakoff psychosis
• Combined presence of Wernicke’s encephalopathy + Korsakoff’s
syndrome (loosing memory/ confabulation; long period of drinking)
• (GPNotebook) Korsakoff's psychosis is a condition that is characterised
by a retrograde amnesia - loss of memory for events before the onset
of the illness - and an inability to memorise new information.
• The condition is most commonly seen in alcoholics where there is loss
of nerve cells in the thalamic and mamillary bodies secondary to a
thiamine deficiency, and occasionally in cases of head injury, cerebral
hypoxia, tumour or encephalitis. There is microglial and capillary
proliferation.


Additional
Information
Reference
Dr Khalid/Rabia e. Korsakoff psychosis
Confabulation of memory in clear consciousness - very characteristic of the
syndrome.
Anterograde amnesia is the main feature of the syndrome.
Retrograde amnesiaTelescoping of events is characteristic - eg, the patient
says something happened recently when it took place many years ago.

Wernicke-Korsakoff syndrome (WKS) is a spectrum of disease resulting from
thiamine deficiency, usually related to alcohol abuse.

Korsakoff's syndrome: late manifestation of the condition, where Wernicke's
encephalopathy has not been adequately treated

• Serum thiamine levels (vitamin B1) levels may be low.


Q: 624 A man sat cross-legged for about 30mins. After this he was unable to dorsiflex
his left foot and had loss of sensation in the web space between the big toe
and the 2nd toe. He also has sensory loss on the same side of the foot after 2h.
Which of the following was affected?
a. Femoral nerve
b. Sural nerve
c. Peroneal nerve
d. Sciatic nerve

cross-legged, unable to dorsiflex his left foot, loss of sensation in the web

113
CNS-System Wise 1700-by Sush and Team. 2016
Susmita, Asad, Manu, Saima, Zohaib, Savia, Shanu, Mona, Manisha, Sitara, Samreena, Sami and Komal

space between the big toe and the 2nd toe.
Clincher(s)
A


B The sural nerve is a sensory nerve in the calf region (sura) of the leg. It is made
up of collateral branches of the tibial nerve and common fibular nerve
C
D



KEY: C Peroneal nerve
Chronic peroneal neuropathy can result from, among other conditions, bed
rest of long duration, hyperflexion of the knee, peripheral neuropathy,
pressure in obstetric stirrups, and conditioning in ballet dancers.

The most common cause is habitual leg crossing that compresses the
common peroneal nerve as it crosses around the head of the fibula. Transient
trauma to the nerve can result from peroneal strike. Damage to this nerve
typically results in foot drop, where dorsiflexion of the foot is compromised
and the foot drags (the toe points) during walking; and in sensory loss to the
dorsal surface of the foot and portions of the anterior, lower-lateral leg.

114
CNS-System Wise 1700-by Sush and Team. 2016
Susmita, Asad, Manu, Saima, Zohaib, Savia, Shanu, Mona, Manisha, Sitara, Samreena, Sami and Komal


Additional If thre is option of deep and sup peritoneal nerve: In deep peroneal nerve:
Information refer.
Reference OHCM, GPNotebook
Dr Khalid/Rabia


Q: 634 A 32yo female with axillary freckles and café au lait spots wants to know the
chances of her child also having similar condition.
a. 1:2
b. 1:4
c. No genetic link
d. 1:16
e. Depends on the genetic makeup of the partner

Clincher(s)
A
B
C

115
CNS-System Wise 1700-by Sush and Team. 2016
Susmita, Asad, Manu, Saima, Zohaib, Savia, Shanu, Mona, Manisha, Sitara, Samreena, Sami and Komal

D
E
KEY: E Depends on genetic makeup of the partner.
[Depends on the make up of the partner, ideally it's 1:2 since it's autosomal
dominant with complete penetrance but if the patients spouse also has
neurofibromatosis, it raises it to at least 75% with a possible 100%].

Cafe au lait spots are brown cutaneous macules which may be present at birth
and, if numerous, are strongly suggestive of the diagnosis of
neurofibromatosis (AD) or Albright's syndrome. The incidence of single
lesions is high and of no significance - a patient is allowed up to four patches of
less than 0.5cm diameter, but no patches greater than 1.5cm diameter.

Neurofibromatosis is a term used to describe two different diseases with
some overlapping features.
The types of neurofibromatosis are:
• von Recklinhausen's disease, peripheral or type I neurofibromatosis
• bilateral acoustic neurofibromatosis, central or type II
neurofibromatosis

116
CNS-System Wise 1700-by Sush and Team. 2016
Susmita, Asad, Manu, Saima, Zohaib, Savia, Shanu, Mona, Manisha, Sitara, Samreena, Sami and Komal

Additional
Information


Reference GPNotebook
Dr Rabia a. 1:2 (or E)?
neurofibromatosis..autosomal dominant..so 1:2
if her partner has also same condition than there 75 %chances while he is
normal than 50 % chances we cant tell her for sure that there is 50 % of getting
her baby affected when he husband is diseased. So i think option E is correct
answer in such scenario

117
CNS-System Wise 1700-by Sush and Team. 2016
Susmita, Asad, Manu, Saima, Zohaib, Savia, Shanu, Mona, Manisha, Sitara, Samreena, Sami and Komal

Q: 637 A 52yo woman speaks rapidly without any pause and ignores interruptions.
She doesn’t even pause to take enough breaths. What term best describes this
kind of speech?
a. Flight of ideas
b. Broca’s aphasia
c. Wernicke’s aphasia
d. Pressure of speech
e. Verbal dysphasia

Clincher(s) speaks rapidly without any pause, ignores interruptions
A Flight of ideas a nearly continuous flow of rapid speech that jumps from topic
to topic
B Broca’s aphasia- have trouble speaking fluently but their comprehension can
be relatively preserved. This type of aphasia is also known as non-fluent or
expressive aphasia.
C Wernicke’s aphasia - in which an individual is unable to understand language in
its written or spoken form. Even though they can speak using grammar, syntax,
rate, and intonation, they typically have difficulty expressing themselves
meaningfully through speech
D
E Verbal dysphasia – disorder of speech
KEY: D Pressure of Speech
Pressure of speech is a tendency to speak rapidly and frenziedly, as if
motivated by an urgency not apparent to the listener. The speech produced,
sometimes called pressured speech, is difficult to interrupt. It may be too fast,
or too tangential for the listener to understand. It is an example of cluttered
speech.
Additional
Information
Reference Dr. Rabia
Dr Khalid/Rabia


Q: 768 A middle aged man complains of a node which has been growing on his nose
for several months. Now it’s firm with central depression. It is 0.6cm in size.
What is the single most likely dx?
a. Basal cell carcinoma
b. Squamous cell carcinoma
c. LN
d. Melanoma
e. Kaposi’s sarcoma
f. Keratoacanthoma
g. Molluscum contagiosum

Clincher(s) On the nose, with central depression of the node
A (ANYTHING ABOVE NECK: basal call ca is most common). Basal (occur more in

118
CNS-System Wise 1700-by Sush and Team. 2016
Susmita, Asad, Manu, Saima, Zohaib, Savia, Shanu, Mona, Manisha, Sitara, Samreena, Sami and Komal

midline of face or auricular area and slow growing. Squamous bleeds on
touch, fast growing, both occur in sun exposed area. Basal: Sq: 4:1
B squamous cell carcinoma of the skin causes: associated with excessive sunlight
exposure, exposure to chemical carcinogens such as coal tar products, chronic
irritation, and in patients with immunosuppression.
There is an increased frequency in persons with non- pigmented skin. It is less
common than basal cell carcinoma (BCC), with an incidence ratio of BCC:SCC
of 4:1.


C
D most malignant melanomas are greater than 10mm in diameter
most benign tumours are less than 6mm
cytology: with increasing malignancy, there is a wider variation in cell shape
and amount of pigment contained withing the melanocyte
E The lesion or lesions of Kaposi's sarcoma may at first appear like bruises,
being coloured purple and blue. However, these become slightly raised and
firmer, with the colour darkening so that the lesions become purple and black.
They are not usually painful or itchy though they may develop at multiple sites
over the body and cause cosmetic distress. Common in HIV pts.
F Keratoacanthoma is a rapidly growing epidermal tumour that resembles
squamous cell carcinoma both clinically and histologically.

119
CNS-System Wise 1700-by Sush and Team. 2016
Susmita, Asad, Manu, Saima, Zohaib, Savia, Shanu, Mona, Manisha, Sitara, Samreena, Sami and Komal

KEY: A Basal Cell carcinoma
presents with pearly nodule with rolled telangiectatic edge on the face. Lesions
on trunk can appear as red scaly plaques with raised smooth edge.
Causes: uv exposure
Treatment: (OHCM) Excision; radiotherapy if a big lesion, eg in the elderly.
Cryo ± curettage can be used if non-critical site;
consider imiquimod if superficial



Additional
Information
Reference GPNotebook, OHCM pg:590
Dr Khalid Ans. The key is A. Basal cell carcinoma. [A pearly white umbilicated ulcer (with
central depression) any where in the face with rolled edges / inverted edge is
basal cell carcinoma. Any ulcer which is located above the neck is always
basal cell carcinoma until proven otherwise. Derma Samson note].


Q: 768** A pt with SNHL (Sensorineural hearing loss) and loss of corneal reflex on the
left side. What is the most definitive inv?
a. CT of internal auditory meatus
b. Nuclear imaging of brain
c. MRI of internal auditory meatus (if CN 5,6,7- MRI brain )
d. Radio isotope scan
e. XR skull

Ans. The key is C.

Clincher(s) SNHL, loss of corneal reflex
A

120
CNS-System Wise 1700-by Sush and Team. 2016
Susmita, Asad, Manu, Saima, Zohaib, Savia, Shanu, Mona, Manisha, Sitara, Samreena, Sami and Komal

B
C
D
E
KEY: C MRI of internal auditory meatus.
SNHL – always suspect acoustic neuroma
An acoustic neuroma is a benign, slow growing neoplasm of the Schwann cells
of the eighth cranial nerve. lesions are usually located in the internal auditory
canal or the cerebellopontine angle causing compression of the vestibular
nerve and resulting eventually in deafness. acoustic neuromas, particularly
bilateral cases, may occur in patients with type 2 neurofibromatosis


Additional 5, 7, 8 cranial nerve involvement
Information
Reference OHCM, GPNotebook
Dr Khalid The likely cause is acoustic neuroma for which appropriate investigation is MRI
brain (mainly posterior fossa) as the tumour arise from inner end of internal

121
CNS-System Wise 1700-by Sush and Team. 2016
Susmita, Asad, Manu, Saima, Zohaib, Savia, Shanu, Mona, Manisha, Sitara, Samreena, Sami and Komal

acoustic meatus but grows mainly in posterior cranial fossa. As it is not given in
option so from the given options MRI of internal acoustic meatus is the best
option.



Q: 770 A 65yo man with cancer of middle 1/3 of the esophagus presents with
dysphagia. What is the most immediate management?
a. Chemotherapy
b. Radiotherapy
c. Stenting
d. Gastrostomy
e. TPN

Clincher(s) Dysphagia, Most immediate management
A
B
C
D A gastrostomy is the formation of an artificial stoma between the stomach and
the abdominal wall. It is indicated for nutritional purposes following
gastrointestinal surgery, coma, and after some neurological conditions
E



KEY: C Stenting

122
CNS-System Wise 1700-by Sush and Team. 2016
Susmita, Asad, Manu, Saima, Zohaib, Savia, Shanu, Mona, Manisha, Sitara, Samreena, Sami and Komal


Additional
Information


Reference OHCM, GPNotebook
Dr Khalid/Rabia Squamous cell carcinoma of the esophagus. Presenting with dysphagia.
Treatment: for early esophageal cancer, endoscopic resection or submucosal
dissection
Endoscopic Esophagectomy TOC for most advanced
Stenting is the first line approach to assist swallowing.
Radio chemo may be of used primarily in reducing bulk of tumour,
Nutritional status: use liguid feeds or PEG tubes.

Q: 797 A 44yo lady who has PCKD (polycystic kidney disease) is concerned because
her 38yo brother has just died of an intracranial insult. She knows he was not
hypertensive. What was the most likely cause of her brother’s death?
a. Subdural hematoma
b. Subarachnoid hemorrhage
c. Cerebral infarct
d. Epidural hematoma

123
CNS-System Wise 1700-by Sush and Team. 2016
Susmita, Asad, Manu, Saima, Zohaib, Savia, Shanu, Mona, Manisha, Sitara, Samreena, Sami and Komal

Clincher(s) PCKD hx, brother had intracranial insult and was not HTN
A Acute subdural haemorrhage is usually associated with severe brain injury
following trauma. It can occur at any age.
Chronic subdural haemorrhage may be traumatic or may arise spontaneously.
It is most common in infants and the elderly
B Pt of PCKD have more SAH as these patients re highly prone to berry aneurism
C
D An epidural haemorrhage results from rupture of one of the meningeal
arteries that run between the dura and the skull. The middle meningeal artery
is most commonly affected.
The usual cause is a skull fracture.
More than half of cases occur in persons below 20 years of age. Epidural
haemorrhage is rare in persons over 40 years. In part, this can be explained by
the dura becoming increasingly adherent to the skull with age.
KEY: B Subarachnoid hemorrhage
Polycystic kidney: genetic marker "P" is the 16th letter of the alphabet.
Autosomal dominant Polycystic kidney disease is associated with abberation
on the 16th chromosome.



Berry aneurysms: are localised, dilatations of an intracranial artery, cause 80%
of subarachnoid haemorrhages
Additional
Information

124
CNS-System Wise 1700-by Sush and Team. 2016
Susmita, Asad, Manu, Saima, Zohaib, Savia, Shanu, Mona, Manisha, Sitara, Samreena, Sami and Komal

125
CNS-System Wise 1700-by Sush and Team. 2016
Susmita, Asad, Manu, Saima, Zohaib, Savia, Shanu, Mona, Manisha, Sitara, Samreena, Sami and Komal


Reference OHCM PG: 312 – PCKD; OHCM PG: 482 – Subarachnoid haemorrhage
Dr.Rabia Answer: B. subarachnoid hemorrhage. subarachnoid hemorrhage is mainly due
to rupture of berry aneurysm ( which is associated with adult polycystic kidney
disease, ehler danlos syndrome and coarctation of aorta)


SUBARACHNOID HEMORRHAGE
Causes
• 85% are due to rupture of berry aneurysms (conditions associated with
berry aneurysms include adult polycystic kidney disease, Ehlers-Danlos
syndrome and coarctation of the aorta)
• AV malformations
• trauma
• tumours

126
CNS-System Wise 1700-by Sush and Team. 2016
Susmita, Asad, Manu, Saima, Zohaib, Savia, Shanu, Mona, Manisha, Sitara, Samreena, Sami and Komal

Investigations
• CT: negative in 5%
• lumbar puncture: done after 12 hrs (allowing time for xanthochromia
to develop)

Complications
• rebleeding (in 30%)
• obstructive hydrocephalus (due to blood in ventricles)
• vasospasm leading to cerebral ischaemia

Management
• neurosurgical opinion: no clear evidence over early surgical
intervention against delayed intervention
• post-operative nimodipine (e.g. 60mg / 4 hrly, if BP allows) has been
shown to reduce the severity of neurological deficits but doesn't reduce
rebleeding.



Q: 798 A 36yo male dx with glioblastoma since last 5m has cerebral edema and is on
dexamethasone. He has diarrhea and vomiting for the last 3 days. He has been
suffering from repeated falls today. What could be the possible cause for his
falls?
a. Adrenal insufficiency
b. Dehydration
c. Dexamethasone therapy
d. Raised ICP

Clincher(s)
A When patient on exo steroidsds which gives negative signals to pituitary and
adrenal secretion become low. When suddenly stopped, there is adrenal
insufficiency. See below
B
C
D
E
KEY: A Adrenal insufficiency.
diarrhea, vomiting, falls (d/t postural hypotension) all point towards diagnosis.
It is because of the steroid resistance and these exogenous steroids would
have caused atrophy of adrenal glands.

For cerebral edema – dexamethasone given (can reduce peri-tumoral edema
(through rearrangement of the blood–brain barrier), diminishing mass effect
and lowering intracranial pressure, with a decrease in headache or drowsiness)

127
CNS-System Wise 1700-by Sush and Team. 2016
Susmita, Asad, Manu, Saima, Zohaib, Savia, Shanu, Mona, Manisha, Sitara, Samreena, Sami and Komal



Adrenal insufficiency is a condition in which the adrenal glands do not produce
adequate amounts of steroid hormones, primarily cortisol; but may also
include impaired production of aldosterone (a mineralocorticoid), which
regulates sodium conservation, potassium secretion, and water retention.
Craving for salt or salty foods due to the urinary losses of sodium is common.

adrenal insufficiency s/s: severe abdominal pains, vomiting, profound muscle
weakness and fatigue, depression, extremely low blood pressure
(hypotension), weight loss, kidney failure, changes in mood and personality,
and shock (adrenal crisis).
Additional
Information
Reference
Dr Rabia Adrenal insufficiency is a condition in which there is destruction of the adrenal
cortex and subsequent reduction in the output of adrenal hormones, ie
glucocorticoids (cortisol) and/or mineralocorticoids (aldosterone). There are
two types of adrenal insufficiency:

Primary insufficiency (Addison's disease) - there is an inability of the adrenal
glands to produce enough steroid hormones. The most common cause for this
in the developed world is autoimmune disease.
Secondary insufficiency - there is inadequate pituitary or hypothalamic
stimulation of the adrenal glands.
Presentation:
Often diagnosed late: lean, tanned, tired, tearful ± weakness, anorexia,dizzy,
faints, fl u-like myalgias/arthralgias. Mood: depression, psychosis, low self
esteem.
GI: nausea/vomiting, abdominal pain, diarrhoea/constipation. Think of
Addison’s in all with unexplained abdominal pain or vomiting. Pigmented

128
CNS-System Wise 1700-by Sush and Team. 2016
Susmita, Asad, Manu, Saima, Zohaib, Savia, Shanu, Mona, Manisha, Sitara, Samreena, Sami and Komal

palmar creases & buccal mucosa (ACTH; cross-reacts with melanin receptors).
Postural hypotension.Vitiligo. Signs of critical deterioration: Shock (↓BP,
tachycardia),T°↑, coma.
investigations:
In a patient with suspected Addison's disease the definite investigation is a
ACTH stimulation test (short Synacthen test).If a ACTH stimulation test is not
readily available (e.g. in primary care) then sending a 9 am serum cortisol can
be useful:
> 500 nmol/l makes Addison's very unlikely
< 100 nmol/l is definitely abnormal
100-500 nmol/l should prompt a ACTH stimulation test to be performed
Associated electrolyte abnormalities are seen in around one-third of
undiagnosed patients:
• hyperkalaemia
• hyponatraemia
• hypoglycaemia
• metabolic acidosis
management:
Patients who have Addison's disease are usually given both glucocorticoid and
mineralocorticoid replacement therapy.
This usually means that patients take a combination of:
• hydrocortisone: usually given in 2 or 3 divided doses. Patients typically
require 20-30 mg per day, with the majority given in the morning dose
fludrocortisone


Q: 802 A woman prv in good health presents with sudden onset of severe occipital
headache and vomiting. Her only physical sign on exam is a stiff neck. Choose
the most likely dx.
a. Subarachnoid hemorrhage
b. Subdural hematoma
c. Cerebellar hemorrhage
d. Migraine
e. Cerebral embolus

Clincher(s) With other headache, other history (e.g. of trauma) will be present
A
B Subdural hematoma usually presents with history of falls in an alcoholic or
elderly patient and may also shows signs of confusion.
C Cerebellar hemorrhage may present with signs and symptoms of cerebellar
lesion like lack of balance , nystagmus e.t.c.
D Migraine presents with unilateral severe throbbing pain with photophobia and
phonophobia. It lasts for about 4-72 hours and may occur along with nausea
and vomiting and aura.

129
CNS-System Wise 1700-by Sush and Team. 2016
Susmita, Asad, Manu, Saima, Zohaib, Savia, Shanu, Mona, Manisha, Sitara, Samreena, Sami and Komal


E Cerebral embolus rarely cause headache and presents with focal neurological
deficits.
KEY: A Subarachnoid haemorrhage
Any Severe headache of Sudden onset in previously well patient is
Subarachnoid hemorrhage unless proven otherwise.
It may also be described as “worst headache of my life.”
Additional
Information
Reference
DrRabia SUBARACHNOID HEMORRHAGE:

Presentation
The most characteristic feature is a sudden explosive headache. This may last a
few seconds or even a fraction of a second.
• Sudden explosive headache may be the only symptom in a third of
patients.
• Of patients who present with a sudden explosive headache as the only
symptoms, around 10% have SAH.
• It is difficult to suspect SAH without sudden headache, but with seizure
or confusional state - there are many other more common causes of these
presentations, but SAH should be on the list of differential diagnoses.
• Vomiting may occur; although this does not distinguish it from other
causes of headache.
• Seizures, occur in about 7%. When they do, they are highly suggestive
of a haemorrhage.
• 1-2% of patients with SAH present with an acute confusional state.
• Neck stiffness and other signs of meningism may be present, although
it usually presents around six hours after onset of SAH.
• Trauma may be confused with SAH if trauma has also occurred.
Patients may also cause a motor vehicle accident as a result of SAH. SAH needs
to be on the differential diagnostic list for patients with altered consciousness,
headache or seizure after trauma, particularly if there is disproportionate
headache or neck stiffness.
• SAH following head injury causes headache, decreased level of
consciousness and hemiparesis. SAH is a frequent occurrence in traumatic
brain injury, and carries a poor prognosis if it is associated with deterioration in
the level of consciousness.
Investigations
CT scanning
• If SAH is suspected, CT scanning (without contrast) is the first line in
investigation because of the characteristically hyperdense appearance of blood
in the basal cisterns.

130
CNS-System Wise 1700-by Sush and Team. 2016
Susmita, Asad, Manu, Saima, Zohaib, Savia, Shanu, Mona, Manisha, Sitara, Samreena, Sami and Komal

Lumbar puncture
• CT is negative in 2% of patients with SAH.
• If the CT scan is negative but the history is suggestive, lumbar puncture
should be undertaken, providing the scan shows no contra-indications. Around
3% of patients with a negative CT scan will prove, on lumbar puncture, to have
had an SAH.
• Lumbar puncture to remove a CSF sample from the lumbar sac should
ideally take place over 12 hours after the onset of the headache because if
there are red cells in the CSF, sufficient lysis will have taken place during that
time for bilirubin and oxyhaemoglobin to have formed
Angiography
• Further investigation should follow immediately acute SAH is
confirmed.
• After an SAH is confirmed, its origin needs to be determined. If the
bleeding is likely to have originated from an aneurysm, the choice is between
cerebral angiography (injecting radiocontrast through a catheter to the brain
arteries) and CT angiography (visualising blood vessels with radiocontrast on a
CT scan) to identify aneurysms. Catheter angiography also offers the possibility
of coiling an aneurysm.


Q: 803 A 34yo housemaid presents with headaches in the back of her head for several
days and pain on flexing her neck. What is the most likely cause?
a. Subdural hemorrhage
b. Cervical spondylosis
c. Subarachnoid hemorrhage
d. Meningitis
e. Cluster headache

Clincher(s) Housemaid, on flexing neck; pain (compression herniation) – over many
days

131
CNS-System Wise 1700-by Sush and Team. 2016
Susmita, Asad, Manu, Saima, Zohaib, Savia, Shanu, Mona, Manisha, Sitara, Samreena, Sami and Komal

A


B
C Subarachnoid hemorrhage is usually sudden in onset and does not prevail for
several days.
D Headache, pyrexia, neck stiff ness, altered mental state
E


KEY: B Cervical spondylosis
pain on flexion of neck shows nerve root irritation due to compression,
herniation or osteophytic outgrowth.

Additional Cervical spondylosis
Information

132
CNS-System Wise 1700-by Sush and Team. 2016
Susmita, Asad, Manu, Saima, Zohaib, Savia, Shanu, Mona, Manisha, Sitara, Samreena, Sami and Komal


Reference OHCM pg: 512 – Cervical spondylosis
DrRabia CERVICAL SPONDYLOSIS:
Cervical spondylosis is chronic cervical disc degeneration with herniation of
disc material, calcification and osteophytic outgrowths. As with simple back
pain, it is multifactorial in origin, reflecting poor posture, muscle strain,
sporting and occupational activities as well as psychological factors.
Cervical spondylosis undoubtedly contributes to this burden, but may also
cause:
• Radiculopathy due to compression, stretching or angulation of the
cervical nerve roots.
• Myelopathy due to compression, compromised blood supply or
recurring minor trauma to the cord.
Symptoms
• Cervical pain worsened by movement.
• Referred pain (occiput, between the shoulder blades, upper limbs).
• Retro-orbital or temporal pain (from C1 to C2).
• Cervical stiffness - reversible or irreversible.
• Vague numbness, tingling or weakness in upper limbs.
• Poor balance.
Signs
• Limited range of movement (forward flexion, backward extension,
lateral flexion, and rotation to both sides).
• Minor neurological changes like inverted supinator jerks (unless
complicated by myelopathy or radiculopathy).
• Poorly localised tenderness.
Investigations
Most patients do not need further investigation and the diagnosis is made on

133
CNS-System Wise 1700-by Sush and Team. 2016
Susmita, Asad, Manu, Saima, Zohaib, Savia, Shanu, Mona, Manisha, Sitara, Samreena, Sami and Komal

clinical grounds alone.
• Plain X-ray of cervical spine showing formation of osteophytes,
narrowing of disc spaces with encroachment of intervertebral foraminae. This
is not diagnostic as these findings are common in normal middle-aged
patients.
• Patients with neurological abnormality will need magnetic resonance
imaging (MRI) of the cervical spine at an early stage, particularly if they have
progressive myelopathy, radiculopathy or intractable pain.

Q: . A 35yo man presents with balance problems, headache, SNHL (sensory
neural hearng loss )and loss of coreneal reflex on the left side. What is the
most definitive inv?
a. CT scan of internal auditory meatus
b. Nuclear imagine of the brain
c. MRI of internal auditory meatus
d. MRI brain


Clincher(s) Clincher: balance problem, headache and unilateral SNHL (acoustic
neuroma)presenting origin from Cerbellopontine angle.
Loss of Corneal reflex shows involvment of trigeminal nerve.

A
B
C
D Most sensible choice (because the location is in the cerebellar pontine,
E
KEY KEY is D.
Diagnosis- Acoustic Neuroma. (7th 8th Nerve involvement)
Investigation of Choice for Diagnosing Acoustic Neuroma is MRI scan of the
cerebellopontine angle
Additional
Information ACOUSTIC NEUROMA:
Acoustic neuromas are tumours of the vestibulocochlear nerve (eighth cranial
nerve), arising from the Schwann cells of the nerve sheath. Most arise from the
vestibular portion and only a few arise from the cochlear (auditory) DIVISION

Presentation
Any unilateral sensorineural hearing loss should be considered as caused by an
acoustic neuroma until proven otherwise.

Consider the diagnosis of acoustic neuroma in patients with:

• Unilateral or asymmetrical hearing loss or tinnitus, whether


progressive or acute in onset.

134
CNS-System Wise 1700-by Sush and Team. 2016
Susmita, Asad, Manu, Saima, Zohaib, Savia, Shanu, Mona, Manisha, Sitara, Samreena, Sami and Komal

• Impaired facial sensation.

As the tumour spreads, there is an increase in hearing loss and disequilibrium,


and symptoms due to compression of other structures may occur:

• Facial pain or numbness due to involvement of the trigeminal


nerve.
• Facial weakness is uncommon despite the tumour pressing on the
facial nerve.
• Earache.
• Ataxia due to cerebellar compression.
• Severe brainstem compression can produce hydrocephalus with
visual loss and persistent headache and even decreased level of
consciousness.
• Balance problems without other explanation.


Reference
Dr Khalid/Rabia


Q:824 A 60yo man was brought in by his wife complaining of ataxia, urinary
incontinence and erectile dysfunction. He also complains of rigidity and
slowing of movement with a pill rolling tremor of the hands. What is the likely
dx?
a. Parkinson’s disease
b. Idiopathic parkinson’s disease
c. Shy-drager syndrome
d. Huntington’s disease

Clincher(s) 60yr, ataxia,urinary incont ,erectile dysfunc,rigidity, pill rolling tremors
(autonomic system involved- but Parkinsonsim does not involve ANS)
A Parkinson's disease (PD) is a movement disorder characterised by:[1]Tremor at
rest , Rigidity, Bradykinesia . diagnosis is almost entirely based on clinical
examination. It is caused by degeneration of the dopaminergic pathways in the
substantia nigra.

B • Parkinson's disease (PD) is used to describe the idiopathic syndrome of


Parkinsonism

C Shy dragor synd- rare degenerative condition resulting from degeneration of


certain nerve cells in the brain and spinal cord. These functions include the

135
CNS-System Wise 1700-by Sush and Team. 2016
Susmita, Asad, Manu, Saima, Zohaib, Savia, Shanu, Mona, Manisha, Sitara, Samreena, Sami and Komal

autonomic (which controls blood pressure, heart rate, and bladder function)
and the motor system (which controls balance and muscle movement).

D Huntingtons disease - autosomal dominant-The symptoms of HD can be
grouped into three main areas:

• Problems with movement- chorea, dystonia,


• Problems with cognition- dementia
• Mood and behavioural problems- irritated ,


E C
KEY KEY is C
SHY DRAGER SYNDROME:
Shy-Drager Syndrome (Multiple System Atrophy) (SDS)(MSA) is a a rare
degenerative condition resulting from degeneration of certain nerve cells in
the brain and spinal cord. Body functions controlled by these areas of the brain
and spinal cord do not function normally in sufferers. These functions include
the autonomic or involuntary nervous system (which controls blood pressure,
heart rate, and bladder function) and the motor system (which controls
balance and muscle movement).
Depending upon which part of the brain is affected first, MSA may appear in
different ways. Sometimes it presents with low blood pressure on standing,
urinary bladder problems, or difficulties with balance and movement that
resemble Parkinson's disease. Early symptoms often include impotence and
urinary incontinence. Most patients complain of severe constipation and later
develop rectal incontinence.
.
Diagnosis criteria for parkinsons disease from patient.info.uk
Step 1: diagnosis of Parkinsonian syndrome
Bradykinesia (slowness of initiation of voluntary movement with progressive
reduction in speed and amplitude or repetitive actions) and at least one of the
following:

• Muscular rigidity.
• 4- to 6-Hz resting tremor.
• Postural instability not caused by primary visual, vestibular, cerebellar or
proprioceptive dysfunction.

Step 2: exclusion criteria for Parkinson's disease

136
CNS-System Wise 1700-by Sush and Team. 2016
Susmita, Asad, Manu, Saima, Zohaib, Savia, Shanu, Mona, Manisha, Sitara, Samreena, Sami and Komal

• History of repeated strokes with stepwise progression of Parkinsonian
features.
• History of repeated head injury.
• History of definite encephalitis.
• Oculogyric crises.
• Neuroleptic treatment at onset of symptoms.
• More than one affected relative.
• Sustained remission.
• Strictly unilateral features after three years.
• Supranuclear gaze palsy.
• Cerebellar signs.
• Early severe autonomic involvement.
• Early severe dementia with disturbances of memory, language and praxis.
• Babinski's sign.
• Presence of a cerebral tumour or communicating hydrocephalus on CT
scan.
• Negative response to large doses of L-dopa (if malabsorption excluded).
• Exposure to MPTP.

Step 3: supportive prospective positive criteria of Parkinson's disease


Three or more are required for the diagnosis of definite PD:

• Unilateral onset.
• Rest tremor present.
• Progressive disorder.
• Persistent asymmetry affecting the side of onset most.
• Excellent response (70-100%) to L-dopa.
• Severe L-dopa-induced chorea.
• L-dopa response for five years or more.
• Clinical course of ten years or more.
• Hyposmia.
• Visual hallucinations.

137
CNS-System Wise 1700-by Sush and Team. 2016
Susmita, Asad, Manu, Saima, Zohaib, Savia, Shanu, Mona, Manisha, Sitara, Samreena, Sami and Komal

Additional
Information Major features supporting diagnosis of probable MSA

System Feature Notes

Autono Severe (symptomatic or otherwise) Blood pressure fall by ≥30 mm


mic orthostatic hypotension. Commonly Hg systolic and ≥15 mm Hg
associated symptoms include light- diastolic within three minutes
headedness, dizziness, weakness of of standing from a previous
legs, fatigue and syncope. Postprandial three-minute supine position.
hypotension may be a major feature. Associated supine hypertension
is common, and is aggravated
by medication used to reduce
orthostatic hypotension.

• Urinary incontinence or • Urinary dysfunction


Urogeni incomplete emptying. is the most frequent
tal • Erectile dysfunction. initial complaint in
women.
• Erectile dysfunction
is the most frequent
initial complaint in
men.

• Bradykinesia.
Extrapyr • Rigidity. Check that postural instability is

138
CNS-System Wise 1700-by Sush and Team. 2016
Susmita, Asad, Manu, Saima, Zohaib, Savia, Shanu, Mona, Manisha, Sitara, Samreena, Sami and Komal

amidal • Postural instability. not caused by primary visual,
tract • Tremor (but not classic pill vestibular, cerebellar, or
rolling). proprioceptive dysfunction.

• Gait/limb ataxia.
Cerebell • Ataxic dysarthria.
ar • Oculomotor dysfunction
function (sustained gaze-evoked
nystagmus).

Reference Patient.info

Dr Khalid/Rabia


Q:827 827. Removal of a glioma, which single most appropriate anatomical
structure will be pierced?
a. Cricoid cartilage
b. Rectus sheath muscle
c. Dura Mater
d. Conjoined tendon
e. Intercostal muscles

Clincher(s) Malignant tissue arising from brain or nervous system ‘s glial cells.
A
B
C
D

139
CNS-System Wise 1700-by Sush and Team. 2016
Susmita, Asad, Manu, Saima, Zohaib, Savia, Shanu, Mona, Manisha, Sitara, Samreena, Sami and Komal

E
KEY Most prob C -dura matter is the answer AS THE QUEST IS INCOMPLETE
Additional
Information
Reference
Dr Khalid/Rabia


Q:897 A 64yo man presents with ipsilateral vertigo, tinnitus and left side hearing loss.
Exam: Rinne test +ve and Weber’s lateralizes to the right ear. What is the most
appropriate inv?
a. CT
b. MRI brain
c. XR
d. Audiometry
e. None
f. Caloric testing






Clincher(s) vertigo, tinnitus, hearing loss
A
B MRI has largely superseded CT scanning as the investigation of choice for
suspected acoustic neuroma. (Patient.co.uk)

C
D Test to check for hearing loss .conductive vs sensorineural hearing loss
E None
F Caloric testing -
KEY Key : b (Acoustic Neruoma)

Additional
Information ACOUSTIC NEUROMA (source : nhs )

An acoustic neuroma is a benign (non-cancerous) growth, or tumour, in the
brain. It's also known as a vestibular schwannoma.
An acoustic neuroma grows on the vestibulocochlear nerve, which helps
control hearing and balance. This nerve runs alongside the facial nerve, which
carries information from the brain to the face muscles.

A small acoustic neuroma can lead to hearing loss or tinnitus, vertigo
A large acoustic neuroma can cause headaches with blurred vision, numbness
or pain on one side of the face, problems with limb co-ordination on one side

140
CNS-System Wise 1700-by Sush and Team. 2016
Susmita, Asad, Manu, Saima, Zohaib, Savia, Shanu, Mona, Manisha, Sitara, Samreena, Sami and Komal

of the body.

The cause of most acoustic neuromas is unknown, but a small number of cases
(about 5%) are caused by a rare, inherited condition called neurofibromatosis
type 2.Acoustic neuromas grow from the cells that cover the vestibulocochlear
nerve, called Schwann cells.

The three types of test you may have are:
Hearing tests, MRI, CT

acoustic neuromas can be treated before they reach advance stage, either
with surgery or radiotherapy
Reference Nhs
Dr rabia
Dr Khalid/Rabia Rene’s – if there is defect in conduction (know)


Q:906 A 23yo man sprained his right ankle 6wks ago while playing football. He was tx
with a below knee walking cast. On removal of the cast, the pt noted to have
right foot drop. He has weakness of extensors of the ankle and toes and
diminished pin prick sensation over the dorsum of the foot. The ankle jerk is
present and plantar reflex is flexor. What is the most likely cause of the foot
drop?
a. Compression of common peroneal nerve
b. Compression of the tibial nerve
c. Compression of the S1 nerve root
d. Rupture of Achilles tendon
e. Tx of the medial collateral leg of the ankle


Clincher(s) right foot drop, weakness of extensors of ankle and toes
diminished pin prick sensation over the dorsum of Rt leg
Means injury to lateral and ant compartment of foot
A PED – Peroneal nerve , ,extensor weakness(Eversion), footDrop
The common peroneal nerve crosses the fibular neck and is susceptible to
injury from a fibular neck fracture or compression from cast.
B Tibial nerve- Damage to the tibial nerve is uncommon. Most usually, there is
paralysis of gastrocnemius which is easily tested by asking the patient to stand
on tiptoe. Plantar flexion and inversion of the foot are also weakened. The
ankle jerk is absent.

Entrapment of the posterior tibial nerve at the level of the medial malleolus
causes posterior tarsal syndrome. It is characterised by pain in the sole.
Treatment is by surgical decompression.

141
CNS-System Wise 1700-by Sush and Team. 2016
Susmita, Asad, Manu, Saima, Zohaib, Savia, Shanu, Mona, Manisha, Sitara, Samreena, Sami and Komal

C • S1 is dermatome S1 lateral aspect of the foot, the heel and most of the
sole


D Achilles (uh-KILL-eez) tendon rupture is an injury that affects the back of your
lower leg. It most commonly occurs in people playing recreational sports.
The Achilles tendon is a strong fibrous cord that connects the muscles in the
back of your calf to your heel bone.

• Pain severe and swelling near your heel


• An inability to bend your foot downward or "push off" the injured leg
when you walk
• An inability to stand on your toes on the injured leg
• A popping or snapping sound when the injury occur

E
KEY A- Common peroneal nerve
Additional
Information Causes of unilateral foot drop (source: OHCM Pg # 471 8th edition)

DM
common peroneal nerve palsy
Stroke
Prolapsed disc
MS (multiple sclerosis)

The common peroneal nerve represents a major nerve terminal of the sciatic
nerve, providing the motor supply to the muscles of the anterior and lateral
compartments of the leg. It is the most commonly damaged nerve in the lower
limb and is relatively unprotected as it traverses the lateral aspect of the head
of the fibula.

Damage may occur from direct trauma, pressure by plaster casts at the back of
the fibula, or stretching after a prolonged period with the knee fully flexed, e.g.
kneeling. Injury is not uncommon in patients lying in bed recovering from
operations.

Clinical features include:

• foot drop
• weakness of dorsiflexion and eversion of the foot
• weakness of extensor hallucis longus
• inversion and plantar flexion are normal
• anaesthesia over the lower lateral part of the leg and dorsum of the
foot; often with little or no sensory loss

142
CNS-System Wise 1700-by Sush and Team. 2016
Susmita, Asad, Manu, Saima, Zohaib, Savia, Shanu, Mona, Manisha, Sitara, Samreena, Sami and Komal

• all reflexes are intact - the ankle jerk is lost in a sciatic nerve lesion
• wasting of the anterior tibial and peroneal muscles

Recovery occurs within a few weeks when the cause is simple compression.
Full knee flexion should be avoided as in kneeling or squatting, and the patient
should not sit with the legs crossed over the unaffected leg. To prevent foot
drop the patient should wear an aluminium night-shoe at night and during the
day, a shoe with plastic inserts.

Surgical exploration is indicated if the weakness progresses or fails to resolve


within 1-2 months, or if there is an obvious local lesion.

Gp notebook

Reference
Dr Khalid/Rabia (findings are opposite in tibial nerve injury to Perineal nerve. When extensors
are affected: leads to eversion) Here cast compressed the extensors and caused
a foot drop.

Q:927 A 5yo child came from Ghana 6wks ago. 2d ago he developed fever, vomiting
and neck stiffness.
He had taken malaria prophylaxis and had no rash. What is the dx?
a. Cerebral abscess
b. Cerebral malaria
c. Meningococcal meningitis
d. SAH
e. Cerebral tumor
f. Pneumonia


Clincher(s) 5yr old child, fever , vomiting , neck stiffness
A ABSCESS- . abscess have typical swinging fever pattern.
B Malaria - Malaria prophylaxis doesn't provide full protection against all
subtypes of malarial parasites
C Meningitis meningococcal meningitis is fever with a rash.

D SAH- headache
E TUMOR-
KEY B-Cerebral malaria
Additional Malaria prophylaxis does not give full protection for all strains of malaria.
Information .

Suspect malaria in any patient has travelled to endemic areas within one year
(patient com). Antimalarial prophylaxis don't give full immunity and infection
can be happened (BNF).
a-e: focal signs

143
CNS-System Wise 1700-by Sush and Team. 2016
Susmita, Asad, Manu, Saima, Zohaib, Savia, Shanu, Mona, Manisha, Sitara, Samreena, Sami and Komal

c- rash
d- sudden
f-cough

Consider malaria in every febrile patient returning from a malaria-endemic
area within the last year, especially in the previous three months, regardless of
whether they have taken chemoprophylaxis, as prompt recognition and
appropriate treatment will improve prognosis and prevent deaths.


Reference
Dr Khalid/Rabia

Q:933 933. A 3yo girl presents with complaints of sudden right facial weakness and
numbness and pain around her ear. There are no symptoms. What is the most
appropriate dx?
a. SAH
b. Bell’s palsy
c. Stroke
d. TIA
e. Subdural hemorrhage


Clincher(s)
A SAH – from trauma or berry aneurysm -sudden severe headache , alt
consciousness , seizures , vomiting.
B Classical features
C In older
D Less likely
E No hx of trauma
KEY B-Bells palsy
Additional Bell’s palsy
Information
Bell's palsy may be defined as an acute, unilateral, idiopathic, facial nerve
paralysis. The aetiology is unknown although the role of the herpes simplex
virus has been investigated previously. The peak incidence is 20-40 years and
the condition is more common in pregnant women.

Features
• lower motor neuron facial nerve palsy - forehead affected*
• patients may also notice post-auricular pain (may precede paralysis),
altered taste, dry eyes, hyperacusis

Management
• in the past a variety of treatment options have been proposed
including no treatment, prednisolone only and a combination of

144
CNS-System Wise 1700-by Sush and Team. 2016
Susmita, Asad, Manu, Saima, Zohaib, Savia, Shanu, Mona, Manisha, Sitara, Samreena, Sami and Komal

aciclovir and prednisolone
• following a National Institute for Health randomised controlled trial it is
now recommended that prednisolone 1mg/kg for 10 days should be
prescribed for patients within 72 hours of onset of Bell's palsy. Adding
in aciclovir gives no additional benefit
• eye care is important - prescription of artificial tears and eye lubricants
should be considered


Reference
Dr Khalid/Rabia With UMN lesion in Facial nerve: sparing of forehead; but in LMN: everything
is dead;


Q:953 A 68yo pt wakes up with slurred speech and right sided weakness. CT shows
cerebral infarct. What is the most appropriate tx?
a. Aspirin
b. Alteplase
c. Warfarin
d. Clopidogrel
e. Dipyridamole

ANS: B


Clincher(s)
A
B : If a patient with ischemic stroke presents within 4.5 hours of ischemic
episode thrombolysis can be tried
C
D
E
KEY b-alteplase
Additional Reason : If a patient with ischemic stroke presents within 4.5 hours of ischemic
Information episode thrombolysis can be tried. Do a CT 24 hrs Post thrombolysis.after that
patients are prescribed Aspirin 300 mg for 2 weeks.( ischemic stroke is a
gradual process and episodes might take place during sleep)
CI:
major infarct or hemorrhage
mild deficit
Recent Birth,Surgery,Trauma
Past CNS bleed
Seizures at presentation
platelets <100
BP >220/130

145
CNS-System Wise 1700-by Sush and Team. 2016
Susmita, Asad, Manu, Saima, Zohaib, Savia, Shanu, Mona, Manisha, Sitara, Samreena, Sami and Komal

Reference
Dr Khalid/Rabia


Q:954 . A 73yo man who is recovering from surgery on the left carotid artery in his
neck. He has slurred speech. On protrusion of his tongue, the tip deviated to
the left. What is the single most appropriate option?
a. Accessory nerve
b. Facial nerve
c. Glossopharyngeal nerve
d. Hypoglossal nerve
e. Vagus nerve
Ans: D



Clincher(s)
A
B
C In UMN leision: Tongue deviates away from side of lesion, and in LMN:
towards lesion
D
E
KEY d-hypoglossal nerve
Additional Reason: Ask the patient to protrude his/her tongue and note any deviation. A
Information fluttering motion called fibrillation rather than fasciculation may be seen with
an LMN lesion.
If the tongue deviates to one side when protruded, this suggests a hypoglossal
nerve lesion. If it is an LMN lesion, the protruded tongue will deviate towards
the side of the lesion. With a UMN lesion, the tongue will deviate away from
the side of the lesion
Causes of a single XII lesion: rare. Polio,syringomyelia tuberculosis, median
branch thrombosis of the vertebral artery
Reference
Dr Khalid/Rabia


Q:971 A 45yo male complains of tremors in hands. Exam: tremors are absent at rest
but present when arms are held outstretched and persist on movement. What
is the most probable dx?
a. Parkinsonism
b. Benign essential tremor
c. Cerebellar disease
d. Liver failure
e. Stroke

146
CNS-System Wise 1700-by Sush and Team. 2016
Susmita, Asad, Manu, Saima, Zohaib, Savia, Shanu, Mona, Manisha, Sitara, Samreena, Sami and Komal



Clincher(s)
A Resting tremors
B no tremors at rest ,present on persisted posture and movement . (Postural
tremors)
C intention tremor: irregular, large-amplitude, worse at the end of purposeful
acts, eg finger-pointing or using a remote control. Cause: cerebellar damage
(eg MS, stroke). [this is not the full picture of cerebellar disease; eg. Vertigo,
posture, ataxia, etc: pnusomic: DASHING: Dys, ataxia, hypotonianystagmus,
gait,]
D Flapping tremors
E
KEY Ans: B
Reason . the patient is suffering from postural tremors , no tremors at rest
,present on persisted posture and movement . Causes : Benign Essential
Tremor

Additional
Information Rest tremor: abolished on voluntary movement. Cause: parkinsonism.
Intention tremor: irregular, large-amplitude, worse at the end of purposeful
acts, eg finger-pointing or using a remote control. Cause: cerebellar damage
(eg MS, stroke). Postural tremor: absent at rest, present on maintained
posture (arms out-stretched) and may persist (but is not worse) on movement.
Causes: Benign essential tremor (autosomal dominant; improves with alcohol),
thyrotoxicosis, anxiety, β-agonists
Reference
Dr Khalid/Rabia



Q: 1352 A 46yo male presents with confusion and drowsiness. What is the most likely
dx?
a. Cryptococcus neoformans
b. Toxoplasma gondii
c. HSV
d. CMV
e. Candida albicans


Clincher(s) Confusion and drowsiness indicating signs of delirium after infection.

A Cryptococcus neoformans is common in immumnocompromised patients.


OHCM pg 440

147
CNS-System Wise 1700-by Sush and Team. 2016
Susmita, Asad, Manu, Saima, Zohaib, Savia, Shanu, Mona, Manisha, Sitara, Samreena, Sami and Komal

B also causes these symptoms. but it is common in immunocompromised

C Herpes simplex encephalitis (HSE) is recognised worldwide as the most


frequent infectious encephalitis. Patient.info. (others for immunocompromised
patients)
D les common.

E same symptoms but common in immunocompromised.

KEY C- HSV. ( Please correct key on 1700 questions)


Additional
Information

Reference OHClinical neurology PG 370

Dr Khalid/Rabia

148
CNS-System Wise 1700-by Sush and Team. 2016
Susmita, Asad, Manu, Saima, Zohaib, Savia, Shanu, Mona, Manisha, Sitara, Samreena, Sami and Komal

Q: 1369 A pt came to the ED after he had banged his car quite a few times on reversing.
He was complaining of seeing double while he tried to look back during the
process of reversing the car, he also complains of double vision on looking at an
outward gaze. Which nerve is involved?

a. Abducent nerve

b. Trochlear nerve

c. Oculomotor nerve

d. Optic nerve

e. Trigeminal nerve


Clincher(s) Double vision on outward gaze.
A Abducent nerve paralysis. most common cause of diplopia. (patient.info)
Outward gaze (can’t use his lat rectus muscle)
B superior oblique muscle depresses, intorts, and abducts the globe. Vertical,
horizontal and torsional diplopia. patient has downward gaze.

C deviated downward and there is also ptosis.(medscape)


D vision loss
E not significant. (no link with extra ocular muscles)

KEY A abducent nerve.


Additional
Information

Reference OHCSpeciality pg 422

Dr Khalid/Rabia

149
CNS-System Wise 1700-by Sush and Team. 2016
Susmita, Asad, Manu, Saima, Zohaib, Savia, Shanu, Mona, Manisha, Sitara, Samreena, Sami and Komal





Q: 1370 A pt had a stroke Now, there is left sided weakness and right side facial
numbness. CT shows ischemic stroke. Which one would you prescribe?

a. Alteplase
b. Aspirin
c. Clopidogrel
d. Heparin
e. Warfarin


Clincher(s) ischemic stroke.
A Considering patient is presented within 4.5 hour of stroke episode, we will
thrombolyse this patient with alteplase.
B Aspirin is given if there is a contraindication to thrombolysis. 300mg PO/PR for
two weeks then 75mg PO daily. (then clopidogrel)
C
D
E A
KEY

150
CNS-System Wise 1700-by Sush and Team. 2016
Susmita, Asad, Manu, Saima, Zohaib, Savia, Shanu, Mona, Manisha, Sitara, Samreena, Sami and Komal

Additional
Information

151
CNS-System Wise 1700-by Sush and Team. 2016
Susmita, Asad, Manu, Saima, Zohaib, Savia, Shanu, Mona, Manisha, Sitara, Samreena, Sami and Komal

Reference OHC neurology PG 80

Dr Khalid/Rabia





Q: 1376 A 32yo male complains of tremors everytime he tends to use his muscles and
when he is pointing at objects. No complaints at rest. His father complained of
similar problems. What is
the most probable dx?
a. Parkinsonism
b. Lithium toxicity
c. Thyrotoxicosis
d. Benign essential tremor


Clincher(s)
A resting tremors
B coarse and fine tremors
C fine tremors
D Essential tremor (previously called benign essential tremor) is an autosomal
dominant condition

E
KEY d
Additional Essential tremor (previously called benign essential tremor) is an autosomal
Information dominant condition which usually affects both upper limbs postural tremor:
worse if arms outstretched improved by alcohol and rest. `head
tremor(titubation) is a most common example of essential tremors.

(Intentional tremor in cerebellar)

Reference
Dr Khalid/Rabia Dr rabia


152
CNS-System Wise 1700-by Sush and Team. 2016
Susmita, Asad, Manu, Saima, Zohaib, Savia, Shanu, Mona, Manisha, Sitara, Samreena, Sami and Komal



Q: 1393 A 32yo man on psychiatric meds presents with coarse tremors and diarrhea.
What is the most likely alternate to the drug causing the prb?

a. Lithium
b. Diazepam

c. Haloperidol
d. Valproate
e. Citalopram


Clincher(s) side effect of lithium
A nausea, vomiting, diarrhea and coarse tremors.
B
C
D
E
KEY Valproate (as it’s the alternate to Li)
Additional
Information
Reference
Dr Khalid/Rabia



153
CNS-System Wise 1700-by Sush and Team. 2016
Susmita, Asad, Manu, Saima, Zohaib, Savia, Shanu, Mona, Manisha, Sitara, Samreena, Sami and Komal

Q: 1408 A 68yo man awoke to find that he is unable to close his left eye and dribbling
saliva from the left angle of his mouth. What is the single most appropriate
option?

a. Facial nerve
b. Glossopharyngeal nerve

c. Hypoglossal nerve
d. Optic nerve
e. Vagus nerve


Clincher(s) Loss of angle of mouth, dribbling saliva and
A Facial nerve Bells Palsy (LMN). features of 7th nerve palsy include drooling
of saliva, failure to close the eye, unilateral sagging of the mouth, absence of
nasolabial fold, loss of taste in anterior 2/3rds of the tongue, hyperacusis
(hypersensitivity to sounds) (loss of innervation to stapedius).

B loss of taste sensation on the posterior one third

C Hypoglossal nerve tongue atrophy, Deviation of the tongue on the same side of
the lesion
D loss of vision
E A
KEY
Additional
Information

Reference ohcm pg 504

Dr Khalid/Rabia (hypoglossal: tongue deviation,

154
CNS-System Wise 1700-by Sush and Team. 2016
Susmita, Asad, Manu, Saima, Zohaib, Savia, Shanu, Mona, Manisha, Sitara, Samreena, Sami and Komal

vagus:
Gag reflex lost in glossopharangeal and vagus (sami will post note)




Q: 1423 A 63yo male presents after having had a seizure. Exam: alert and oriented.
Exam: inattention on the left side and hyperreflexia of the arm. What is the most
probable dx?
a. Cerebral tumor

b. Pituitary adenoma
c. Cerebellar abscess
d. Huntington’s chorea
e. Parkinsonism


Clincher(s) space occupying lesion, seizure.
A cerebral tumor is the cause of focal seizure and hyperreflexia (in 50% cases,
first seizure indicates cerebellar tumor)
B no Pituitary adenoma signs like gigantism, glactorrea, diabetes insidious.
C no Cerebellar abscess sign of fever, head, neck rigidity.
D no Huntington’s chorea signs of repetition of movements and family history.
E there is no seizures in parkinsonism.

KEY A
Additional
Information

155
CNS-System Wise 1700-by Sush and Team. 2016
Susmita, Asad, Manu, Saima, Zohaib, Savia, Shanu, Mona, Manisha, Sitara, Samreena, Sami and Komal

Reference ohcm pg 502

Dr Khalid/Rabia





Q: 1433 A 67yo female presents with balance prbs. Exam: nystagmus on left lateral gaze,
a loss of the let corneal reflex and reduced hearing in the left ear. What is the
most likely dx?
a. Meniere’s disease

b. Acoustic neuroma
c. Cerebral abscess
d. Pituitary tumor
e. Gentamicin


Clincher(s)
A

B Acoustic Neuroma: typical presentation: Balance problems, nystagmus, loss of


corneal reflex and reduced unilateral hearing points towards the diagnosis.
C

D
E no drug history
KEY B
Additional
Information

Reference OHCM pg 466

Dr Khalid/Rabia


156
CNS-System Wise 1700-by Sush and Team. 2016
Susmita, Asad, Manu, Saima, Zohaib, Savia, Shanu, Mona, Manisha, Sitara, Samreena, Sami and Komal



Q: 1437 A 72yo male presents with acute confusion. He has been in the hosp for 2wks
having been treated for a DVT. The nurses have noticed that he became
increasingly drowsy. Exam: small scalp laceration, a GCS of 8 and bilateral up-
going plantar response.> UMN

a. Infection toxicity
b. Delirium tremens
c. Extradural hematoma

d. Subdural hematoma
e. Electrolyte imbalance


Clincher(s)
A
B
C
D subdural Hematoma. elderly patient, having been treated for DVT so on
anticoagulants,increasing drowsiness, scalp lacerations (head trauma) all points
towards the diagnosis. (also happens in alcoholic, on epileptic, elderly, patients)

E
KEY D

Additional
Information
Reference

Dr Khalid/Rabia




157
CNS-System Wise 1700-by Sush and Team. 2016
Susmita, Asad, Manu, Saima, Zohaib, Savia, Shanu, Mona, Manisha, Sitara, Samreena, Sami and Komal

Q: 1525 (?) A 56yo male who presented with epilepsy like symptoms has been dx with an
intracranial space occupying lesion. He now complains of thirst and mild
dehydration. His blood glucose is also increased. What is the single most
appropriate immediate tx?

a. Insulin

b. IV fluids
c. Stop dexamethasone
d. Stop sodium valproate and change to another anti-epileptic


Clincher(s)
A
B IV fluids for replacement.
C
D
E
KEY B
Additional Diabetes insipidus is a condition in which your ability to control the balance of
Information water within your body is not working properly. Your kidneys are not able to
retain water and this causes you to pass large amounts of urine. Because of this,
you become more thirsty and want to drink more. There are two different types
of diabetes insipidus: cranial and nephrogenic.Treatment includes drinking
plenty of fluids so that you do not become dehydrated. Treatment with
medicines may be also needed for both types of diabetes insipidus.

Reference
Dr Khalid/Rabia

158
CNS-System Wise 1700-by Sush and Team. 2016
Susmita, Asad, Manu, Saima, Zohaib, Savia, Shanu, Mona, Manisha, Sitara, Samreena, Sami and Komal

Q: 1541 A 62yo man with chronic schizophrenia presents with a mask like face and
involuntary pill rolling movement in both hands. He complains of chronic cough
and forgetfulness. He is on long term antipsychotic meds. What is the single
most likely dx?

a. Shy drager syndrome

b. Parkinsonism
c. Huntington’s chorea

d. Tardive dyskinesia

e. Akathisia (a feeling of inner restlessness)

Key B
Clinchers pin rolling tremors, chronic schizophrenia, longterm antipsychotic medicines. B

A Shy drager syndrome A progressive disorder of the central and sympathetic


nervous systems, also called multiple system atrophy with postural
hypotension (an excessive drop in blood pressure when the patient stands up,
causing dizziness or momentary blackouts). Present like parkinsonism but with
no response to dopamine. Constipatiation, impotence, impaired speech,
difficulty with breathing and swallowing, anhydrosis . Death in 7-10 yr.
B Parkinsonism is a clinical syndrome characterized by tremor, bradykinesia,
rigidity, and postural instability which May later be associated with dementia or
depression.
About 7% of people with parkinsonism have developed their symptoms
following treatment with particular medications. Side effect of medications,
mainly neuroleptic antipsychotics.

c Huntington's disease (HD) is a neurodegenerative genetic disorder that affects


muscle coordination and leads to mental decline and behavioral symptoms

D Tardive dyskinesia a disorder resulting in involuntary, repetitive body


movements. also occurs in antipyschotic treatment.

E Akathisia Movement disorder characterized by a feeling of inner restlessness


and a compelling need to be in constant motion, as well as by actions such as
rocking while standing or sitting, lifting the feet as if marching on the spot, and
crossing and uncrossing the legs while sitting. first generation antipsychotics
drugs cause this condition. (e.g. restless leg syndrome)

B
Dyskinea, akathisia and Parkinsonism also SE of haloperidol

159
CNS-System Wise 1700-by Sush and Team. 2016
Susmita, Asad, Manu, Saima, Zohaib, Savia, Shanu, Mona, Manisha, Sitara, Samreena, Sami and Komal







Q: 55 A 75yo man collapsed while walking in his garden. He recovered fully within 30
mins with BP 110/80 mmHg and regular pulse of 70bpm. He has a systolic
murmur on examination. His activities have been reduced lately which he
attributes to old age. What is the definitive diagnostic inv that will assist you
with his condition?
a. ECG
b. Echo
c. 24h ECG monitoring
d. 24h BP monitoring
e. Prv CIN




Clincher(s) Exertional-Syncope, sys murmur
A
B Aortic stenosis – more likely in elderly. ii) hypertrophic cardiomyopathy – less
likely in this age as presentation may present in an earlier age.
C
D
E
Key B. Echo.
Additional
Information
Reference
Dr Khalid/Rabia Aortic stenosis (AS)
Causes: Senile calcification is the commonest. Others: congenital (bicuspid
valve, William’s syndrome, rheumatic heart disease.

Presentation: Think of AS in any elderly person with chest pain, exertional
dyspnoea or syncope. The classic triad includes angina, syncope, and heart
failure (usually after age 60). Also: dyspnoea; dizziness; faints; systemic emboli
if infective endocarditis; sudden death.

Signs: Slow rising pulse with narrow pulse pressure (feel for diminished and
delayed carotid upstroke—parvus et tardus); heaving, non-displaced apex
beat; LV heave; aortic thrill; ejection systolic murmur (heard at the base, left
sternal edge and the aortic area, radiates to the carotids).

160
CNS-System Wise 1700-by Sush and Team. 2016
Susmita, Asad, Manu, Saima, Zohaib, Savia, Shanu, Mona, Manisha, Sitara, Samreena, Sami and Komal

There may be an ejection click (pliable valve) or an S4 (said to occur more
often with bicuspid valves, but not in all populations).

Tests: ECG: P-mitrale, LVH with strain pattern; LBBB or complete AV block
(calcified ring).
CXR: LVH; calcified aortic valve post-stenotic dilatation of ascending aorta.

Echo: diagnostic.
Doppler echo can estimate the gradient across valves
Cardiac catheter can assess: valve gradient; LV function; coronary artery
disease
Management: If symptomatic, prognosis is poor without surgery.
If moderate-to-severe and treated medically, mortality can be as high as 50%
at 2yrs, therefore prompt valve replacement is usually recommended.
In asymptomatic patients with severe AS and a deteriorating ECG, valve
replacement is also recommended.

If the patient is not medically fit for surgery, percutaneous
valvuloplasty/replacement (TAVI = transcatheter aortic valve implantation)
may be attempted.



Q: 59 59. A 62yo woman complains of unsteadiness when walking. On examination
she has pyramidal weakness of her left lower limb (IPSI-Pyr/CST) and reduced
pain and temp sensation on right leg and right side of trunk up to the umbilicus
(CONTRA-STT). Joint position sense is impaired at her left great toe (IPSI-
DCML) but is normal elsewhere. She has a definite left extensor plantar
response (IPSI- CST) and the right plantar response is equivocal. Where is the
lesion?
Brown Sequard syndrome:

DCML (Propriocetion/fine touch/vibration/joint position sense)= Ipsi
STT (crude Pain/temp/pressure)=Contra
CST (limbs/axial muscle- extensor reflex/spastic etc)=Ipsi
At level of injury=Total sensory loss

To get answer quick:

SIDE: See which side is pain and temp (STT): so leision is contralateral to that
side. Here pain and tem reduced on right, so lesion on rleft side

LEVEL: See which level is cutaneous sensation loss (and lesion is at that level or
lower): here it is at trunk: so mid thoracic

a. Left cervical cord

161
CNS-System Wise 1700-by Sush and Team. 2016
Susmita, Asad, Manu, Saima, Zohaib, Savia, Shanu, Mona, Manisha, Sitara, Samreena, Sami and Komal

b. Midline mid-thoracic cord
c. Right mid-thoracic cord
d. Left mid-thoracic cord
e. Left lumbo-sacral plexus



Clincher(s)
A
B
C
D Brown Sequard syndrome:
A lesion in one half of the spinal cord (due to hemisection or unilateral cord
lesion)- can be due to trauma. Gunshot, stab, metastatic tumour, infraction,
MS etc (will affect that level and anything below it)

s/s
Consider whether deficit are ipsi or contra to side of lesion (see pic in note
book):

DCML (Proprioception/fine touch/vibration/joint position sense)= Ipsi
STT (crude Pain/temp/pressure)=Contra
CST (limbs/axial muscle- spastic)=Ipsi
At level of injury=Total sensory loss

one sided motor loss/ spastic hemiplegia
one sided fine touch and vibration sensation loss
and other sided pain and temp loss


Inv: MRI (best intial)

Rx: Underlying cause Rx, non penetrating: steorids (reduce swelling),
Penetrating: surgery

Ref: CRASH

TRACTS:

Sensory
DCML: Fine touch/vibration
Spinothalamic tracts:
Lat: pain amd temp
Ant: ccrude touch and pressure

Motor

162
CNS-System Wise 1700-by Sush and Team. 2016
Susmita, Asad, Manu, Saima, Zohaib, Savia, Shanu, Mona, Manisha, Sitara, Samreena, Sami and Komal

Corticospinal (pyramidal) tract
Lat: Primarily limbs
Ant: axial and back muscles

DCML Spinothalamic
Sensation Fine touch and vib Crude touch and pain
and temp
Decussation medulla Spinal cord (lower)
Spinal injury Ispilat loss of fine touch Contralat loss of crude
and vibration below touch , pain temp
injury below injury
Sensory cortex injury Contralat loss of fine Contralat loss of crude
touch and vibration touch, pain and temp

E
KEY d. Left mid-thoracic cord.
Additional Khalid:
Information
Presentation: •Ipsilateral UMN weakness below the lesion (severed
corticospinal tract, causing spastic paraparesis, brisk reflexes, extensor
plantars) •Ipsilateral loss of proprioception and vibration (dorsal column
severed) •Contralateral loss of pain and temperature sensation (severed
spinothalamic tract which has crossed over
Causes: Bullet, stab, dart, kick, tumour, disc hernia, cervical spondylosis, MS,
neuroschistosomiasis, myelitis, septic emboli (eg meningococcal).
Imaging: MRI

Reference
Dr Khalid/Rabia Pain & temperature: carried by lateral spinothalamic tract… dicussate to the
opposite side within the spinal cord. Vibration, proprioception: carried by the
medial lemniscal system… decussate at the junction of pons and medulla.
Upper motor neurons in pyramidal tract also decussate close to medulla. So
from this we know that the lesion is on the left side. And since the symptoms
are below the umbillicus it rules out a cervical lesion. Lesion of lumbo sacral
plexus will impair the whole of the lower limb. So the correct answer is D.




Q: 62 A woman has electric pains in her face that start with the jaw and move
upwards. Her corneal reflexes are normal. What is the most likely dx?
a. Atypical face pain
b. Trigeminal neuralgia
c. Tempero-mandibular joint dysfunction
d. GCA

163
CNS-System Wise 1700-by Sush and Team. 2016
Susmita, Asad, Manu, Saima, Zohaib, Savia, Shanu, Mona, Manisha, Sitara, Samreena, Sami and Komal

e. Herpes zoster



Clincher(s)
A
B
1. Stabbing / electric facial pain caused by Trigeminal Neuralgia

(In Q: involving the face and it's muscles , which are mostly supplied by the 5th
CN ; Trigeminal nerve . So this CN is involved in this type of neuralgia. Maxillary
branch of Trigeminal).

2. Both Trigeminal Neuralgia and Herpetic Keratoconjunctivitis (H. Zoster
ophthalmicus) can cause absent corneal reflex.

PRESENTATION: The episodes are sporadic and sudden and often like 'electric
shocks', lasting from a few seconds to several minutes. Pain is unilateral, brief,
stabbing, recurrent in the distribution of CN5. Can be provoked by light touch
to the face, eating, cold winds, or vibrations typically occurs after shaving,
brushing teeth.
Cause is a compression of CN5.
No Investigations: MRI needed to exclude secondar
TREATMENT: Carbamezapine is the first line. Rhizotomy (surgery) may also be
done

C Shooting or electric pain is neurological. But TMJ dysfunction pain will be
mainly associated with movement and once occur may stay few moments.
Corneal reflex may be absent but may be this feature not always present.

D Giant cell arteritis (GCA) is a condition in which medium and large arteries,
usually in the head and neck, become inflamed. It's sometimes called temporal
arteritis because the arteries around the temples are usually affected.

Clincher: temp headache, pain with mov of jaw, old age, pain combing
• Symptoms:
• severe headaches, pain and tenderness over your temples and scalp.
• prominence or tenderness of the blood vessels at your temples.
• fatigue.
• flu-like symptoms.
• weight loss.
• loss of vision or double vision.
• pain in your jaw or tongue when you chew.

Iv: ESR raised, biopsy,

164
CNS-System Wise 1700-by Sush and Team. 2016
Susmita, Asad, Manu, Saima, Zohaib, Savia, Shanu, Mona, Manisha, Sitara, Samreena, Sami and Komal

Immediate with IV prednisolone,

E Herpes zoster should have characteristic vesicles and it will be dermetomal in
distribution

KEY b. Trigeminal neuralgia.
Additional
Information
Reference 461 OHCM
Dr Khalid/Rabia Trigerminal Neuralgia...Facial pains.



Q: 64 64. A 56yo man comes with hx of right sided weakness & left sided visual loss.
Where is the occlusion?
a. Ant meningeal artery
b. Mid meningeal artery
c. Mid cerebral artery
d. Carotid artery
e. Ant cerebral artery
f. Ant communicating artery



Clincher(s)
A Cause hematoma
B
C Middle cerebral artery occlusion: paralysis or weakness of contralateral face
and arm (faciobracheal). Sensory loss of the contralateral face and arm.
(supplies frontal/parietal/temporal)

D Carotid Artery occlusion:
PRESENTATION: Patients may present with TIAs or CVE (cardio vas event)s.
Typical symptoms are contralateral weakness or sensory disturbance,
ipsilateral blindness, and (if the dominant hemisphere is involved) dysphasia,
aphasia or speech apraxia.
• Sudden severe headache
• Dizziness/loss of balance
• Inability to move limb
• Sudden weakness or numbness in face/limbs on one side
• Sudden loss of vision


Carotid bruit may or may not be present
INVESTIGATIONS: For diagnosis: CAROTID ANGIOGRAPHY GOLD STANDARD.

165
CNS-System Wise 1700-by Sush and Team. 2016
Susmita, Asad, Manu, Saima, Zohaib, Savia, Shanu, Mona, Manisha, Sitara, Samreena, Sami and Komal

MR angio and angio CT can also be used.
Echo colour Doppler ultrasonography is the screening method of choice

TREATMENT: Medical: Antiplatelets, Anti HTN, Statins
Surgery: Carotid endartarectomy (or stenting). Symptomatic patients
with greater than 50% stenosis and healthy, asymptomatic patients
with greater than 60% stenosis warrant consideration for carotid
endarterectomy


E Anterior cerebral artery occlusion: paralysis or weakness of the contralateral
foot and leg. Sensory loss at the contralateral foot and leg. (supplies frontal)
.

KEY d. Carotid artery.
Additional
Information
Reference
Dr Khalid/Rabia


Q: 76 A 65yo HTN man presents with lower abdominal pain and back pain. An
expansive abdominal mass is palpated lateral and superior to the umbilicus.
What is the single most discriminating inv?
a. Laparascopy
b. KUB XR
c. Pelvic US
d. Rectal exam
e. Abdominal US


Clincher(s)
A
B
C
D
E The diagnosis is Abdominal aortic aneurism. Points in favour of AAA are i)
hypertension ii) abdominal pain iii) back pain iv) expansile abdominal mass
lateral and superior to the umbilicus.
KEY E. Abdominal US.
Additional
Information
Reference
Dr Khalid/Rabia RISK Factors include :
• Family Hx , tobacco smoking is an important factor.
• Male sex.

166
CNS-System Wise 1700-by Sush and Team. 2016
Susmita, Asad, Manu, Saima, Zohaib, Savia, Shanu, Mona, Manisha, Sitara, Samreena, Sami and Komal

• Increasing age.
• Hypertension.
• Chronic obstructive pulmonary disease.
• Hyperlipidaemia

UNRUPTURED AAA is commonly asymptomatic and is an accidental finding.

Ruptured AAA may present with:
• Pain in the abdomen, back or loin - the pain may be sudden and severe.
• Syncope, shock or collapse:
• The degree of shock varies according to the site of rupture and whether
it is contained - eg, rupture into the peritoneal cavity is usually
dramatic, with death before reaching hospital; whereas rupture into
the retroperitoneal space may be contained initially by a temporary
seal forming.
Inv:
Ultrasound is simple and cheap; it can assess the aorta to an accuracy of 3 mm.
It is used for initial assessment and follow-up.

SCANS :
CT Scan provides more anatomical details - eg, it can show the visceral
arteries, mural thrombus, the 'crescent sign' (blood within the thrombus,
which may predict imminent rupture) and para-aortic inflammation. CT with
contrast can show rupture of the aneurysm.

MRI angiography may be used.

Rx: If size exceeds 5.5cm, we go for surgery.


Q: 95 A 56yo lady has developed severe right sided headache which worsens
whenever she comes to bright light since the last 4 days. She feels nauseated,
but doesn’t vomit. What is the most likely dx?
a. SAH
b. Brain tumor
c. Migraine
d. Cluster headache
e. Subdural headache


Clincher(s)
A A subarachnoid haemorrhage is an uncommon type of stroke caused by
bleeding on the surface of the brain. It is a very serious condition and can be
fatal.
The main symptom of a subarachnoid haemorrhage is a sudden and very
severe headache that is often described as a blinding pain, unlike anything

167
CNS-System Wise 1700-by Sush and Team. 2016
Susmita, Asad, Manu, Saima, Zohaib, Savia, Shanu, Mona, Manisha, Sitara, Samreena, Sami and Komal

experienced before.
Other symptoms can include:
• a stiff neck
• being sick (vomit?)
• sensitivity to light
• blurred or double vision
• seizures (fits) or loss of consciousness (NHS)

B • Brain Tumour headache:
• Steady persistent pain that is worse upon waking in the morning and
gets better within a few hours.
• Persistent, non-migraine headache.
• May be accompanied by vomiting.
• May or may not be throbbing, depending on the location of the tumor.

C It is migraine without aura. There is presence of trigger (bright light).

Criteria of migraine without aura:
≥5 headaches lasting 4-72 hours + nausea/vomiting (or photo/phono-phobia)

+ any 2 of: i) unilateral ii) pulsating iii) worsen by routine activity

[OHCM, 9th edition, page-462].
D Cluster headaches begin suddenly and without warning. The pain is very
severe and is often described as a sharp, burning or piercing sensation on one
side of the head. The pain is typically felt around the eye, temple and
sometimes face, and typically recurs on the same side for each attack.

Often people feel restless and agitated during an attack because the pain is so
intense, and may react by rocking, pacing or banging their head against the
wall.

They commonly also have at least one of the following associated symptoms:
• a red and watering eye

• drooping and swelling of one eyelid


• a smaller pupil in one eye
• a sweaty face
• a blocked or runny nostril
• a red ear

These attacks generally last between 15 minutes and three hours, and typically
occur between one and eight times a day. (Ref: NHS)

168
CNS-System Wise 1700-by Sush and Team. 2016
Susmita, Asad, Manu, Saima, Zohaib, Savia, Shanu, Mona, Manisha, Sitara, Samreena, Sami and Komal

Typically, a number of attacks occur over several weeks - a bout (cluster) of
attacks. (night and men) The headaches then usually go for weeks, months or
years until a next cluster of headaches develops. (ref: pt info)

E The symptoms of a subdural haematoma can develop soon after a severe head
injury, or gradually over days or weeks after a more minor head injury.
Sometimes you may not remember hitting your head at all.
Symptoms of a subdural haematoma can include:
• a headache that keeps getting worse
• feeling and being sick
• confusion
• personality changes, such as being unusually aggressive or having rapid
mood swings
• feeling drowsy and finding it difficult to keep your eyes open
• speech problems, such as slurred speech
• problems with your vision, such as double vision
• paralysis (loss of movement) on one side of the body
• problems walking and frequent falls
• seizures (fits)
• loss of consciousness (NHS)


Key C. Migraine.
Additional
Information
Reference
Dr Khalid/Rabia It should be noted that as a general rule 5-HT receptor agonists are used in the
acute treatment of migraine whilst 5-HT receptor antagonists are used in
prophylaxis. NICE produced guidelines in 2012 on the management of
headache, including migraines. (update the below information: e.g. from
other Q/A)

Acute treatment
• first-line: offer combination therapy with an oral triptan and an NSAID,
or an oral triptan and paracetamol
• for young people aged 12-17 years consider a nasal triptan in
preference to an oral triptan
• if the above measures are not effective or not tolerated offer a non-
oral preparation of metoclopramide* or prochlorperazine and consider
adding a non-oral NSAID or triptan

Prophylaxis
• prophylaxis should be given if patients are experiencing 2 or more
attacks per month. Modern treatment is effective in about 60% f
patients.
• NICE advise either topiramate or propranolol 'according to the person's

169
CNS-System Wise 1700-by Sush and Team. 2016
Susmita, Asad, Manu, Saima, Zohaib, Savia, Shanu, Mona, Manisha, Sitara, Samreena, Sami and Komal

o
• preference, comorbidities and risk of adverse events'. Propranolol
should be used in preference to topiramate in women of child bearing
age as it may be teratogenic and it can reduce the effectiveness of
hormonal contraceptives
• if these measures fail NICE recommend 'a course of up to 10 sessions of
acupuncture over 5-8 weeks' or gabapentin
• NICE recommend: 'Advise people with migraine that riboflavin (400 mg
once a day) may be effective in reducing migraine frequency and
intensity for some people'
• for women with predictable menstrual migraine treatment NICE
recommend either frovatriptan (2.5 mg twice a day) or zolmitriptan
(2.5 mg twice or three times a day) as a type of 'mini-prophylaxis'
• pizotifen is no longer recommend. Adverse effects such as weight gain
& drowsiness are common

*caution should be exercised with young patients as acute dystonic reactions


may develop


Q: 109 A 33yo male involved in a street fight presents with bruises and deformity in
the upper part of his leg. XR shows fx of the neck of fibula. What is the single
most associated nerve injury?
a. Sciatic nerve
b. Gluteal nerve
c. Musculocutaneous nerve
d. Lateral peroneal nerve
e. Tibial nerve
f. Femoral nerve


Clincher(s)
A
B
C
D Its Common peroneal nerve injury (due to fibula fracture) leading to foot drop.
Peroneal nerve injury is called foot drop/drop foot. Peroneal nerve is also the
‘fibular nerve’. (Johns Hopkins)

Common peroneal (from the schiatic nerve) is bifurcated to superficial (or Lat
PN) and deep
These nerves provide sensation to the anterior (front) and lateral (side) parts
of the legs and to the top of the feet. They innervate muscles in the legs which
lift the ankle and toes upward (dorsi flexion). (JH)
Lateral peroneal nerve is other name of superficial peroneal nerve. It causes

170
CNS-System Wise 1700-by Sush and Team. 2016
Susmita, Asad, Manu, Saima, Zohaib, Savia, Shanu, Mona, Manisha, Sitara, Samreena, Sami and Komal

foot eversion. (see picture at bottom)




E
KEY Ans. is D. Lateral peroneal nerve.
Additional
Information
Reference
Dr Khalid/Rabia

171
CNS-System Wise 1700-by Sush and Team. 2016
Susmita, Asad, Manu, Saima, Zohaib, Savia, Shanu, Mona, Manisha, Sitara, Samreena, Sami and Komal



Q: 145 A 53yo woman presented with pain in the eye, blurry vision and clumsiness for
3 months. She has a hx of difficulty in swallowing and weakness in her right
upper limb 2y ago. What is the inv of choice? (Multiple nerve involvement=
MS)
a. CSF analysis
b. EEG
c. EMG
d. MRI brain
e. Visual evoked response test


Clincher(s)
A
B

172
CNS-System Wise 1700-by Sush and Team. 2016
Susmita, Asad, Manu, Saima, Zohaib, Savia, Shanu, Mona, Manisha, Sitara, Samreena, Sami and Komal

C
D (NHS) Diagnosis is multiple sclerosis. When MS damages the myelin coating
around the nerve fibres that carry messages to and from your brain,
symptoms can occur in any part of your body.
There are many different symptoms of MS and they affect each person
differently. Some of the most common symptoms include:
• extreme tiredness (fatigue)-common
• numbness and tingling
• Visual problem: e.g. blurry/loss of vision/colour blindness/ pain in both
eyes/ nystagmus- invol eye movements(In around one in five cases of
MS, the first noticeable symptom due to optic neuritis : in ONE of the
eyes. )
• problems with mobility and balance (ataxia/vertigo/tremor)
• muscle weakness and tightness / spasticity (stiff and resistant to
movement)
• half of people experience pain (neuropathic- stabbing/burning or
musculosketetal)
• Cog dysfunction-problem with thinking, learning, planning
• depression/anxiety
• erectile dysfunction
• urge incontinence
• dysphagia
• bowel constipation/incontinece


Cause: Autoimmnue This means your immune system mistakes the
myelin for a foreign substance and attacks it.

The myelin becomes inflamed in small patches (called plaques or
lesions), which can be seen on an MRI scan.

Rx: NICE: http://www.nice.org.uk/guidance/CG8
The NICE guideline states that if you have MS, you should have access
to a specialist neurological rehabilitation team. Care for people with MS
using a coordinated multidisciplinary approach.


E
KEY D. MRI brain.
Additional
Information
Reference
Dr Khalid/Rabia Multiple sclerosis: Discrete plaques of demyelination occur at multiple CNS
sites. Early exposure to sunlight/vit. D is important, and vit. D status relates to
prevention of MS,
PRESENTATION: Usually monosymptomatic: unilateral optic neuritis (pain on

173
CNS-System Wise 1700-by Sush and Team. 2016
Susmita, Asad, Manu, Saima, Zohaib, Savia, Shanu, Mona, Manisha, Sitara, Samreena, Sami and Komal

eye movement and rapid central vision); numbness or tingling in the limbs; leg
weakness; brainstem or cerebellar symptoms (eg diplopia, ataxia). The disease
has a relapsing - remitting course.


INVESTIGATIONS: This is clinical, as no test is pathognomonic. MRI is sensitive
but not specific for plaque detection.
CSF: Oligoclonal bands of IgG on electrophoresis suggest CNS inflammation but
does not confirm MS.. Delayed visual, auditory, and somatosensory evoked
potentials.
MANAGEMENT: Stress free life. Give vit. D to achieve serum 25(OH)D levels of
50nmol/L
Methylprednisolone shortens relapse doesnt alter overall prognosis.
Beta interferon reduces replaces by 30% but does not reduce overall disability
Monoclonal antibodies: Alemtuzumab and natalizumab reduce relapses by
68%. (for autoimmune)
Glatiramer in secondary progressive
Azathioprine in relapsing-remiting
Palliation: Spasticity: Baclofen, diazepam, dantrolene or tizanidine.
Tremor: Botulinum toxin
Urgency/frequency: If post-micturition residual urine >100mL, teach
intermittent self-catheterization; if <100mL, try tolterodine



Q: 153 A 44yo pt comes with right hemiparesis. Exam: left sided ptosis and left dilated
pupil. Where is the lesion?
a. Cerebral infarct
b. Cerebellar infarct
c. Medulla oblongata
d. Pons
e. Midbrain

174
CNS-System Wise 1700-by Sush and Team. 2016
Susmita, Asad, Manu, Saima, Zohaib, Savia, Shanu, Mona, Manisha, Sitara, Samreena, Sami and Komal



Clincher(s)
A
B
C Common syndromes related to the medulla and pons are Lateral medullary
syndrome and Lateral pontine syndrome.

Same picture along with sensory picture as it has the sensory tracts

They both present with
1.Ipsilateral horners syndrome (ptosis, anhydrosis myosis,: PAM)
2. Ipsilateral cerebellar ataxia and
3. Contralateral loss of pain and temp. (STT)

The two conditions are differentiated by the fact that the Medulla also
presents with dysphagia, dysarthria and dysphonia, whereas,

the Pons has ipsilateral facial nerve involvement (LMN facial palsy, loss of taste
ant. 2/3 tongue & reduced lacrimation and salivation).

Lateral medullary syndrome = Posterior inferior cerebellar artery (PICA).

Lateral pontine syndrome = Anterior inferior cerebellar artery (AICA) (Ref: FB)


D See above
E Midbrain. The occulomotor nerve nucleus is located in the midbrain. When the
brainstem is affected (occulomotor nerve damage) it causes ipsilateral cranial
nerve injury and contralateral hemiparesis.


Weber’s syndrome (superior alternating hemiplegia)

• Ipsilateral oculomotor nerve palsy with contralateral hemiplegia,

• due to infarction of one-half of the midbrain,

• after occlusion of the (paramedian branches of the basilar or)
posterior cerebral arteries. (Dr Khalid)

KEY E. Midbrain.

Additional
Information
Reference

175
CNS-System Wise 1700-by Sush and Team. 2016
Susmita, Asad, Manu, Saima, Zohaib, Savia, Shanu, Mona, Manisha, Sitara, Samreena, Sami and Komal

Dr Khalid/Rabia Ptosis + miosis = horners syndrome Ptosis + mydriasis (pupil dilatation)=
oculomotor nerve palsy.

CN3 nucleus lies in the midbrain.
Fore brain: CN 1,2
Mid brain: CN 3,4
Pons: CN 5,6,7,8
Medulla: CN 9,10, 11? 12





Q: 164 A 23yo male has a tonic clonic seizure whilst at college. His GCS (Glasgow
comma scale) is 12, BP=120/77mmHg,
HR=99bpm. What is the most appropriate inv for his condition?
a. CT
b. MRI
c. Serum blood glucose
d. Serum drug levels


Clincher(s)
A in seizure CT scan is not the appropriate investigation except with the h/o
trauma or when suspecting Hhg.

EEG is what is done for seizures not ct


B
C Should be blood glucose, then drug level and ct /mri. Question is not asking
definitive so it's blood glucose.

Below 12 GCS= CT done, after 2 hrs of admission if GCS is 15= CT (for other
cases)

we need to rule out hypoglycaemia first ..it can cause drop in GCS level aswell.
it can also cause seizure.

This case will be treated as status epilepticus. Normally status occurs in a


patient with known epilepsy but here no such history is given. Even in a person
who presents with his first seizure we need to look for hypoglycemia first
before going onto later tests.

[it is also possible that he may have taken drug, even though first we have to
do serum glucose as its presence can be very easily managed and it needs

176
CNS-System Wise 1700-by Sush and Team. 2016
Susmita, Asad, Manu, Saima, Zohaib, Savia, Shanu, Mona, Manisha, Sitara, Samreena, Sami and Komal

urgent management to save life. If it is excluded then we can look for other
causes which may be not fatal in short time as hypoglycaemia].
D
E
KEY C. Serum blood glucose
Additional • The tonic-clonic seizure (old- grand mal) is what most people think of
Information when they think of a convulsive seizure.
• they combine the characteristics of tonic seizures (stiffen) and clonic
seizures (jerking).
• A person loses consciousness, muscles stiffen, and jerking movements
are seen.
• These types of seizures usually last 1 to 3 minutes and take much
longer for a person to recover.
• A tonic-clonic seizure lasting more than 5 minutes is a medical
emergency.

Epilepsy Causes (NHS):

Idiopathic and symptomatic

Symptomatic:

• cerebrovascular disease (problems with the blood vessels that supply


the brain) – such as a stroke or subarachnoid haemorrhage
• brain tumours
• severe head injuries
• drug abuse and alcohol misuse
• infections that can damage the brain – such as meningitis
• problems during birth that cause a baby to be deprived of oxygen –
such as the umbilical cord getting twisted or compressed during labour
• some parts of the brain not developing properly
Triggers:
• stress
• lack of sleep
• drinking alcohol
• some medications and illegal drugs
• in women, monthly periods
• flashing lights (this is an uncommon trigger that affects only 5% of
people with epilepsy, and is known as photosensitive epilepsy)
Inv:
EEG (An EEG test can detect unusual brain activity associated with epilepsy by

177
CNS-System Wise 1700-by Sush and Team. 2016
Susmita, Asad, Manu, Saima, Zohaib, Savia, Shanu, Mona, Manisha, Sitara, Samreena, Sami and Komal

measuring the electrical activity of your brain through electrodes placed on
your scalp.)

MRI (It can be useful in cases of suspected epilepsy because it can often detect
possible causes of the condition, such as defects in the structure of your brain
or the presence of a brain tumour.)

RX:
Treatment for epilepsy is used to control seizures, although not everyone with
the condition will need to be treated. Anti-epileptic drugs (AEDs) are usually
the first choice of treatment for epilepsy. About 70% of people with the
condition are able to control their seizures with AEDs. (see epi drugs)

Reference
Dr Khalid/Rabia Status epilepticus: (Sush: Is tonic clonic status epilepticus? If not: why this
note )

This means seizures lasting for >30min, or repeated seizures without
intervening consciousness.
Also consider eclampsia in mind if the patient is female and the abdomen is
distended.

Investigations
• Bedside glucose, the following tests can be done once treatment has started:
lab glucose,
ABG, U&E, Ca2+, FBC, ECG.
• Consider anticonvulsant levels, toxicology screen, LP, culture blood and
urine, EEG,
CT, carbon monoxide level. • Pulse oximetry, cardiac monitor.

Treatment: Try to control seizure in less than 20mins as there could be
permanent brain damage.
1. IV lorazepam:0.1mg/kg. Repeat if no response in 10mins. Be careful about
respiratory depression. If there is no IV access give PR Diazepam.
2.Buccal midazolam: Alternative oral route. squirt half the volume between
the lower gum and the cheek on each side.
3.Phenytoin infusion: 15–20mg/kg IVI (roughly 1g if 60kg, and 1 . 5g if 80kg;
max 2g. 2nd line! Dont use if bradycardic or hypotensiv. ECG monitoring is
recommended.
Diazepam infusion: eg 100mg in 500mL of 5% dextrose. It is most unusual for
seizures to remain unresponsive following this. If they do, allow the idea to
pass through your mind that they could be pseudoseizures, particularly if there
are odd features (pelvic thrusts; resisting attempts to open lids and your

178
CNS-System Wise 1700-by Sush and Team. 2016
Susmita, Asad, Manu, Saima, Zohaib, Savia, Shanu, Mona, Manisha, Sitara, Samreena, Sami and Komal

attempts to do passive movements; arms and legs flailing around).
4. Dexamethasone: 10mg IV if vasculitis/cerebral oedema (tumour) possible.
5. General anaesthesia: For refractory status: get anaesthetist/ICU involved
early




Q: to do from here 1542. A 34yo female presented with vomiting preceded by an occipital
headache of acute onset. Exam: conscious and alert with photophobia
but no neck stiffness. CT: normal. What is the most
appropriate further management?
a. CT brain with contrast
b. Repeat CT brain in 24h
c. CSF exam
d. Cerebral angio
e. MRI brain

179
CNS-System Wise 1700-by Sush and Team. 2016
Susmita, Asad, Manu, Saima, Zohaib, Savia, Shanu, Mona, Manisha, Sitara, Samreena, Sami and Komal


Clincher(s)
A
B
C After 12 hours lumber puncture is done and if ct finding and csf analysis both
are negative then subarachnoid hemorrhage is ruled out.
D
E
KEY c
Additional
Information
Reference
Davidson page 1190


Q: . A 76yo woman presents with deep stroke 6h ago. What would the
immediate tx be?
aAspirin75mg
b.Aspirin300mg
c.Streptokinase
d.IVheparin
e. Dipyridamole 200mg

Clincher(s)
A
B
C
D
E
KEY
Additional Fist 300mg and then 75mg
Information
For prophylyxis:

clopidegrol is a monotherapy.

Asprin and dipyridamole can be given together


Reference
Dr Khalid/Rabia


Q: 1553. A 50 y/o man with a known history of stroke is unable to get out of
his house because he can’t find where the door is. He refuses help from
his wife and says he is not blind. What is the single most likely defect?
a. Paracentral scotoma

180
CNS-System Wise 1700-by Sush and Team. 2016
Susmita, Asad, Manu, Saima, Zohaib, Savia, Shanu, Mona, Manisha, Sitara, Samreena, Sami and Komal

b. Tunnel vision
c. Total blindness
d. Central scotoma
e. Cortical blindness


Clincher(s)
A
B Retinitis pigmentosa
C
D Maculer degeneration or maculer edema
E Cortex is concerned for perception of blindness (cannot perceive he is blind)
KEY
Additional
Information Tunnel vision: peripheral vision is lost
Reference
Dr Khalid/Rabia


Q: 1645. A 32yo woman has had 3 episodes of slurred speech and 2
episodes of transient weakness of both legs in the past 5yrs. Each
episode has resolved in 3m. What is the SINGLE most likely dx a.
Meningioma
b. Migraine
c. Multiple sclerosis
d. Stroke
e. Transient ischaemic attack


Clincher(s)
A
B
C Discrete patch of demylination occur at multiple site of cns due to t cell
mediated immune response. Demylination heals poorly, causing remitting and
relapsing pattern. Prolonged demylination causes exonal loss and clinically
progressive disease.
D
E The sudden onset of focal cns symtoms due to temporary occlusion of part of
cerebral circulation,usually by emboli.
KEY
Additional
Information
Reference
Dr Khalid/Rabia

181
CNS-System Wise 1700-by Sush and Team. 2016
Susmita, Asad, Manu, Saima, Zohaib, Savia, Shanu, Mona, Manisha, Sitara, Samreena, Sami and Komal


Q: 1647. A 38yo man with longstanding alcohol dependence has vertigo
and a tremor every morning.
What is the SINGLE most likely dx?
a. Anxiety
b. Benign positional vertigo
c. Cerebellar degeneration
d. Optic neuritis
e. Temporal lobe epilepsy


Clincher(s)
A
B Vertigo associated with change in the position of head.
C
D Pain on eye movement and decrease in vision.
E Ohcm page 495, complex motor phenomenon,but with impared awareness
and no recollection afterwards
KEY
Additional
Information
Reference
Dr Khalid/Rabia

Q:

Clincher(s)
A
B
C
D
E
KEY
Additional
Information
Reference
Dr Khalid/Rabia

Q:

Clincher(s)
A
B
C
D

182
CNS-System Wise 1700-by Sush and Team. 2016
Susmita, Asad, Manu, Saima, Zohaib, Savia, Shanu, Mona, Manisha, Sitara, Samreena, Sami and Komal

E
KEY
Additional
Information
Reference
Dr Khalid/Rabia


Q:

Clincher(s)
A
B
C
D
E
KEY
Additional
Information
Reference
Dr Khalid/Rabia


Q:

Clincher(s)
A
B
C
D
E
KEY
Additional
Information
Reference
Dr Khalid/Rabia


Q:

Clincher(s)
A
B
C
D

183
CNS-System Wise 1700-by Sush and Team. 2016
Susmita, Asad, Manu, Saima, Zohaib, Savia, Shanu, Mona, Manisha, Sitara, Samreena, Sami and Komal

E
KEY
Additional
Information
Reference
Dr Khalid/Rabia



Q:438 438. A 44yo man comes with hx of early morning headaches and vomiting. CT
brain shows ring enhancing lesions. What is the single most appropriate
option?
a. CMV
b. Streptococcus
c. Toxoplasmosis
d. NHL Non Hodgkins lymphoma
e. Pneumocystis jerovii


Clincher(s) RING ENHANCING LESION
A CMV
B STREPTOCOCCUS
C TOXOPLASMOSIS
D NHL
E PNEUMOCYSTIS JEROVII
KEY C
Additional DR MAGIC L- RING ENHANCING CEREBRAL LESION
Information
In some institutions, "L" is added to the mnemonic for lymphoma.

• D: demyelinating disease
• R: radiation necrosis or resolving haematoma

• M: metastasis
• A: abscess
• G: glioblastoma
• I: infarct (subacute phase)
• C: contusion

• L: lymphoma


Reference
Dr Khalid/Rabia Causes of ring enhancing lesions on CT brain:
-Brain abscess
-Primary or secondary tumour

184
CNS-System Wise 1700-by Sush and Team. 2016
Susmita, Asad, Manu, Saima, Zohaib, Savia, Shanu, Mona, Manisha, Sitara, Samreena, Sami and Komal

-CNS lymphoma
-CNS toxoplasmosis
-Nocardia infection.
Out of the options, toxoplasmosis is the right answer, and it is commonly
found in HIV patients. TREATMENT with pyrimethamine/sulfadiazine and
folinic acid. OR clindamycin if intolerant FOR 4-6 WEEKS.
If immunocompromised, PROPHYLAXIS with Trimethoprim+sulfamethoxazole.



Q:449 A young pt is complaining of vertigo whenever she moves sideways on the bed
while lying supine. What would be the most appropriate next step?
a. Head roll test
b. Reassure
c. Advice on posture
d. Carotid Doppler
e. CT


Clincher(s)
A Head roll test
B Reassure
C Advice on Posture
D Carotid Doppler
E CT
KEY A
Additional Benign paroxysmal positional vertigo (BPPV)
Information
a) This is a clinical presentation where patients experience frequent but brief
episodes of vertigo, usually of less than three minutes duration. Each episode
is triggered by a specific movement.

b) BPPV is the most common cause of vertigo on neck extension, such as


looking upwards, and has often been misdiagnosed in the past as
vertebrobasilar insufficiency. However, in BPPV there are no other symptoms
of interruption to the posterior circulation blood supply.

c) Vertigo also occurs on first lying down in bed at night, first getting out of
bed in the morning, bending forward and turning in bed, usually to one side
only.

d) BPPV is caused by an accumulation of otoliths within a semicircular canal,


and the pathophysiological mechanism of generation of symptoms is
considered to be canalolithiasis.

This otoconial debris is free to move within the lumen of the canal, an effect

185
CNS-System Wise 1700-by Sush and Team. 2016
Susmita, Asad, Manu, Saima, Zohaib, Savia, Shanu, Mona, Manisha, Sitara, Samreena, Sami and Komal

that occurs during the postural movements described above.

The movement of otoliths causes fluid movement and is perceived as vertigo.

e) Fifty percent of cases are of unknown aetiology but known causes include
infection, vascular problems, and trauma.
Reference
Dr Khalid/Rabia Dx? Benign Paroxysmal Positional Vertigo (BPPV)- Most common cause of
vertigo. Vertigo triggered by change in head position. Might be accompanied
by nausea and nystagmus. Less commonly, vomiting and syncope.
Diagnosis: Dix-Hallpike and Head roll test.
Management: Epley and Semont Maneuver


Q: 452 A 50yo woman presents following a fall. She reports pain and weakness in her
hands for several months, stiff legs, swallowing difficulties, and has bilateral
wasting of the small muscles of her hands. Reflexes in the upper limbs are
absent. Tongue fasciculations are present and both legs show increased tone,
pyramidal weakness and hyper-reflexia with extensor plantars. Pain and
temp sensation are impaired in the upper limbs. What is the most likely dx?
a. MS
b. MND
c. Syringobulbia
d. Syringomyelia
e. Myasthenia gravis

Clincher(s)
A MS- Usually monosymptomatic: unilateral optic neuritis (pain on eye
movement and rapid central vision); numbness or tingling in the limbs; leg
weakness;brainstem or cerebellar symptoms (eg diplopia, ataxia). Symptoms
may worsen with heat (eg hot bath) or exercise. Rarely polysymptomatic.
B MND- Think of MND in those >40yrs with stumbling spastic gait, foot-drop }
proximal myopathy, weak grip (door-handles don’t turn) and shoulder
abduction (hair-washing is hard), or aspiration pneumonia. Look for UMN
signs: spasticity, brisk refl exes, plantars; and LMN signs: wasting, fascic
ulation of tongue, abdomen, back, thigh. Is speech or swallowing aff ected
(bulbar signs)? Fasciculation is not enough to diagnose an LMN lesion: look for
weakness too. Frontotemporal dementia occurs in ~25%.
C SYRINGOBULBIA- ; Syringo bulbia (brain stem involvement):nyst agmus, tongue
atrophy,
D SYRINGOMYELIA- Cardinal signs: Dissociated sensory loss (absent pain and T°
sensation, with preserved
light touch, vibration, and joint-position sense) due to pressure from the syrinx
on the decussating antero lateral pathway (fi g 1) in a root dis tribution
reflecting the location of the syrinx (eg for typical cervical syrinx then sensory
loss is over trunk and arms); wasting/weakness of hands } claw-hand (then

186
CNS-System Wise 1700-by Sush and Team. 2016
Susmita, Asad, Manu, Saima, Zohaib, Savia, Shanu, Mona, Manisha, Sitara, Samreena, Sami and Komal

arms sho ulders respirat ory muscles). Anterior horn cells are also
vulnerable.Other signs: Horner’s syndrome; UMN leg signs; body asymm etry,
limb hemi-hypertrophy,or uni lat eral odo- or chiromegaly (enlarged hand or
foot), perhaps from release of trophic factors via anterior horn cells; Syringo
bulbia (brain stem involvement):nyst agmus, tongue atrophy,dysphagia,
pharyngeal/palatal weak ness,Vth nerve sensory loss. Charcot’s
(neuropathic)joints: Increased range of movement (from lost joint pro prio
ception), destroys the joint, which becomes swollen
and mobile (see fi g 2, p205).380 Causes: tabes dorsalis (eg knee), diabetic
neuropathy, paraplegia (eg hips),381 syringomyelia (shoulder,wrist),382
leprosy, spinal osteolysis/ cord atrophy (systemic sclerosis).382
E MYASTHENIA GRAVIS-Increasing muscular fatigue. Muscle groups aff ected, in
order: extraocular;
bulbar (swallowing, chewing); face; neck; limb girdle; trunk. Look for: ptosis,
diplopia, myasthenic snarl on smiling, ‘peek sign’.1 On counting to 50, the
voice fades (dysphonia is a rare presentation). Tendon refl exes are normal.
Weakness is
worsened by: pregnancy, K+DECREASES , infection, over-treatment, change of
climate, emotion, exercise, gentamicin, opiates, tetracycline, quinine,
procainamide, BETA blockers.
KEY C
Additional
Information
Reference
Dr Khalid/Rabia In MS, there are characteristic relapse and remission which is absent here.
MND is purely motor, there is no sensory deficit;

In myasthenia gravis there is muscular weakness without atrophy.

Syringomyelia is a condition in which there is fluid-filled tubular cyst (syrinx)
within the central, usually cervical, spinal cord. The syrinx can elongate,
enlarge and expand into the grey and white matter and, as it does so, it
compresses the nervous tissue of the corticospinal and spinothalamic tracts
and the anterior horn cells. This leads to various neurological symptoms and
signs, including pain, paralysis, stiffness and weakness in the back, shoulders
and extremities. It may also cause loss of extreme temperature sensation,
particularly in the hands, and a cape-like loss of pain and temperature
sensation along the back and arms.
** If the syrinx extends into the brainstem, syringobulbia results. This may
affect one or more cranial nerves, resulting in facial palsies. Sensory and motor
nerve pathways may be affected by interruption and/or compression of
nerves.


Q:463 A young adult presents to the ED after a motorcycle crash. The pt has bruises
around the left orbital area. GCS=13, examination notes alcoholic breath.

187
CNS-System Wise 1700-by Sush and Team. 2016
Susmita, Asad, Manu, Saima, Zohaib, Savia, Shanu, Mona, Manisha, Sitara, Samreena, Sami and Komal

Shortly afterwards, his GCS drops to 7.
What is the single most important initial assessment test?
a. MRI brain
b. CT brain
c. CXR
d. CT angio brain
e. Head XR

Clincher(s) GCS DROP OF 13 TO 7
A MRI BRAIN
B CT BRAIN
C CXR- NOT INDICATED FOR HEAD INJURY
D CT ANGIO BRAIN
E HEAD XR
KEY B
Additional INDICATION OF CT
Information
GCS <13 at initial assessment
• GCS <15 at 2h post injury
• Open or depressed skull or basal
skull fracture
• Post-traumatic fi t
• Focal CNS defi cit
• >1 episode of vomiting
• Retrograde amnesia >.h
Reference OHCS 729
Dr Khalid/Rabia


Q:475 A 32yo woman presents to the ED with headache and vomiting. She was
decorating her ceiling that morning when the headache began, felt mainly
occipital with neck pain. Some 2hs later she felt nauseated, vomited and was
unable to walk. She also noticed that her voice had altered. She
takes no reg meds and has no significant PMH. Exam: acuity, field and fundi
are normal. She has upbeat nystagmus in all directions of gaze with normal
facial muscles and tongue movements.
Her uvulas deviated to the right and her speech is slurred. Limb exam: left arm
past-pointing and dysdiadochokinesis with reduced pin prick sensation in her
right arm and leg. Although power is Normal, she can’t talk as she feels too
unsteady. Where is the most likely site of lesion?
a. Right medial medulla
b. Left medial pons
c. Left cerebellar hemisphere
d. Right lateral medulla
e. Left lateral medulla

188
CNS-System Wise 1700-by Sush and Team. 2016
Susmita, Asad, Manu, Saima, Zohaib, Savia, Shanu, Mona, Manisha, Sitara, Samreena, Sami and Komal

Clincher(s)
A Right medial medulla
B Left medial pons
C Left cerebellar hemisphere
D Right lateral medulla
E Left lateral medulla
KEY E
Additional
Information
Reference
Dr Khalid/Rabia Lateral medullary syndrome affects:
-Contralateral spinothalamic tract (loss of pain and temperature on the
opposite side of the body)
-Ipsilateral Sympathetic tract- Horner’s syndrome.
-Ipsilateral Spinal trigeminal nucleus (loss of pain,temperature and corneal
reflex on same side of the face)
-Nucleus ambigous- Dysphagia and Dysarthria
-Inferior cerebellar peduncle- Ataxia
Ipsilateral Cranial nerves- IX, X and XI (dysphagia, loss of gag reflex, palate
paralysis)
Cause- Occlusion of PICA (posterior inferior cerebellar artery)
Medial medullary syndrome affects:
Contralateral corticospinal tract/pyramids- weakness of arms and legs
opposite side.
Contralateral Medial lemniscus/dorsal column- loss of proprioception and
vibration.
Ipsilateral hypoglossal nerve- weakness of tongue on the same side.
[Hypoglossal nerve affection manifests as protrusion of the tongue to the side
of the weakness while at rest, it deviates to the contralateral side)
Cause- Occlusion of Anterior spinal artery

Q:502 For a pt presenting with Parkinson’s disease which of the following drugs is
most useful in the management of the tremor?
a. Apomorphine
b. Cabergoline
c. Selegiline
d. Amantadine
e. Benzhexol

Clincher(s)
A Apomorphine
B Cabergoline
C Selegiline
D Amantadine
E Benzhexol
KEY E

189
CNS-System Wise 1700-by Sush and Team. 2016
Susmita, Asad, Manu, Saima, Zohaib, Savia, Shanu, Mona, Manisha, Sitara, Samreena, Sami and Komal

Additional
Information
Reference
Dr Khalid/Rabia Antimuscarinics
• block cholinergic receptors
• now used more to treat drug-induced parkinsonism rather than
idiopathic Parkinson's disease
• help tremor and rigidity
• e.g. procyclidine, benzotropine, trihexyphenidyl (benzhexol)



Q:503 A 26yo woman has become aware of increasing right sided hearing deficiency
since her recent pregnancy. Her eardrums are normal. Her hearing tests show:
BC-normal. Weber test lateralizes to the right ear. What is the single most
likely dx?
• a. Encephalopathy
• b. Functional hearing loss
• c. Tympano-sclerosis
• d. Otosclerosis
• e. Sensorineural deafness



Clincher(s)
A Encephalopathy
B Functional hearing loss
C Tympano-sclerosis
D Otosclerosis
E Sensorineural deafness
KEY D
Additional
Information
Reference
Dr Khalid/Rabia key is D. Otosclerosis. [There are no features of encephalopathy. As Weber
test is lateralized it is unlikely to be functional hearing loss. In tympanosclerosis
ear drum becomes chalky white. So as the ear drum is normal it is not
tympanosclerosis. Weber test is lateralized to right and deafness is also on the
right. So it not sensorineural deafness but conductive deafness which makes
otosclerosis as the most likely diagnosis.
Rinne's test
· air conduction (AC) is normally better than bone conduction (BC)
· if BC > AC then conductive deafness

Weber's test
· in unilateral sensorineural deafness, sound is localised to the unaffected side

190
CNS-System Wise 1700-by Sush and Team. 2016
Susmita, Asad, Manu, Saima, Zohaib, Savia, Shanu, Mona, Manisha, Sitara, Samreena, Sami and Komal

· in unilateral conductive deafness, sound is localised to the affected side



Q:537 537. A woman presented with blurred vision and intermittent clumsiness for
3m. Reflexes are brisk in her arm and optic disc is pale. What is the single most
appropriate test to confirm dx?
a. CSF analysis
b. CT
c. MRI
d. EEG
e. EMG

Clincher(s)
A CSF analysis
B CT
C MRI
D EEG
E EMG
KEY C
Additional
Information
Reference
Dr Khalid/Rabia multiple sclerosis, investigation of choice is gadolinium enhanced mri.

Multiple sclerosis: features


• 3 times more common in women
• most commonly diagnosed in people aged 20-40 years

Patient's with multiple sclerosis (MS) may present with non-specific features,
for example around 75% of patients have significant lethargy.

Visual
• optic neuritis: common presenting feature
• optic atrophy
• Uhthoff's phenomenon: worsening of vision following rise in body
temperature
• internuclear ophthalmoplegia

Sensory
• pins/needles
• numbness
• trigeminal neuralgia
• Lhermitte's syndrome: paraesthesiae in limbs on neck flexion

Motor

191
CNS-System Wise 1700-by Sush and Team. 2016
Susmita, Asad, Manu, Saima, Zohaib, Savia, Shanu, Mona, Manisha, Sitara, Samreena, Sami and Komal

• spastic weakness: most commonly seen in the legs

Cerebellar
• ataxia: more often seen during an acute relapse than as a presenting
symptom
• tremor

Others
• urinary incontinence
• sexual dysfunction
• intellectual deterioration



Q:538 A 63yo man presents after having a seizure. Exam: alert, orientated,
inattention on the left side and hyperreflexia of the arm. What is the most
probable dx?
a. Cerebral tumor
b. Pituitary adenoma
c. Cerebellar abscess
d. Huntingtons chorea
e. Parkinsonism

Clincher(s)
A CEREBRAL TUMOR
B PITUITARY ADENOMA
C CEREBELLAR ABSCESS
D HUNTINGTON’S CHOREA
E PARKINSONISM
KEY A
Additional
Information
Reference
Dr Khalid/Rabia Inattention or neglect is a feature of parietal lobe lesion.
If lesion is on right side there will be left sided inattention.
That is patient is unaware of his left side and he when shaves do it only to right
half of face, during eating eats only from the right half of plate and can not
drive as he only aware of his right side and totally unaware of left side of the
road nothing else fit thats the best expl..

pituitary will give bitemporal vision loss

cerebellar signs are nystagmus ataxia etc.

Chorea is repeated movements.. its focal ant lobe lesion.

192
CNS-System Wise 1700-by Sush and Team. 2016
Susmita, Asad, Manu, Saima, Zohaib, Savia, Shanu, Mona, Manisha, Sitara, Samreena, Sami and Komal

Seizure n hyperflexia (exaggerated reflexes) are indicators of space occupied
lesion (SOL)

Because the pt has got upper motor neuron signs.
Its not pituitary because no signs of optic n compression and its not bilateral.

Its not cerebellar dis other wise he should have ataxia, pass pointing
rombergism.

Its not chorea cuz no symptoms of chorea.

Not parkinson because no hypokinesia, tremor
seizure causing neuro deficit in elderly... first D/D should b cerebral tumour
unless specified otherwise


Q:539 A 40yo man with a 25y hx of smoking presents with progressive hoarseness of
voice, difficulty swallowing and episodes of hemoptysis. He mentioned that he
used to be a regular cannabis user. What is the single most likely dx?
a. Nasopharyngeal cancer
b. Pharyngeal carcinoma
c. Sinus squamous cell carcinoma
d. Squamous cell laryngeal cancer
e. Hypopharyngeal tumor


Clincher(s)
A Nasopharyngeal cancer
B Pharyngeal carcinoma
C Sinus squamous cell carcinoma
D Squamous cell laryngeal cancer
E Hypopharyngeal tumor
KEY D
Additional
Information
Reference
Dr Khalid/Rabia Hoarseness of voice is localizing the problem to the larynx
The symptoms of cancer of the pharynx differ according to the type:
· Oropharynx: common symptoms are a persistent sore throat, a lump in the
mouth or throat, pain in the ear.
· Hypopharynx: problems with swallowing and ear pain are common
symptoms and hoarseness is not uncommon.
· Nasopharynx: most likely to cause a lump in the neck but may also cause
nasal obstruction, deafness and postnasal discharge.

Laryngeal Cancer

193
CNS-System Wise 1700-by Sush and Team. 2016
Susmita, Asad, Manu, Saima, Zohaib, Savia, Shanu, Mona, Manisha, Sitara, Samreena, Sami and Komal

· Smoking is the main avoidable risk factor for laryngeal cancer
· Chronic hoarseness is the most common early symptom.
· urgent CXR to decide where to refer
· Flexible laryngoscopy is the best way to inspect the larynx
· staging include CT and/or MRI scans.




Q:1076 A 7yo school boy has been dx with meningococcal meningitis. What is the
advice for schoolmates and staff?
a. Rifampicin for the whole class and family
b. Rifampicin for the whole school and family
c. Meningococcal vaccine for the family
d. Benzyl penicillin
e. IV cefotaxime


Clincher(s)
A Rifampicin for the whole class and family
B Rifampicin for the whole school and family
C Meningococcal vaccine for the family
D Benzyl penicillin
E IV cefotaxime
KEY A
Additional Prophylaxis: (discuss with public health/ID) •Household contacts in droplet
Information range.
•Those who have kissed the patient’s mouth. Give rifampicin (600mg/12h PO
for 2d;child ren >1yr 10mg/kg/12h; <1yr 5mg/kg/12h) or ciprofl oxacin (500mg
PO, 1 dose child 5–12yrs: 250mg stat): neither is guaranteed in pregnancy, but
are recommended.

Household contacts People sharing a bedroom, kitchen, bathroom or sitting
room with the index case
OHCM- 832
Reference
Dr Khalid/Rabia NA
Q:1088 A 74yo lady who has had a stroke in the past has an indwelling catheter for
10m. She presents with bluish-purple discoloration of the catheter bag. What
is the most likely explanation for this?
a. Normal change
b. Catheter degradation
c. Acidic urine
d. Alkaline urine
e. Bacterial colonization of the urinary tract

194
CNS-System Wise 1700-by Sush and Team. 2016
Susmita, Asad, Manu, Saima, Zohaib, Savia, Shanu, Mona, Manisha, Sitara, Samreena, Sami and Komal


Clincher(s)
A
B
C
D
E Bacterial colonization of the urinary tract-
KEY
Additional • Presence of purple pigment Indigo and blue pigment Indirubin
Information • Altered motility of gut
• Intestinal bacterial overgrowth
• Tryptophan exposure to intestinal flora before absorption
• Bacteria deaminates tryptophan to yield Indole absorbed from bowel
• Undergoes conjugation in liver to Indoxyl Sulphate
• Indoxyl Sulfate converted to Indole in urine
• Alkaline environment favors conversion of Indoxyl to Indigo
• Low oxygen favors production of Indirubin

Due to bacterial contamination : colour changes
Reference Oxford Journal
Dr Khalid/Rabia NA


Q: 1158 An 08yrs old boy develops a seizure affecting his right arm, seizure lasts for
several mins. He doesn’t remember anything what happened. On his CT: lesion
in left hemisphere. What is the most probable dx?
a. Epilepsy
b. Space occupying lesion
c. Dementia
d. Huntington’s chorea
e. Intracranial HTN

Wrong question
Clincher(s)
A Epilepsy- too generalised
B Space occupying lesion- clincher is: Ct/lesion (50% of patient with seizure has
space occupying lesion- eg. Tumour in the space)

195
CNS-System Wise 1700-by Sush and Team. 2016
Susmita, Asad, Manu, Saima, Zohaib, Savia, Shanu, Mona, Manisha, Sitara, Samreena, Sami and Komal

C Dementia
D Huntington’s chorea
E Intracranial HTN
KEY
Additional CT lesion >>> Space occupying lesion
Information
brain tumors in children

the most common subtype >>> astrocytoma followed by

the second most common >>> embryonal tumours ( primitive


neuroectodermal tumours and medulloblastoma)

Presentation:

• increased intracranial pressure : Headache, nausea and vomiting,


abnormalities of gait and coordination, and papilloedema.

• frontal lobe tumours are associated with personality change and


occipital lobe tumours are associated with visual deficits

• Brainstem tumours: Abnormal gait and coordination, cranial nerve


palsies, pyramidal signs, headache and squint.

• Central brain tumours: Headache, abnormal eye movements,


squint, and nausea and vomiting

• Supratentorial tumours: Unspecified symptoms and signs of raised


intracranial pressure, seizures and papilloedema.

• Posterior fossa tumours: Nausea and vomiting, headache,


abnormal gait and coordination, and papilloedema

investigations :

• MRI , CT : MRI is better , it provides better images and there is no


radiation involved.

• Excision biopsy


Reference
Dr Khalid/Rabia


Q:1500 A 43yo presents with severe vertigo on moving sidewards whilst sleeping.
What test would you
do to confirm the dx?
a. Hallpikes maneovure
b. Romberg’s test test
c. Trendelenburg test

196
CNS-System Wise 1700-by Sush and Team. 2016
Susmita, Asad, Manu, Saima, Zohaib, Savia, Shanu, Mona, Manisha, Sitara, Samreena, Sami and Komal

d. Heel-shin test


Clincher(s)
A Hallpikes maneovure- Benign positional vertigo is due to canalolithiasis—
debris in the semicircular canal, disturbed by head movement, resettles
causing vertigo lasting a few seconds after the movement (often turning over
in bed). Nystagmus on performing the Hallpike manoeuvre is diagnostic.Epley
manoeuvres clear the debris from the semicircular canals (OHCS p555).
B Romberg’s test test
C Trendelenburg test
D Heel-shin test
E
KEY A. Hallpikes manoeuvre
Additional Benign positional vertigo
Information disturbed by head
movement,
Nystagmus on performing the Hallpike manoeuvre is diagnostic.
Epley manoeuvres clear the debris from the semicircular canals (OHCS p555).
Acute labyrinthitis (vestibular neuronitis): Abrupt onset on
prostration. No deafness or tinnitus.
Causes: virus; vascular lesion. Severe
vertigo subsides in days
complete recovery takes 3–4wks.
Reassure. Sedate.
Ménière’s disease: Endolymphatic hydrops causes recurrent attacks of vertigo
lasting >20min
fl uctuating (or permanent) sensorineural hearing loss, tinnitus (with a sense of
aural fullness drop attacks (no loss of consciousness or vertigo, but falling to
one side, ie Tumarkin otolithic catastrophes)) : Acute attacks—bed rest and
reassurance.
An antihistamine (eg cinnarizine) is useful if prolonged,buccal prochlorperazine
if severe, for up to 7d.
In very severe disease,consider endolymphatic sac surgery or ablation of the
vestibular organ with gent amicin.
Ototoxicity: Amino glycosides, loop diuretics or cisplatin can cause deafness
vertigo.
Acoustic neuroma
Schwannoma (not neuroma)
vestibular (not auditory) nerve.
Unilateral hearing loss, with vertigo.
With progression, ipsilateral Vth, VIth, IXth, & Xth nerves may be affected
(also ipsilateral cerebellar signs).
Paradoxically, the VIIth nerve is rarely involved pre-operatively.
Signs of raised ICP occur late, indicating a large tumour.
They account for 80% of cerebello-pontine angle tumours ( meningioma).

197
CNS-System Wise 1700-by Sush and Team. 2016
Susmita, Asad, Manu, Saima, Zohaib, Savia, Shanu, Mona, Manisha, Sitara, Samreena, Sami and Komal

Traumatic damage involving the petrous temporal bone or the cerebello-
pontine angle often affects the auditory nerve, causing vertigo, deafness
and/or tinnitus.
Herpes zoster: Herpetic eruption of the external auditory meatus; facial palsy
deafness, tinnitus, and vertigo (Ramsay Hunt syndrome, see p505).

Reference Ohcm
Dr Khalid/Rabia


Q:1501 A 23yo man is having difficulty in speaking following a stab wound to the right
of his neck. On being asked to protrude his tongue, the tip deviated to the
right. Which anatomical site is most likely to be affected?
a. Facial nerve
b. Hypoglossal nerve
c. Vagus nerve
d. Trigeminal nerve
e. Glossopharyngeal nerve


Clincher(s)
A Facial nerve- Supplies the muscles of facial expression and taste to anterior
two-thirds of the tongue (via chorda tympani branch).
• Lower motor neuron (LMN) facial palsies result in complete ipsilateral
facial weakness.
• The upper face is bilaterally innervated—frontalis and to a lesser extent
orbicularis oculi are spared in upper motor neuron (UMN) palsies.
B Hypoglossal nerve-Observe for fasciculations—may be difficult. Observe with
tongue inside
the mouth.
• Tongue strength assessed by asking patient to push inside the mouth
against cheek.
• Tongue movement dexterity assessed by asking patient to move tongue
side to side. Slowness without wasting suggests spasticity.
• In LMN lesions tongue deviates to the side of the lesion.
C Vagus nerve-Motor fibres innervate the striated muscles of palate, pharynx,
larynx.
• Soft palate observed as patient says ‘aahh’:
• deviation away from side of lesion.
• Lesions of recurrent laryngeal branch cause ipsilateral vocal cord
paralysis with dysphonia and a weak cough.
• Parasympathetic autonomic fibres travel in the vagus nerve to the
respiratory, GI, and cardiovascular systems
D Trigeminal nerve- • Ophthalmic (V ). Extends posteriorly to the vertex.
• Sensation but not taste to anterior two-thirds of the tongue also
supplied by TG nerve.

198
CNS-System Wise 1700-by Sush and Team. 2016
Susmita, Asad, Manu, Saima, Zohaib, Savia, Shanu, Mona, Manisha, Sitara, Samreena, Sami and Komal

• Motor fibres to muscles of mastication (temporalis, masseter, and
pterygoids via mandibular division).
• Jaw deviates to side of weak pterygoid muscle.
• Corneal reflex has a consensual component. Useful in the presence of
an ipsilateral facial palsy.
• Jaw jerk—if brisk indicates pathology above midbrain level.
• Roger’s sign = numb chin syndrome due to metastatic deposit around
inferior alveolar branch. Breast cancer, lymphoma.
E Glossopharyngeal nerve- Taste fibres from posterior third of the tongue.
• G eneral sensation tympanic membrane, mucous membranes from
posterior pharynx, tonsils, and soft palate.
• Afferent part of the gag reflex.
KEY B
Additional The hypoglossal nerve is the twelfth cranial nerve XII, and innervates muscles
Information of the tongue.The nerve is involved in controlling tongue movements required
for speech, food manipulation (i.e. formation of bolus), and
swallowing.Damage or lesions affecting functions of the nerve may be
supranuclear (as in cortical or rostrally to the hypoglossal nucleus in the
brainstem), nuclear or infranuclear (as in below or distal to the nucleus), as
well as unilateral or bilateral and will give symptoms which differ
accordingly.[2] Supranuclear injury of the brainstem affecting the hypoglossal
nerve may give rise to crossed symptoms due to a majority of the supranuclear
innervation to the hypoglossal nucleus being crossed. Symptoms often show
deviation of the tongue towards the paralyzed side when it is stuck out. This is
because of the weaker genioglossal muscle
Reference
Dr Khalid/Rabia


Q:1506 A 69yo woman presents with a sudden onset of weakness of her right arm and
leg. She is known to be hypertensive. There has been no headache, LOC, visual,
speech or sensory symptoms.
Exam: BP=180/90mmHg, pulse=100 and regular heart sounds, no carotid bruit.
Higher mental function tests are normal. No apraxia or neglect. Speech,
swallowing and sensation are normal.
There are no visual field defects. There is a mild facial weakness sparing the
forehead. The right arm and leg are flaccid and weak. Reflexes and tone are
normal. There is a right extensor plantar response. What is the most likely
cause of this pt’s symptoms?
a. Cardioembolic stroke
b. Lacunar stroke
c. Right internal carotid artery atheroembolic stroke
d. Right internal carotid artery dissection
e. Right vertebral artery atheroembolic stroke

199
CNS-System Wise 1700-by Sush and Team. 2016
Susmita, Asad, Manu, Saima, Zohaib, Savia, Shanu, Mona, Manisha, Sitara, Samreena, Sami and Komal


Clincher(s) Since the affected side of the body is right side, C.D.E can be excluded .
So, B is the answer. What's more, the symptoms resemble the lacunar stroke
in which aphasia, neglect and visual field defects are always absent.
A Cardioembolic stroke (no bruit)
B Lacunar stroke (deep stroke_
C Right internal carotid artery atheroembolic stroke
D Right internal carotid artery dissection
E Right vertebral artery atheroembolic stroke
KEY B
Additional Answer: B
Information Lacunar infarcts (25%):
Small infarcts around the basal ganglia, internal capsule, thalamus and
pons.May cause pure motor, pure sensory, mixed motor and sensory signs, or
ataxia.
Intact cognition/consciousness.

Lacunar strokes
• Pure motor strokes (face, arm, and leg) in the posterior limb of internal
capsule.
• Pure sensory stroke (thalamus).
• Ataxic hemiparesis (weakness and ataxia affecting the same side) due to a
pontine lesion.
• Clumsy hand/dysarthria due to a lesion in the pons or internal capsule.
Paradoxical embolic stroke
Embolus from venous thromboembolism entering arterial circulation.
Increasingly recognized cause of strokes especially in young:
• Caused by venous thrombosis, e.g. oral contraceptive pill, long plane journey
or thrombophilic tendency.
• Cardioembolic stroke, e.g. posterior cerebral artery territory or small cortical
infarct.
• Intracardiac shunt identified, e.g. patent foramen ovale (PFO), but this is
present in 25% of population. Associated atrial septal aneurysm i risk

Reference
Dr Khalid/Rabia

Q: 1510. A 74yo man has been admitted unconscious with no hx. He has a GCS=6
(means in comma) and a dilated left pupil which becomes insensitive to light.
What is the single most likely dx?
a. Extradural hematoma
b. Meningitis
c. Opioid OD (pin point pupil- so excluded)
d. Pontine hemorrhage (pin point pupil)
e. SAH

200
CNS-System Wise 1700-by Sush and Team. 2016
Susmita, Asad, Manu, Saima, Zohaib, Savia, Shanu, Mona, Manisha, Sitara, Samreena, Sami and Komal



Clincher(s)
A Extradural hematoma- Extradural haematoma (EDH)
• Classically after a head injury
• Instant LO C, followed by a lucid interval and later by a progressive
decline in GCS.
• Haemorrhage is arterial (usually posterior branch of middle meningeal
artery is torn at site of skull fracture).
• Bleeding is extradural and strips the dura mater from the inner aspect of
the skull, compressing the brain.
Imaging
CT
• Does not cross suture lines.
• 20% can develop or enlarge after a delay of 36 hours.
• 50% associated with other lesions, e.g. contre-coup contusions, SDH,
and SAH.
Management
• True neurosurgical emergency: if necessary resuscitate during transfer.
• Surgical procedure: burr hole over pterion (to ensure that further
haemorrhage escapes instead of expanding the clot further) followed by
craniotomy and evacuation of the haematoma.
Outcome
• Depends on preoperative status.
• Patients with bilateral fixed dilated pupils may still recover if surgery is
immediate.
• If preoperative GCS ≥ 8, 90–100% recovery. If GCS < 8, mortality rate
30%; good outcome 50–60%.

B Meningitis- Presenting features typically with headache, fever, photophobia,
neck stiffness.
In addition:
• cranial nerve palsies (III, IV, VI, VII);
• focal neurological deficits;
• seizures (20–30%)—usually in S.pneumoniae and Haemophilus influenzae
meningitis;
• i ICP (altered conscious level, hypertension, bradycardia, abnormal
respiratory pattern, papilloedema (late));
• purpura or petechial haemorrhages (non-blanching with glass test)—
Neisseria meningitidis;
• septic shock: N.meningitidis;
• tuberculous meningitis may be more insidious with gradual development
of fever, weight loss, headache with progression to focal deficit, altered
consciousness;
• look for evidence of immunosuppression as may be the first
presentation of HIV or lymphoproliferative disorder (e.g. oral

201
CNS-System Wise 1700-by Sush and Team. 2016
Susmita, Asad, Manu, Saima, Zohaib, Savia, Shanu, Mona, Manisha, Sitara, Samreena, Sami and Komal

candidiasis, lymphadenopathy). 151 OHN
C Opioid OD
D Pontine hemorrhage- Small, reactive = pontine lesion (‘pin-point pontine
pupils’)
E SAH-

The central feature of classic SAH is sudden onset of severe headache


(thunderclap headache), often described as the "worst headache of my life."

Photophobia and visual changes are common. Focal neurologic deficits may
also occur.

Sudden loss of consciousness (LOC) occurs at the ictus in as many as 45% of


patients as intracranial pressure (ICP) exceeds cerebral perfusion pressure.

LOC often is transient

however, approximately 10% of patients remain comatose for several days,


depending on the location of the aneurysm and the amount of bleeding.

Proposed decision rule for diagnosis of SAH focuses on the following 7


characteristics, which are strongly associated with SAH:

• Aged 40 years or older


• Witnessed loss of consciousness
• Complaint of neck pain or stiffness
• Onset of manifestations with exertion
• Arrival by ambulance
• Vomiting
• Diastolic blood pressure ≥100 mm Hg or systolic blood pressure ≥160
mm Hg

Cranial nerve palsies, along with memory loss, are present in 25% of patients.
The most frequent is oculomotor nerve palsy with or without ipsilateral
mydriasis, which results from rupture of a posterior communicating artery
aneurysm. Abducens nerve palsy is usually due to increased ICP rather than a
true localizing sign. Monocular vision loss can be caused by an ophthalmic
artery aneurysm compressing the ipsilateral optic nerve.

Hemiparesis results from middle cerebral artery (MCA) aneurysm, ischemia or


hypoperfusion in the vascular territory, or intracerebral clot. Patients may also
have aphasia, hemineglect, or both. Leg monoparesis or paraparesis with or
without akinetic mutism/abulia points to anterior communicating aneurysm
rupture.


KEY E

202
CNS-System Wise 1700-by Sush and Team. 2016
Susmita, Asad, Manu, Saima, Zohaib, Savia, Shanu, Mona, Manisha, Sitara, Samreena, Sami and Komal

Additional
Information
Reference
Dr Khalid/Rabia Ruptured posterior communicating artery aneurysm causes unilateral 3rd
nerve palsy and also SAH.(EDH also producing 3rd nerve palsy due to uncal
herniation and raised ICF and compressed brain but absent of relevant history
regarding EDH such as head trauma).
In elderly dura mater becomes more sticky and adherent to skull and it hardly
allow any blood to take place extradurally! So in elderly extradural hematoma
is very unlikely.

Subarachnoid haemorrhage (SAH) is usually the result of bleeding from a berry
aneurysm in the Circle of Willis. These are called berry aneurysms because of
their shape. They were once thought to be mostly congenital but it is now
thought that the aetiology may involve susceptibility of the elastic lamina, in
some patients, to stressors such as hypertension and atherosclerosis.
There may be warning symptoms in the three weeks prior to SAH that
represent small leaks. These are called sentinel bleeds or expansion of the
aneurysm. These are usually headaches with the characteristics of SAH but
which resolve by themselves without further symptoms. They are estimated to
occur in 10-15% of patients.

The most common symptoms are headache (48%), dizziness (10%), orbital pain
(7%), diplopia (4%) and visual loss (4%).
Signs may accompany these sentinel bleeds: sensory or motor disturbance
(6%), seizures (4%), ptosis (3%), bruits (3%) and dysphasia (2%).
If a sentinel bleed is suspected, patients should be admitted urgently for
investigations (treat as if an SAH has occurred).
Examination
Conscious level: on admission to hospital two thirds have a depressed level of
consciousness, of whom half are in coma. However, SAH patients can also walk
into the surgery, complaining of sudden onset of headache.
Neck stiffness may occur due to meningeal irritation by blood in the CSF, but it
is not invariable.
Ophthalmoscopy will show intraocular haemorrhages in around 15%,
especially in those with a depressed level of consciousness.
Isolated pupillary dilation with loss of light reflex may indicate brain herniation
as a result of rising intracranial pressure.
There may be focal neurological signs, suggestive of a stroke. Complete or
partial palsy of the oculomotor nerve is well recognised, especially with
rupture of aneurysms of the internal carotid artery at the origin of the
posterior communicating artery.
Intraocular haemorrhage may occur in response to the raised pressure and is
more common in more severe SAH
Oculomotor nerve impairment may indicate bleeding from the posterior
communicating artery.

203
CNS-System Wise 1700-by Sush and Team. 2016
Susmita, Asad, Manu, Saima, Zohaib, Savia, Shanu, Mona, Manisha, Sitara, Samreena, Sami and Komal

Hypertension is a risk factor for the condition but a marked rise in blood
pressure may also occur as a sympathetic reflex following intracerebral
haemorrhage. This sympathetic reflex can raise blood pressure to life-
threatening levels, and surges of adrenaline (epinephrine) may contribute to
associated cardiac arrhythmias which may both confuse the diagnosis and
further threaten the patient.
In 3% of cases cardiac arrest follows SAH.
SAH in a person known to have seizures is suggestive of an arteriovenous
malformation. New-onset seizures are more indicative of ruptured berry
aneurysm.


Q:1511 A 27yo man presents to the ED with 2d hx of severe headache and pyrexia
(38.9C). CT: petechial hemorrhage in the temporal and inf frontal lobes. What
is the most likely dx?
a. Brain abscess
b. Meningococcal meningitis
c. Cerebral malaria
d. Herpes simplex encephalitis
e. New variant CJD


Clincher(s)
A Space occupying lesion
B Skin rashes
C No travel
D Petechial hemmohages
E
KEY D:
Additional Answer: D
Information Herpes simplex encephalitis (HSE) is recognised worldwide as the most
frequent infectious encephalitis.
In children older than 3 months and in adults: HSE is usually caused by herpes
simplex virus type 1 (HSV-1) and is localised to the temporal and frontal lobes.
In neonates: HSE is usually caused by herpes simplex virus type 2 (HSV-2)
acquired at the time of delivery, and brain involvement is generalised.
Other herpes viruses may cause encephalitis but much less frequently than
HSV. However, cytomegalovirus (CMV) encephalitis should be considered in
those with immunodeficiency
Reference
Dr Khalid/Rabia


Q:1512 A 44yo woman with memory loss, poor concentration and inability to
recognize household projects. She has right-handed involuntary writhing
movement. There is strong fam hx of similar complain. What is the single most

204
CNS-System Wise 1700-by Sush and Team. 2016
Susmita, Asad, Manu, Saima, Zohaib, Savia, Shanu, Mona, Manisha, Sitara, Samreena, Sami and Komal

likely dx?
a. Pic’s dementia
b. Wilson’s disease
c. Huntington’s disease
d. HIV associated dementia
e. Fronto-temporal dementia
Involuntary writhing movement
Clincher(s)
A
B
C Family hx
D
E
KEY
Additional Answer: C
Information Huntington's disease is associated with cell loss within the basal ganglia and
cortex.
It is an autosomal-dominant, progressive neurodegenerative disorder with a
distinct phenotype, including chorea and dystonia, incoordination, cognitive
decline, and behavioural difficulties.[1] Huntington's disease was first
described by George Huntington in 1872. The disease is associated with
increases in the length of a cysteine-adenosine-guanine (CAG) triplet repeat
present in a gene called 'huntingtin' located on chromosome
4p16.3.Huntington's disease is the most common genetic cause of chorea.

Signs and Symptoms:
The mean age at onset of symptoms is 30-50 years.
In some cases symptoms start before the age of 20 years with behavioural
disturbances and learning difficulties at school (juvenile Huntington's disease)
Early signs may be personality change, self-neglect, apathy with clumsiness,
fidgeting with fleeting facial grimaces.

Huntington's disease then leads to progressive chorea, rigidity and dementia.
It is frequently associated with seizures.
Chorea is initially mild but may be severe and cause uncontrollable limb
movements.

As the disease progresses, chorea is gradually replaced by dystonia and
Parkinsonian features.

Dysarthria, dysphagia and abnormal eye movements are common. There may
also be other movement disorders - eg, tics and myoclonus.
Huntington's disease is associated with increasing depression, bradykinesia,
cognitive impairment and aggression as the disease progresses.Behavioural
difficulties include apathy or lack of initiative, dysphoria, irritability, agitation
or anxiety, poor self-care, poor judgment and inflexibility.[1]Late features

205
CNS-System Wise 1700-by Sush and Team. 2016
Susmita, Asad, Manu, Saima, Zohaib, Savia, Shanu, Mona, Manisha, Sitara, Samreena, Sami and Komal

include spasticity, clonus, supranuclear gaze palsy and extensor plantar
responses. The rate of cognitive decline is very variable.

Investigations:
MRI and CT scans in moderate-to-severe Huntington's disease show a loss of
striatal volume and increased size of the frontal horns of the lateral ventricles,
but scans are usually unhelpful for diagnosis of early disorder.
If genetic testing is considered then extensive genetic counselling in a
specialist unit is required in view of the implications of an untreatable, familial,
progressive, neurodegenerative disease.
Treatment:
Current drug therapy has no effect on the progression of disability.
Hyperkinesias and psychiatric symptoms may respond well to
pharmacotherapy, but neuropsychological deficits and dementia remain
untreatable.[8]
Patients, their families and carers require a great deal of physical and
emotional support.
Chorea:
Benzodiazepines, valproic acid, dopamine-depleting agents (eg, tetrabenazine)
and neuroleptics may be useful.


Reference
Dr Khalid/Rabia


Q:1513 A 54yo man has collapsed suddenly following a headache. He has hypertension
and takes warfarin for prosthetic heart valve. GCS=4 and dilated left pupil.
What is the single most likely dx?
a. Ant circulation stroke (ant cerebral artery)
b. Post circulation stroke
c. Intracerebral hemorrhage
d. Intracerebellar hemorrhage (other symptoms)
e. Pontine hemorrhage


Clincher(s)
A
B
C . Intracerebral hemorrhage (HTN, takes warfarin- stroke)
D
E
KEY C
Additional Intracerebral hemorrhage occurs when a diseased blood vessel within the
Information brain bursts, allowing blood to leak inside the brain. (The name means within
the cerebrum or brain). The sudden increase in pressure within the brain can

206
CNS-System Wise 1700-by Sush and Team. 2016
Susmita, Asad, Manu, Saima, Zohaib, Savia, Shanu, Mona, Manisha, Sitara, Samreena, Sami and Komal

cause damage to the brain cells surrounding the blood. If the amount of blood
increases rapidly, the sudden buildup in pressure can lead to unconsciousness
or death. Intracerebral hemorrhage usually occurs in selected parts of the
brain, including the basal ganglia, cerebellum, brainstem, or cortex.

What causes it?
The most common cause of intracerebral hemorrhage is high blood pressure
(hypertension). Since high blood pressure by itself often causes no symptoms,
many people with intracranial hemorrhage are not aware that they have high
blood pressure, or that it needs to be treated. Less common causes of
intracerebral hemorrhage include trauma, infections, tumors, blood clotting
deficiencies, and abnormalities in blood vessels (such as arteriovenous
malformations). View an interactive tutorial on arteriovenous malformations
from the Toronto Brain Vascular Malformation Study Group.

Reference
Dr Khalid/Rabia


Q:1520 1520. An 82yo woman has been admitted from a nursing home with dense
hemiplegia and homonymous hemianopia. She is dysphasic. What vessel is
most likely to be involved?
a. Ant cerebral artery
b. Mid cerebral artery
c. Post cerebral artery
d. Internal carotid artery
e. Post inf cerebellar artery

Clincher(s)
A Ant cerebral artery
B Mid cerebral artery- homonymous hemianopia
C Post cerebral artery- supplies occipital lobe- visual symp
D Internal carotid artery
E Post inf cerebellar artery
KEY B OCHm- Page - 453
Additional Areas supplied by the middle cerebral artery include:
Information
The bulk of the lateral surface of the hemisphere; except for the superior inch
of the frontal and parietal lobe (anterior cerebral artery), and the inferior part
of the temporal lobe.
Superior division supplies latero inferior frontal lobe (location of Broca's area
i.e. language expression)
Inferior division supplies lateral temporal lobe (location of Wernicke's area i.e.
language comprehension)
Deep branches supply the basal ganglia as well as the internal capsule.
Occlusion of the middle cerebral artery results in Middle cerebral artery

207
CNS-System Wise 1700-by Sush and Team. 2016
Susmita, Asad, Manu, Saima, Zohaib, Savia, Shanu, Mona, Manisha, Sitara, Samreena, Sami and Komal

syndrome, potentially showing the following defects:

Paralysis (-plegia) or weakness (-paresis) of the contralateral face and arm
(faciobrachial)
Sensory loss of the contralateral face and arm.
Damage to the dominant hemisphere (usually the left hemisphere) results in
aphasia i.e. Broca's or Wernicke's
Damage to the non-dominant hemisphere (usually the right hemisphere)
results in contralateral neglect syndrome
Large MCA infarcts often have déviation conjuguée, a gaze preference towards
the side of the lesion, especially during the acute period. Contralateral
homonymous hemianopsia is often present.

Reference
Dr Khalid/Rabia




Q: 300 A 65yo HTN man wakes up in the morning with slurred speech, weakness of
the left half of his body and drooling. Which part of the brain is affected?
a. Left parietal lobe
b. Right internal capsule
c. Right midbrain
d. Left frontal lobe



Clincher(s) H/O hypertension, left side of body affected
A •Hemisensory loss; •lost 2-point discrimination; •Astereognosis (unable
to recognize an object by touch alone); •Sensory inattention; •Dysphasia
(p80); •Gerstmann’s syndrome (p714).
B Lacunar infarcts: (25%) In basal ganglia,3 internal capsule, thalamus, and pons.
5 syndromes: ataxic hemiparesis, pure motor, pure sensory, sensorimotor, and
dysarthria/clumsy hand. Cognition/consciousness are intact except thalamic
stroke

C (eg pineal tumours or midbrain infarction) Failure of up or down gaze;
light/near dissociated pupil responses (p79), with convergence globe-
retracting nystagmus 284 from co-contraction of opposing horizontal muscles,
on attempted up-gaze. 285 Elicited by looking at a down-moving target.

Lesion in midbrain: Cranial nerve , 3rd 4th CN
D
Hemiparesis; •Personality change (indecent, indolent, indiscreet,
facetious, tendency to pun; see also orbitofrontal syndrome below); •Release

208
CNS-System Wise 1700-by Sush and Team. 2016
Susmita, Asad, Manu, Saima, Zohaib, Savia, Shanu, Mona, Manisha, Sitara, Samreena, Sami and Komal

phenomena such as the grasp reflex (fingers drawn across palm are grasped),
significant only if unilateral; •Broca’s dysphasia (p80), or more subtle difficulty
with initiating and planning speech with intact repetition and no anomia—but
loss of coherence; •Unilateral anosmia (loss of smell); •General lack of drive or
initiative;
•Concrete thinking; •Perseveration (unable to switch from one line of thinking
to another); •Executive dysfunction (unable to plan tasks); • absent verbal
fluency, eg unable to list words beginning with the letter ‘A’ or ‘F’ (normal is
~15 words in 1 min).

CN in pons 5,6,7,8, rest in medulla

Frontal lobe: speech
Temporal lobe: memory
Paretal: sensoory
Brianstem: MPM: CN 2 , 4, 4 number
E -
KEY B
Additional The primary motor cortex sends its axons through the posterior limb of the
Information internal capsule. Lesions, therefore, result in a contralateral hemiparesis or
hemiplegia. While symptoms of weakness due to an isolated lesion of the
posterior limb can initially be severe, recovery of motor function is sometimes
possible due to spinal projections of premotor cortical regions that are contained
more rostrally in the internal capsule
Reference

Dr Khalid/Rabia


Q: 325 In 85% of the population this artery is dominant. What is the single most
appropriate option?
a. Left ant descending artery
b. Coronary sinus
c. Circumflex artery
d. Left main stem, post descending artery
e. Right coronary artery



Clincher(s)
A
B

C
D
E

209
CNS-System Wise 1700-by Sush and Team. 2016
Susmita, Asad, Manu, Saima, Zohaib, Savia, Shanu, Mona, Manisha, Sitara, Samreena, Sami and Komal

KEY E
Coronary artery dominance (main supply to SA node)

The artery that supplies the posterior descending artery (PDA)[2] determines the
coronary dominance.[3]

• If the posterior descending artery is supplied by the right coronary artery


(RCA), then the coronary circulation can be classified as "right-
dominant".
• If the posterior descending artery is supplied by the circumflex artery
(CX), a branch of the left artery, then the coronary circulation can be
classified as "left-dominant".
• If the posterior descending artery is supplied by both the right coronary
artery and the circumflex artery, then the coronary circulation can be
classified as "co-dominant".

Approximately 70% of the general population are right-dominant, 20% are co-
dominant, and 10% are left-dominant.[3] A precise anatomic definition of
dominance would be the artery which gives off supply to the AV node i.e. the
AV nodal artery. Most of the time this is the right coronary artery.


Dr Khalid/Rabia wikipedia






Q: 326 A 54 yo lady presents with sudden, severe pain in the left half of her skull. She
also complains of pain around her jaw. What is the next likely step?
a. CT
b. MRI
c. Fundoscopy
d. ESR
e. Temporal artery biopsy




Clincher(s)
A
B
C
D

E

210
CNS-System Wise 1700-by Sush and Team. 2016
Susmita, Asad, Manu, Saima, Zohaib, Savia, Shanu, Mona, Manisha, Sitara, Samreena, Sami and Komal


KEY D- ESR (most definite or diagnostiic E)- Temporal artery biosy
Additional Trigeminal neuralgia: Symptoms: Paroxysms of intense, stabbing pain, lasting
Information seconds, in the trigeminal
nerve distribution. It is unilateral, typically aff ecting mandibular or maxillary
div isions. The face screws up with pain (hence tic doloureux). Triggers:
Washing
affected area, shaving, eating, talking, dental prostheses. Typical patient:
>50yrs old; in Asians : ≈ 2:1. Secondary causes: Compression of the
trigeminal root by anomalous or aneurysmal intracranial vessels or a tumour,
chronic meningeal inflammation, MS, zoster, skull base malformation (eg
Chiari). MRI is necessary to exclude secondary causes (~14% of cases). 27 蕂:
Carbamazepine (start at 100mg/12h PO; max 400mg/6h; lamotrigine;
phenytoin 200–400mg/24h PO; or gabapentin (p508). If drugs fail, surgery may
be necessary. This may be directed at the peripheral nerve, the trigeminal
ganglion or the nerve root. Microvascular decompression: Anomalous vessels
are separated from the trigem inal root. Stereotactic gamma knife surgery can
work, but length of pain relief and the time to treatment response are limiting
factors. Facial pain 兪兪: p71.

Temporal arteritis:
Temporal arteritis is large vessel vasculitis which overlaps with polymyalgia
rheumatica (PMR). Histology shows changes which characteristically 'skips'
certain sections of affected artery whilst damaging others.

Features

• typically patient > 60 years old


• usually rapid onset (e.g. < 1 month)
• headache (found in 85%)
• jaw claudication (65%)
• visual disturbances secondary to anterior ischemic optic neuropathy
• tender, palpable temporal artery
• features of PMR: aching, morning stiffness in proximal limb muscles
(not weakness)
• also lethargy, depression, low-grade fever, anorexia, night sweats

Investigations

• raised inflammatory markers: ESR > 50 mm/hr (note ESR < 30 in 10%
of patients). CRP may also be elevated
• temporal artery biopsy: skip lesions may be present
• note creatine kinase and EMG normal

Treatment

• high-dose prednisolone - there should be a dramatic response, if not the

211
CNS-System Wise 1700-by Sush and Team. 2016
Susmita, Asad, Manu, Saima, Zohaib, Savia, Shanu, Mona, Manisha, Sitara, Samreena, Sami and Komal

diagnosis should be reconsidered
• urgent ophthalmology review. Patients with visual symptoms should be
seen the same-day by an ophthalmologist. Visual damage is often
irreversible


Reference ohcm
Dr Khalid/Rabia


Q:332 A man presents with outward deviation of his right eye and diplopia. Which
nerve is affected?
a. Left trochlear
b. Left oculomotor
c. Right trochlear
d. Right abducens
e. Right oculomotor



Clincher(s) Outward deviation shows unopposed lateral rectus action, so Medial rectus
problem
A
Trochlear nerve supplies superior oblique muscle
B

C

D
Abducens nerve supplies lateral rectus muscle
E

KEY E
Additional The oculomotor nerve innervate skeletal muscle of the levator palpebrae
Information superioris, superior rectus, medial rectus, inferior rectus, and inferior oblique
muscles.


Reference wikipedia
Dr Khalid/Rabia LR6 SO4 and R3 (rest -3)


Q: 334 A 12yo girl presented with tics, LOC, no residual sign and no post-ictal phase.
EEG abnormality in temporal lobe. The girl had a rapid recovery. What is the
most probably dx?
a.Generalized tonic-clonic
b.Myoclonic

212
CNS-System Wise 1700-by Sush and Team. 2016
Susmita, Asad, Manu, Saima, Zohaib, Savia, Shanu, Mona, Manisha, Sitara, Samreena, Sami and Komal

c.Partial!generalized seizure
d. Atonic seizure
e.Febrile convulsion


Clincher(s) Eeg abnormality in one lobe, no post ictal phase
A Tonic–clonic seizures: Loss of consciousness. Limbs stiff en (tonic), then jerk
(clonic). May have one without the other. Post-ictal confusion and drowsiness.

No residual symptoms in the question

B Myoclonic seizures: Sudden jerk of a limb, face or trunk. The patient may be
thrown suddenly to the ground, or have a violently disobedient limb: one
patient described it as ‘my flying-saucer epilepsy’, as crockery which happened
to be in the hand would take off. No LOC

C
D • Atonic (akinetic) seizures: Sudden loss of muscle tone causing a fall, no LOC
E With history of fever
KEY C
Additional Partial seizures Focal onset, with features referable to a part of one
Information hemisphere (EEG show in one part- temp lobe) . Often seen with underlying
structural disease.
• Simple partial seizure: Awareness is unimpaired, with focal motor, sensory
(olfactory, visual, etc), autonomic or psychic symptoms. No post-ictal
symptoms.
• Complex partial seizures: Awareness is impaired. May have a simple partial
onset (=aura), or impaired awareness at onset. Most commonly arise from the
temporal lobe. Post-ictal confusion is common with seizures arising from the
temporal lobe, whereas recovery is rapid after seizures in the frontal lobe.
• Partial seizure with secondary generalization: In ⅔ of patients with partial
seizures, the electrical disturbance, which start s focally (as either a simple or
complex partial seizure), spreads widely, causing a secondary generalized
seizure, which is typically convulsive.

Reference ohcm
Dr Khalid/Rabia

Q: 344 A 68yo man gets repeated attacks of LOC and TIA. What is the most likely
cause for this?
a. Atrial fib
b. Mitral stenosis
c. Aortic stenosis
d. HOCM (hypertrophic obstructive cardiomyopathy- in younger
age)

213
CNS-System Wise 1700-by Sush and Team. 2016
Susmita, Asad, Manu, Saima, Zohaib, Savia, Shanu, Mona, Manisha, Sitara, Samreena, Sami and Komal

e. Carotid artery stenosis



Clincher(s)
A Atrial fibrillation: Symptoms May be asymptomatic or cause chest pain,
palpitations, dysp noea, or faintness. Signs Irregularly irregular pulse, the
apical pulse rate is greater than the radial rate and the 1st heart sound is of
variable intensity; signs of LVF (p812).
Examine the whole patient: AF is often associated with non-cardiac disease.
Tests ECG shows absent P waves, irregular QRS complexes. Blood tests: U&E,
cardiac enzymes, thyroid function tests. Consider echo to look for left atrial
enlarge ment, mitral valve disease, poor LV function, and other structural
abnormalities.
B
C
D
E
KEY
Additional Carotid artery plaque can be stable and asymptomatic, or it can be a source of
Information embolization. Emboli break off from the plaque and travel through the
circulation to blood vessels in the brain. As the vessel gets smaller, they can
lodge in the vessel wall and restrict blood flow to parts of the brain which that
vessel supplies. This ischemia can either be temporary, yielding a transient
ischemic attack, or permanent resulting in a thromboembolic stroke.

Clinically, risk of stroke from carotid stenosis is evaluated by the presence or


absence of symptoms and the degree of stenosis on imaging.

Transient ischemic attacks (TIAs) are a warning sign, and are often followed by
severe permanent strokes, particularly within the first two days. TIAs by
definition last less than 24 hours and frequently take the form of a weakness or
loss of sensation of a limb or the trunk on one side of the body, or the loss of
sight (amaurosis fugax) in one eye. Less common symptoms are artery sounds
(bruits), or ringing in the ears (tinnitus).


Reference
Dr Khalid/Rabia



Q: 348 A 56yo woman with MS presents with drooping of the left side of her lips. She
also has loss of sensation over her face, hearing impairment and some in-
coordination of her movements. What is the most likely anatomical site
affected?
a. Cerebellum

214
CNS-System Wise 1700-by Sush and Team. 2016
Susmita, Asad, Manu, Saima, Zohaib, Savia, Shanu, Mona, Manisha, Sitara, Samreena, Sami and Komal

b. Cerebrum
c. Spinal cord
d. Brain stem
e. Optic nerve



Clincher(s)
A
B
C
D
E
KEY D
Additional Diseases of the brainstem can result in abnormalities in the function of cranial
Information nerves that may lead to visual disturbances, pupil abnormalities, changes in
sensation, muscle weakness, hearing problems, vertigo, swallowing and
speech difficulty, voice change, and co-ordination problems. Localizing
neurological lesions in the brainstem may be very precise, although it relies on
a clear understanding on the functions of brainstem anatomical structures and
how to test them.

Cranial nerves 3-12 from the brainstem and adjacent to cerebellum
So brain stem problems have high mortality

Reference
Dr Khalid/Rabia Need to understand and memorise about sensory and motor

Q: 55 A 75yo man collapsed while walking in his garden. He recovered fully within 30
mins with BP 110/80 mmHg and regular pulse of 70bpm. He has a systolic
murmur on examination. His activities have been reduced lately which he
attributes to old age. What is the definitive diagnostic inv that will assist you
with his condition?
a. ECG
b. Echo
c. 24h ECG monitoring
d. 24h BP monitoring
e. Prv CIN




Clincher(s) Exertional-Syncope, sys murmur
A
B Aortic stenosis – more likely in elderly. ii) hypertrophic cardiomyopathy – less
likely in this age as presentation may present in an earlier age.

215
CNS-System Wise 1700-by Sush and Team. 2016
Susmita, Asad, Manu, Saima, Zohaib, Savia, Shanu, Mona, Manisha, Sitara, Samreena, Sami and Komal

C
D
E
Key B. Echo.
Additional
Information
Reference
Dr Khalid/Rabia Aortic stenosis (AS)
Causes: Senile calcification is the commonest. Others: congenital (bicuspid
valve, William’s syndrome, rheumatic heart disease.

Presentation: Think of AS in any elderly person with chest pain, exertional
dyspnoea or syncope. The classic triad includes angina, syncope, and heart
failure (usually after age 60). Also: dyspnoea; dizziness; faints; systemic emboli
if infective endocarditis; sudden death.

Signs: Slow rising pulse with narrow pulse pressure (feel for diminished and
delayed carotid upstroke—parvus et tardus); heaving, non-displaced apex
beat; LV heave; aortic thrill; ejection systolic murmur (heard at the base, left
sternal edge and the aortic area, radiates to the carotids).
There may be an ejection click (pliable valve) or an S4 (said to occur more
often with bicuspid valves, but not in all populations).

Tests: ECG: P-mitrale, LVH with strain pattern; LBBB or complete AV block
(calcified ring).
CXR: LVH; calcified aortic valve post-stenotic dilatation of ascending aorta.

Echo: diagnostic.
Doppler echo can estimate the gradient across valves
Cardiac catheter can assess: valve gradient; LV function; coronary artery
disease
Management: If symptomatic, prognosis is poor without surgery.
If moderate-to-severe and treated medically, mortality can be as high as 50%
at 2yrs, therefore prompt valve replacement is usually recommended.
In asymptomatic patients with severe AS and a deteriorating ECG, valve
replacement is also recommended.

If the patient is not medically fit for surgery, percutaneous
valvuloplasty/replacement (TAVI = transcatheter aortic valve implantation)
may be attempted.



Q: 59 59. A 62yo woman complains of unsteadiness when walking. On examination
she has pyramidal weakness of her left lower limb and reduced pain and temp
sensation on right leg and right side of trunk up to the umbilicus. Joint position

216
CNS-System Wise 1700-by Sush and Team. 2016
Susmita, Asad, Manu, Saima, Zohaib, Savia, Shanu, Mona, Manisha, Sitara, Samreena, Sami and Komal

sense is impaired at her left great toe but is normal elsewhere. She has a
definite left extensor plantar response and the right plantar response is
equivocal. Where is the lesion?
a. Left cervical cord
b. Midline mid-thoracic cord
c. Right mid-thoracic cord
d. Left mid-thoracic cord
e. Left lumbo-sacral plexus



Clincher(s)
A
B
C
D Brown Sequard syndrome:
A lesion in one half of the spinal cord (due to hemisection or unilateral cord
lesion)- can be due to trauma. Gunshot, stab, metastatic tumour, infraction,
MS etc (will affect that level and anything below it)

s/s
one sided motor loss/ spastic hemiplegia
one sided fine touch and vibration sensation loss
and other sided pain and temp loss


Inv: MRI (best intial)

Rx: Underlying cause Rx, non penetrating: steorids (reduce swelling),
Penetrating: surgery

Ref: CRASH

TRACTS:

Sensory
DCML: Fine touch/vibration
Spinothalamic tracts:
Lat: pain amd temp
Ant: ccrude touch and pressure

Motor
Corticospinal (pyramidal) tract
Lat: Primarily limbs
Ant: axial and back muscles

217
CNS-System Wise 1700-by Sush and Team. 2016
Susmita, Asad, Manu, Saima, Zohaib, Savia, Shanu, Mona, Manisha, Sitara, Samreena, Sami and Komal

DCML Spinothalamic
Sensation Fine touch and vib Crude touch and pain
and temp
Decussation Medulla Spinal cord (lower)
Spinal injury Ispilat loss of fine touch Contralat loss of crude
and vibration below touch , pain temp
injury below injury
Sensory cortex injury Contralat loss of fine Contralat loss of crude
touch and vibration touch, pain and temp

E
KEY d. Left mid-thoracic cord.
Additional Khalid:
Information
Presentation: •Ipsilateral UMN weakness below the lesion (severed
corticospinal tract, causing spastic paraparesis, brisk reflexes, extensor
plantars) •Ipsilateral loss of proprioception and vibration (dorsal column
severed) •Contralateral loss of pain and temperature sensation (severed
spinothalamic tract which has crossed over
Causes: Bullet, stab, dart, kick, tumour, disc hernia, cervical spondylosis, MS,
neuroschistosomiasis, myelitis, septic emboli (eg meningococcal).
Imaging: MRI

Reference
Dr Khalid/Rabia Pain & temperature: carried by lateral spinothalamic tract… dicussate to the
opposite side within the spinal cord. Vibration, proprioception: carried by the
medial lemniscal system… decussate at the junction of pons and medulla.
Upper motor neurons in pyramidal tract also decussate close to medulla. So
from this we know that the lesion is on the left side. And since the symptoms
are below the umbillicus it rules out a cervical lesion. Lesion of lumbo sacral
plexus will impair the whole of the lower limb. So the correct answer is D.




Q: 62 A woman has electric pains in her face that start with the jaw and move
upwards. Her corneal reflexes are normal. What is the most likely dx?
a. Atypical face pain
b. Trigeminal neuralgia
c. Tempero-mandibular joint dysfunction
d. GCA
e. Herpes zoster


218
CNS-System Wise 1700-by Sush and Team. 2016
Susmita, Asad, Manu, Saima, Zohaib, Savia, Shanu, Mona, Manisha, Sitara, Samreena, Sami and Komal

Clincher(s)
A
B
1. Stabbing / electric facial pain caused by Trigeminal Neuralgia

(In Q: involving the face and it's muscles , which are mostly supplied by the 5th
CN ; Trigeminal nerve . So this CN is involved in this type of neuralgia. Maxillary
branch of Trigeminal).

2. Both Trigeminal Neuralgia and Herpetic Keratoconjunctivitis (H. Zoster
ophthalmicus) can cause absent corneal reflex.

C Shooting or electric pain is neurological. But TMJ dysfunction pain will be
mainly associated with movement and once occur may stay few moments.
Corneal reflex may be absent but may be this feature not always present.

D Giant cell arteritis (GCA) is a condition in which medium and large arteries,
usually in the head and neck, become inflamed. It's sometimes called temporal
arteritis because the arteries around the temples are usually affected.


• Symptoms:
• severe headaches, pain and tenderness over your temples and scalp.
• prominence or tenderness of the blood vessels at your temples.
• fatigue.
• flu-like symptoms.
• weight loss.
• loss of vision or double vision.
• pain in your jaw or tongue when you chew.

E Herpes zoster should have characteristic vesicles and it will be dermetomal in
distribution

KEY b. Trigeminal neuralgia.
Additional
Information
Reference
Dr Khalid/Rabia Trigerminal Neuralgia...Facial pains.

PRESENTATION: The episodes are sporadic and sudden and often like 'electric
shocks', lasting from a few seconds to several minutes. Pain is unilateral, brief,
stabbing, recurrent in the distribution of CN5. Can be provoked by light touch
to the face, eating, cold winds, or vibrations typically occurs after shaving,
brushing teeth.
Cause is a compression of CN5.

219
CNS-System Wise 1700-by Sush and Team. 2016
Susmita, Asad, Manu, Saima, Zohaib, Savia, Shanu, Mona, Manisha, Sitara, Samreena, Sami and Komal

No Investigations
TREATMENT: Carbamezapine is the first line. Rhizotomy (surgery) may also be
done



Q: 64 64. A 56yo man comes with hx of right sided weakness & left sided visual loss.
Where is the occlusion?
a. Ant meningeal artery
b. Mid meningeal artery
c. Mid cerebral artery
d. Carotid artery
e. Ant cerebral artery
f. Ant communicating artery



Clincher(s)
A
B
C Middle cerebral artery occlusion: paralysis or weakness of contralateral face
and arm (faciobracheal). Sensory loss of the contralateral face and arm.

D Carotid Artery occlusion:
PRESENTATION: Patients may present with TIAs or CVE (cardio vas event)s.
Typical symptoms are contralateral weakness or sensory disturbance,
ipsilateral blindness, and (if the dominant hemisphere is involved) dysphasia,
aphasia or speech apraxia.
Carotid bruit may or may not be present



E Anterior cerebral artery occlusion: paralysis or weakness of the contralateral
foot and leg. Sensory loss at the contralateral foot and leg.

KEY d. Carotid artery.
Additional
Information
Reference
Dr Khalid/Rabia Carotid Artery occlusion:
PRESENTATION: Patients may present with TIAs or CVE (cardio vas event)s.
Typical symptoms are contralateral weakness or sensory disturbance,
ipsilateral blindness, and (if the dominant hemisphere is involved) dysphasia,
aphasia or speech apraxia.
Carotid bruit may or may not be present
INVESTIGATIONS: For diagnosis: CAROTID ANGIOGRAPHY GOLD STANDARD.

220
CNS-System Wise 1700-by Sush and Team. 2016
Susmita, Asad, Manu, Saima, Zohaib, Savia, Shanu, Mona, Manisha, Sitara, Samreena, Sami and Komal

MR angio and angio CT can also be used.
Echo colour Doppler ultrasonography is the screening method of choice
TREATMENT: Medical: Antiplatelets, Anti HTN, Statins
Surgery: Carotid endartarectomy. Symptomatic patients with greater than 50%
stenosis and healthy, asymptomatic patients with greater than 60% stenosis
warrant consideration for carotid endarterectomy.



Q: 76 A 65yo HTN man presents with lower abdominal pain and back pain. An
expansive abdominal mass is palpated lateral and superior to the umbilicus.
What is the single most discriminating inv?
a. Laparascopy
b. KUB XR
c. Pelvic US
d. Rectal exam
e. Abdominal US


Clincher(s)
A
B
C
D
E The diagnosis is Abdominal aortic aneurism. Points in favour of AAA are i)
hypertension ii) abdominal pain iii) back pain iv) expansile abdominal mass
lateral and superior to the umbilicus.
KEY E. Abdominal US.
Additional
Information
Reference
Dr Khalid/Rabia RISK Factors include :
• Family Hx , tobacco smoking is an important factor.
• Male sex.
• Increasing age.
• Hypertension.
• Chronic obstructive pulmonary disease.
• Hyperlipidaemia

UNRUPTURED AAA is commonly asymptomatic and is an accidental finding.

Ruptured AAA may present with:
• Pain in the abdomen, back or loin - the pain may be sudden and severe.
• Syncope, shock or collapse:
• The degree of shock varies according to the site of rupture and whether
it is contained - eg, rupture into the peritoneal cavity is usually

221
CNS-System Wise 1700-by Sush and Team. 2016
Susmita, Asad, Manu, Saima, Zohaib, Savia, Shanu, Mona, Manisha, Sitara, Samreena, Sami and Komal

dramatic, with death before reaching hospital; whereas rupture into
the retroperitoneal space may be contained initially by a temporary
seal forming.
Inv:
Ultrasound is simple and cheap; it can assess the aorta to an accuracy of 3 mm.
It is used for initial assessment and follow-up.

SCANS :
CT Scan provides more anatomical details - eg, it can show the visceral
arteries, mural thrombus, the 'crescent sign' (blood within the thrombus,
which may predict imminent rupture) and para-aortic inflammation. CT with
contrast can show rupture of the aneurysm.

MRI angiography may be used.

Rx: If size exceeds 5.5cm, we go for surgery.


Q: 95 A 56yo lady has developed severe right sided headache which worsens
whenever she comes to bright light since the last 4 days. She feels nauseated,
but doesn’t vomit. What is the most likely
dx?
a. SAH
b. Brain tumor
c. Migraine
d. Cluster headache
e. Subdural headache


Clincher(s)
A A subarachnoid haemorrhage is an uncommon type of stroke caused by
bleeding on the surface of the brain. It is a very serious condition and can be
fatal.
The main symptom of a subarachnoid haemorrhage is a sudden and very
severe headache that is often described as a blinding pain, unlike anything
experienced before.
Other symptoms can include:
• a stiff neck
• being sick (vomit?)
• sensitivity to light
• blurred or double vision
• seizures (fits) or loss of consciousness (NHS)

B • Brain Tumour headache:
• Steady pain that is worse upon waking in the morning and gets better
within a few hours.

222
CNS-System Wise 1700-by Sush and Team. 2016
Susmita, Asad, Manu, Saima, Zohaib, Savia, Shanu, Mona, Manisha, Sitara, Samreena, Sami and Komal

• Persistent, non-migraine headache.
• May be accompanied by vomiting.
• May or may not be throbbing, depending on the location of the tumor.

C It is migraine without aura. There is presence of trigger (bright light) .

Criteria of migraine without aura:
≥5 headaches lasting 4-72 hours + nausea/vomiting (or photo/phono-phobia)

+ any 2 of: i) unilateral ii) pulsating iii) worsen by routine activity

[OHCM, 9th edition, page-462].
D Cluster headaches begin suddenly and without warning. The pain is very
severe and is often described as a sharp, burning or piercing sensation on one
side of the head. The pain is typically felt around the eye, temple and
sometimes face, and typically recurs on the same side for each attack.

Often people feel restless and agitated during an attack because the pain is so
intense, and may react by rocking, pacing or banging their head against the
wall.

They commonly also have at least one of the following associated symptoms:
• a red and watering eye

• drooping and swelling of one eyelid


• a smaller pupil in one eye
• a sweaty face
• a blocked or runny nostril
• a red ear

These attacks generally last between 15 minutes and three hours, and typically
occur between one and eight times a day. (Ref: NHS)

Typically, a number of attacks occur over several weeks - a bout (cluster) of
attacks. The headaches then usually go for weeks, months or years until a next
cluster of headaches develops. (ref: pt info)

E The symptoms of a subdural haematoma can develop soon after a severe head
injury, or gradually over days or weeks after a more minor head injury.
Sometimes you may not remember hitting your head at all.
Symptoms of a subdural haematoma can include:
• a headache that keeps getting worse
• feeling and being sick
• confusion

223
CNS-System Wise 1700-by Sush and Team. 2016
Susmita, Asad, Manu, Saima, Zohaib, Savia, Shanu, Mona, Manisha, Sitara, Samreena, Sami and Komal

• personality changes, such as being unusually aggressive or having rapid
mood swings
• feeling drowsy and finding it difficult to keep your eyes open
• speech problems, such as slurred speech
• problems with your vision, such as double vision
• paralysis (loss of movement) on one side of the body
• problems walking and frequent falls
• seizures (fits)
• loss of consciousness (NHS)

Key C. Migraine.
Additional
Information
Reference
Dr Khalid/Rabia It should be noted that as a general rule 5-HT receptor agonists are used in the
acute treatment of migraine whilst 5-HT receptor antagonists are used in
prophylaxis. NICE produced guidelines in 2012 on the management of
headache, including migraines. (update the below information: e.g. from
other Q/A)

Acute treatment
• first-line: offer combination therapy with an oral triptan and an NSAID,
or an oral triptan and paracetamol
• for young people aged 12-17 years consider a nasal triptan in
preference to an oral triptan
• if the above measures are not effective or not tolerated offer a non-
oral preparation of metoclopramide* or prochlorperazine and consider
adding a non-oral NSAID or triptan

Prophylaxis
• prophylaxis should be given if patients are experiencing 2 or more
attacks per month. Modern treatment is effective in about 60% of
patients.
• NICE advise either topiramate or propranolol 'according to the person's
preference, comorbidities and risk of adverse events'. Propranolol
should be used in preference to topiramate in women of child bearing
age as it may be teratogenic and it can reduce the effectiveness of
hormonal contraceptives
• if these measures fail NICE recommend 'a course of up to 10 sessions of
acupuncture over 5-8 weeks' or gabapentin
• NICE recommend: 'Advise people with migraine that riboflavin (400 mg
once a day) may be effective in reducing migraine frequency and
intensity for some people'
• for women with predictable menstrual migraine treatment NICE
recommend either frovatriptan (2.5 mg twice a day) or zolmitriptan
(2.5 mg twice or three times a day) as a type of 'mini-prophylaxis'

224
CNS-System Wise 1700-by Sush and Team. 2016
Susmita, Asad, Manu, Saima, Zohaib, Savia, Shanu, Mona, Manisha, Sitara, Samreena, Sami and Komal

• pizotifen is no longer recommend. Adverse effects such as weight gain
& drowsiness are common

*caution should be exercised with young patients as acute dystonic reactions


may develop


Q: 109 A 33yo male involved in a street fight presents with bruises and deformity in
the upper part of his leg. XR shows fx of the neck of fibula. What is the single
most associated nerve injury?
a. Sciatic nerve
b. Gluteal nerve
c. Musculocutaneous nerve
d. Lateral peroneal nerve
e. Tibial nerve
f. Femoral nerve


Clincher(s)
A
B
C
D Its Common peroneal nerve injury (due to fibula fracture) leading to foot drop.
Peroneal nerve injury is called foot drop/drop foot. Peroneal nerve is also the
‘fibular nerve’. (Johns Hopkins)

Common peroneal (from the schiatic nerve) is bifurcated to superficial and
deep
These nerves provide sensation to the anterior (front) and lateral (side) parts
of the legs and to the top of the feet. They innervate muscles in the legs which
lift the ankle and toes upward (dorsi flexion). (JH)
Lateral peroneal nerve is other name of superficial peroneal nerve. It causes
foot eversion. (see picture at bottom)

225
CNS-System Wise 1700-by Sush and Team. 2016
Susmita, Asad, Manu, Saima, Zohaib, Savia, Shanu, Mona, Manisha, Sitara, Samreena, Sami and Komal




E
KEY Ans. is D. Lateral peroneal nerve.
Additional
Information
Reference
Dr Khalid/Rabia

226
CNS-System Wise 1700-by Sush and Team. 2016
Susmita, Asad, Manu, Saima, Zohaib, Savia, Shanu, Mona, Manisha, Sitara, Samreena, Sami and Komal



Q: 145 A 53yo woman presented with pain in the eye, blurry vision and clumsiness for
3 months. She has a hx of difficulty in swallowing and weakness in her right
upper limb 2y ago. What is the inv of choice?
a. CSF analysis
b. EEG
c. EMG
d. MRI brain
e. Visual evoked response test


Clincher(s)
A
B
C

227
CNS-System Wise 1700-by Sush and Team. 2016
Susmita, Asad, Manu, Saima, Zohaib, Savia, Shanu, Mona, Manisha, Sitara, Samreena, Sami and Komal

D (NHS) Diagnosis is multiple sclerosis. When MS damages the myelin coating
around the nerve fibres that carry messages to and from your brain,
symptoms can occur in any part of your body.
There are many different symptoms of MS and they affect each person
differently. Some of the most common symptoms include:
• extreme tiredness (fatigue)-common
• numbness and tingling
• Visual problem: e.g. blurry/loss of vision/colour blindness/ pain in both
eyes/ nystagmus- invol eye movements(In around one in five cases of
MS, the first noticeable symptom due to optic neuritis : in ONE of the
eyes. )
• problems with mobility and balance (ataxia/vertigo/tremor)
• muscle weakness and tightness / spasticity (stiff and resistant to
movement)
• half of people experience pain (neuropathic- stabbing/burning or
musculosketetal)
• Cog dysfunction-problem with thinking, learning, planning
• depression/anxiety
• erectile dysfunction
• urge incontinence
• dysphagia
• bowel constipation/incontinece


Cause: Autoimmnue This means your immune system mistakes the
myelin for a foreign substance and attacks it.

The myelin becomes inflamed in small patches (called plaques or
lesions), which can be seen on an MRI scan.

Rx: NICE: http://www.nice.org.uk/guidance/CG8
The NICE guideline states that if you have MS, you should have access
to a specialist neurological rehabilitation team. Care for people with MS
using a coordinated multidisciplinary approach.


E
KEY D. MRI brain.
Additional
Information
Reference
Dr Khalid/Rabia Multiple sclerosis: Discrete plaques of demyelination occur at multiple CNS
sites. Early exposure to sunlight/vit. D is important, and vit. D status relates to
prevention of MS,
PRESENTATION: Usually monosymptomatic: unilateral optic neuritis (pain on
eye

228
CNS-System Wise 1700-by Sush and Team. 2016
Susmita, Asad, Manu, Saima, Zohaib, Savia, Shanu, Mona, Manisha, Sitara, Samreena, Sami and Komal

movement and rapid central vision); numbness or tingling in the limbs; leg
weakness; brainstem or cerebellar symptoms (eg diplopia, ataxia). The disease
has a relapsing - remitting course.


INVESTIGATIONS: This is clinical, as no test is pathognomonic. MRI is sensitive
but not specific for plaque detection.
CSF: Oligoclonal bands of IgG on electrophoresis suggest CNS inflammation but
does not confirm MS.. Delayed visual, auditory, and somatosensory evoked
potentials.
MANAGEMENT: Stress free life. Give vit. D to achieve serum 25(OH)D levels of
50nmol/L
Methylprednisolone shortens relapse doesnt alter overall prognosis.
Beta interferon reduces replaces by 30% but does not reduce overall disability
Monoclonal antibodies: Alemtuzumab and natalizumab reduce relapses by
68%.
Glatiramer in secondary progressive
Azathioprine in relapsing-remiting
Palliation: Spasticity: Baclofen, diazepam, dantrolene or tizanidine.
Tremor: Botulinum toxin
Urgency/frequency: If post-micturition residual urine >100mL, teach
intermittent self-catheterization; if <100mL, try tolterodine



Q: 153 A 44yo pt comes with right hemiparesis. Exam: left sided ptosis and left dilated
pupil. Where is the lesion?
a. Cerebral infarct
b. Cerebellar infarct
c. Medulla oblongata
d. Pons
e. Midbrain

229
CNS-System Wise 1700-by Sush and Team. 2016
Susmita, Asad, Manu, Saima, Zohaib, Savia, Shanu, Mona, Manisha, Sitara, Samreena, Sami and Komal



Clincher(s)
A
B
C Common syndromes related to the medulla and pons are Lateral medullary
syndrome and Lateral pontine syndrome.

They both present with
1.Ipsilateral horners syndrome (ptosis, myosis, anhydrosis)
2. Ipsilateral cerebellar ataxia and
3. Contralateral loss of pain and temp.

The two conditions are differentiated by the fact that the Medulla also
presents with dysphagia, dysarthria and dysphonia, whereas, the Pons has
ipsilateral facial nerve involvement (LMN facial palsy, loss of taste ant. 2/3
tongue & reduced lacrimation and salivation).

Lateral medullary syndrome = Posterior inferior cerebellar artery (PICA).
Lateral pontine syndrome = Anterior inferior cerebellar artery (AICA) (Ref: FB)


D See above
E Midbrain. The occulomotor nerve nucleus is located in the midbrain. When the
brainstem is affected (occulomotor nerve damage) it causes ipsilateral cranial
nerve injury and contralateral hemiparesis.


Weber’s syndrome (superior alternating hemiplegia)

• Ipsilateral oculomotor nerve palsy with contralateral hemiplegia,

• due to infarction of one-half of the midbrain,

• after occlusion of the (paramedian branches of the basilar or)
posterior cerebral arteries. (Dr Khalid)

KEY E. Midbrain.

Additional
Information
Reference
Dr Khalid/Rabia Ptosis + miosis = horners syndrome Ptosis + mydriasis= oculomotor nerve
palsy.
CN3 nucleus lies in the midbrain.
Fore brain: CN 1,2

230
CNS-System Wise 1700-by Sush and Team. 2016
Susmita, Asad, Manu, Saima, Zohaib, Savia, Shanu, Mona, Manisha, Sitara, Samreena, Sami and Komal

Mid brain: CN 3,4
Pons: CN 5,6,7,8
Medulla: CN 9,10,12





Q: 164 A 23yo male has a tonic clonic seizure whilst at college. His GCS (Glasgow
comma scale) is 12, BP=120/77mmHg,
HR=99bpm. What is the most appropriate inv for his condition?
a. CT
b. MRI
c. Serum blood glucose
d. Serum drug levels


Clincher(s)
A in seizure CT scan is not the appropriate investigation except with the h/o
trauma or when suspecting Hhg.

EEG is what is done for seizures not ct


B
C Should be blood glucose, then drug level and ct /mri. Question is not asking
definitive so it's blood glucose.

we need to rule out hypoglycaemia first ..it can cause drop in GCS level aswell.
it can also cause seizure.

This case will be treated as status epilepticus. Normally status occurs in a


patient with known epilepsy but here no such history is given. Even in a person
who presents with his first seizure we need to look for hypoglycemia first
before going onto later tests.

[it is also possible that he may have taken drug, even though first we have to
do serum glucose as its presence can be very easily managed and it needs
urgent management to save life. If it is excluded then we can look for other
causes which may be not fatal in short time as hypoglycaemia].
D
E
KEY C. Serum blood glucose
Additional • The tonic-clonic seizure (old- grand mal) is what most people think of
Information when they think of a convulsive seizure.
• they combine the characteristics of tonic seizures (stiffen) and clonic

231
CNS-System Wise 1700-by Sush and Team. 2016
Susmita, Asad, Manu, Saima, Zohaib, Savia, Shanu, Mona, Manisha, Sitara, Samreena, Sami and Komal

seizures (jerking).
• A person loses consciousness, muscles stiffen, and jerking movements
are seen.
• These types of seizures usually last 1 to 3 minutes and take much
longer for a person to recover.
• A tonic-clonic seizure lasting more than 5 minutes is a medical
emergency.

Epilepsy Causes (NHS):

Idiopathic and symptomatic

Symptomatic:

• cerebrovascular disease (problems with the blood vessels that supply


the brain) – such as a stroke or subarachnoid haemorrhage
• brain tumours
• severe head injuries
• drug abuse and alcohol misuse
• infections that can damage the brain – such as meningitis
• problems during birth that cause a baby to be deprived of oxygen –
such as the umbilical cord getting twisted or compressed during labour
• some parts of the brain not developing properly
Triggers:
• stress
• lack of sleep
• drinking alcohol
• some medications and illegal drugs
• in women, monthly periods
• flashing lights (this is an uncommon trigger that affects only 5% of
people with epilepsy, and is known as photosensitive epilepsy)
Inv:
EEG (An EEG test can detect unusual brain activity associated with epilepsy by
measuring the electrical activity of your brain through electrodes placed on
your scalp.)

MRI (It can be useful in cases of suspected epilepsy because it can often detect
possible causes of the condition, such as defects in the structure of your brain
or the presence of a brain tumour.)

232
CNS-System Wise 1700-by Sush and Team. 2016
Susmita, Asad, Manu, Saima, Zohaib, Savia, Shanu, Mona, Manisha, Sitara, Samreena, Sami and Komal

RX:
Treatment for epilepsy is used to control seizures, although not everyone with
the condition will need to be treated. Anti-epileptic drugs (AEDs) are usually
the first choice of treatment for epilepsy. About 70% of people with the
condition are able to control their seizures with AEDs. (see epi drugs)

Reference
Dr Khalid/Rabia Status epilepticus:

This means seizures lasting for >30min, or repeated seizures without
intervening consciousness.
Also consider eclampsia in mind if the patient is female and the abdomen is
distended.

Investigations
• Bedside glucose, the following tests can be done once treatment has started:
lab glucose,
ABG, U&E, Ca2+, FBC, ECG.
• Consider anticonvulsant levels, toxicology screen, LP, culture blood and
urine, EEG,
CT, carbon monoxide level. • Pulse oximetry, cardiac monitor.

Treatment: Try to control seizure in less than 20mins as there could be
permanent brain damage.
1. IV lorazepam:0.1mg/kg. Repeat if no response in 10mins. Be careful about
respiratory depression. If there is no IV access give PR Diazepam.
2.Buccal midazolam: Alternative oral route. squirt half the volume between
the lower gum and the cheek on each side.
3.Phenytoin infusion: 15–20mg/kg IVI (roughly 1g if 60kg, and 1 . 5g if 80kg;
max 2g. 2nd line! Dont use if bradycardic or hypotensiv. ECG monitoring is
recommended.
Diazepam infusion: eg 100mg in 500mL of 5% dextrose. It is most unusual for
seizures to remain unresponsive following this. If they do, allow the idea to
pass through your mind that they could be pseudoseizures, particularly if there
are odd features (pelvic thrusts; resisting attempts to open lids and your
attempts to do passive movements; arms and legs flailing around).
4. Dexamethasone: 10mg IV if vasculitis/cerebral oedema (tumour) possible.
5. General anaesthesia: For refractory status: get anaesthetist/ICU involved
early

233
CNS-System Wise 1700-by Sush and Team. 2016
Susmita, Asad, Manu, Saima, Zohaib, Savia, Shanu, Mona, Manisha, Sitara, Samreena, Sami and Komal

234

Você também pode gostar